STUDY-MODE - Practice Exam 4 Flashcards

1
Q

Carl Jung identified which of the following as the source of psychic energy?
Select one:

A. libido
B. archetypes
C. persona
D. personal unconscious

A

Jung believed that all psychic phenomena are manifestations of energy.

a. CORRECT In contrast to Freud, Jung described libido as the source of all psychic energy (rather than as the source of sexual energy only).
b. Incorrect Archetypes are inherited images, ideas, and emotions that reside in the collective unconscious and predispose us to view the world in certain ways.
c. Incorrect As defined by Jung, the persona refers to the aspects of oneself (usually the ideal aspects of oneself) that are presented to others.
d. Incorrect Jung distinguished between the collective and the personal unconscious and defined the latter as the aspect of the unconscious that contains lost or repressed perceptions of personally experienced events.

The correct answer is: libido

How well did you know this?
1
Not at all
2
3
4
5
Perfectly
2
Q

When working with an African American family, the best approach is likely to be which of the following?
Select one:

A. focusing initially on parent-child subsystems
B. relying on nondirective, nonstructured techniques
C. adopting a multisystems approach
D. working initially with the parents only

A

Because of the impact of environmental factors on African Americans, therapy is usually most effective when it takes into account social, political, socioeconomic, and other contextual factors.

a. Incorrect See explanation for response c.
b. Incorrect See explanation for response c.
c. CORRECT A multisystems (ecostructural) approach has been recommended by several experts for therapists working with African American families (e.g., Aponte, 1994; Boyd-Franklin, 1989). Information about this approach is provided in the Clinical Psychology chapter of the written study materials.
d. Incorrect See explanation for response c.

The correct answer is: adopting a multisystems approach

How well did you know this?
1
Not at all
2
3
4
5
Perfectly
3
Q

Role disputes, role transitions, unresolved grief, and interpersonal deficits are the primary targets of:
Select one:

A. interpersonal therapy.
B. reality therapy.
C. solution-focused therapy.
D. narrative therapy.

A

Of the therapies listed in the answers, only one explicitly targets the problem areas listed in the question.

a. CORRECT Interpersonal therapy (IPT) is a brief structured manual-based therapy that was originally developed as a treatment for depression but has since been applied to other disorders. Its primary targets are the four problem areas listed in this question.
b. Incorrect See explanation for response a.
c. Incorrect See explanation for response a.
d. Incorrect See explanation for response a.

The correct answer is: interpersonal therapy.

How well did you know this?
1
Not at all
2
3
4
5
Perfectly
4
Q

From the perspective of structural family therapy, a therapist’s failure to __________ is likely to produce resistance among family members.
Select one:

A. form a therapeutic triangle
B. join the family
C. act as a “blank screen”
D. remain disengaged from the family’s emotional system

A

Familiarity with Minuchin’s terminology would have helped you choose the correct answer to this question.

a. Incorrect Bowen (not Minuchin) forms a therapeutic triangle with two family members in therapy.
b. CORRECT Minuchin considers joining to be a prerequisite for restructuring the family.
c. Incorrect Minuchin does not view the therapist’s role to be that of a “blank screen.”
d. Incorrect The “family emotional system” is a term most associated with Bowen.

The correct answer is: join the family

How well did you know this?
1
Not at all
2
3
4
5
Perfectly
5
Q

During your first session with a Japanese client, you decide that your primary goals are to normalize the client’s problems and instill a sense of hope. According to Sue and Zane (1987), these goals are:
Select one:

A. examples of “gift-giving.”
B. ways of establishing ascribed credibility.
C. ways of fostering “amae.”
D. manifestations of “cultural expressiveness.”

A

Sue and Zane argue that therapy outcome for members of minority groups is enhanced by two factors – credibility and giving [S. Sue and N. Zane, The role of culture and cultural techniques in psychotherapy: A critique and reformulation, American Psychologist, 42(1), 37-45, 1987].

Answer A is correct. Gift-giving is an important ritual in interpersonal relations in Asian cultures, but Sue and Zane feel that, in therapy, it is useful for other minority clients as well, especially for reducing premature termination from treatment. In therapy, giving can take several forms including the normalization of the client’s problems and installation of hope.

The correct answer is: examples of “gift-giving.”

How well did you know this?
1
Not at all
2
3
4
5
Perfectly
6
Q

For Irvin Yalom (1985), seeing a client in both individual therapy and group therapy is inadvisable because:
Select one:

A. the client will spend too much time in individual therapy discussing problems that arise during the group.
B. the client will terminate individual therapy prematurely because of the benefits he/she is deriving from the group.
C. participation in individual therapy may decrease the client’s involvement and participation in the group.
D. participation in both individual and group therapy is likely to be too anxiety-evoking for most clients.

A

Yalom (1985), an expert on group therapy, has explicitly addressed this issue.

a. Incorrect See explanation for response c.
b. Incorrect See explanation for response c.
c. CORRECT Yalom generally discourages seeing a client in both individual and group therapy because he feels that the client’s participation in individual therapy will detract from his participation in the group.
d. Incorrect While this may be true for some clients, it is not the primary objection of Yalom and other experts.

The correct answer is: participation in individual therapy may decrease the client’s involvement and participation in the group.

How well did you know this?
1
Not at all
2
3
4
5
Perfectly
7
Q

During the first therapy session with a husband and wife who are experiencing marital problems, a therapist requests that, during the next week, they identify things in their relationship that they want to continue. Most likely, this therapist is a practitioner of:
Select one:

A. psychoeducational family therapy.
B. behavioral marital therapy.
C. interpersonal therapy.
D. solution-focused therapy.

A

The therapist has given the couple a task that will help them focus on the positive aspects of their relationship.

a. Incorrect See explanation for response d.
b. Incorrect See explanation for response d.
c. Incorrect See explanation for response d.
d. CORRECT In the context of solution-focused therapy, this assignment is an example of a “formula task.” Its purpose is to help the couple identify positive aspects of their relationship that, in turn, can lead to solutions to their marital difficulties.

The correct answer is: solution-focused therapy.

How well did you know this?
1
Not at all
2
3
4
5
Perfectly
8
Q
In her discussion of African American middle-class families, Boyd-Franklin (1989) notes that, in terms of family roles, these families tend to be:
Select one:

A. patriarchal.
B. matriarchal.
C. egalitarian.
D. linear.

A

Nancy Boyd-Franklin discusses this issue in her book Black Families in Therapy: A Multisystems Approach, New York, Guilford Press, 1989.

a. Incorrect See explanation for response c.
b. Incorrect See explanation for response c.
c. CORRECT The structure of the African American middle-class family often reflects equal sharing of power by the spouses as well as more permeable boundaries in general. For example, these families are open to influences from the extended family and the African American community.
d. Incorrect See explanation for response c.

The correct answer is: egalitarian.

How well did you know this?
1
Not at all
2
3
4
5
Perfectly
9
Q

Which of the following illustrates “diagnostic overshadowing”?
Select one:

A. A therapist lists an inaccurate diagnosis on an insurance form because she knows the insurance company will be unwilling to pay for therapy for the correct diagnosis.
B. A therapist does not recognize a co-existing clinical disorder in an intellectually disabled patient because of the salience of the intellectual disability.
C. A therapist misdiagnoses a patient because the salience of an atypical symptom causes her to overlook the patient’s other symptoms.
D. A therapist’s own cultural beliefs and values impede her ability to recognize how cultural differences impact diagnosis and treatment.

A

Diagnostic overshadowing was originally described by S. Reiss and colleagues in connection with the assessment of people with an intellectual disability (Emotional disturbance and mental retardation: Diagnostic overshadowing, American Journal on Mental Deficiency, 87, 396-402, 1982). It has since been applied to other situations and diagnoses.

a. Incorrect See explanation for response b.
b. CORRECT Overshadowing occurs when the salience of one disorder or condition “overshadows” consideration or recognition of another disorder.
c. Incorrect See explanation for response b.
d. Incorrect See explanation for response b.

The correct answer is: A therapist does not recognize a co-existing clinical disorder in an intellectually disabled patient because of the salience of the intellectual disability.

How well did you know this?
1
Not at all
2
3
4
5
Perfectly
10
Q

According to Sue and Zane (1987), in terms of ensuring good treatment outcomes, the most important factor in culturally sensitive therapy is:
Select one:

A. demonstrating cultural knowledge.
B. using culture-specific techniques.
C. using techniques that establish credibility.
D. being authentic and empathic.

A

Sue and Zane note that, when discussing cross-cultural counseling, most authors focus on cultural knowledge and the use of culture-specific techniques. These investigators argue, however, that these factors are only the means to an end, which is establishing credibility [S. Sue and N. Zane, The role of culture and cultural techniques in psychotherapy: A critique and reformulation, American Psychologist, 42(1), 37-45, 1987].

a. Incorrect Sue and Zane recognize the importance of cultural knowledge but argue that it is not sufficient to ensure positive outcomes in cross-cultural counseling.
b. Incorrect These authors also recognize the usefulness of culture-specific techniques but, again, view them as insufficient by themselves.
c. CORRECT According to Sue and Zane, the therapist’s credibility is a key determinant of therapy outcome. Cultural knowledge and the use of culture-specific techniques are beneficial only to the extent that they contribute to the therapist’s credibility.
d. Incorrect Sue and Zane don’t address the issues of authenticity and empathy.

The correct answer is: using techniques that establish credibility.

How well did you know this?
1
Not at all
2
3
4
5
Perfectly
11
Q

Savin-Williams and Diamond (2000) investigated the sexual identity trajectories among sexual-minority youth and found that:
Select one:

A. adolescent females had an earlier onset of all milestones.
B. adolescent males had an earlier onset of all milestones.
C. adolescent females had an earlier onset of all milestones except first same-sex attraction.
D. adolescent males had an earlier onset of all milestone except first disclosure of sexual orientation to another person.

A

R. C. Savin-Williams and L. M. Diamond compared the sexual identity trajectories of male and female sexual-minority youth in terms of four milestones - first same-sex attraction, self-labeling, first same-sex sexual contact, and first disclosure [Sexual identity trajectories among sexual-minority youths: Gender comparisons, Archives of Sexual Behavior, 29(6), 607-627, 2000].

a. Incorrect See explanation for response d.
b. Incorrect See explanation for response d.
c. Incorrect See explanation for response d.
d. CORRECT Males had an earlier onset than females for all milestones except first disclosure of sexual orientation to another person (age at first disclosure was not significantly different for males and females).

The correct answer is: adolescent males had an earlier onset of all milestone except first disclosure of sexual orientation to another person.

How well did you know this?
1
Not at all
2
3
4
5
Perfectly
12
Q

A therapy client states that she believes she may have been sexually abused as a child but has no explicit memories of the abuse. She asks her therapist - who often uses hypnosis in his practice - to hypnotize her as a means of retrieving those memories. The psychologist is familiar with the statement issued by the APA’s Working Group on Investigation of Memories of Child Abuse. Consequently, the psychologist will:
Select one:

A. agree to hypnotize the woman but be careful not to ask leading questions.
B. agree to hypnotize the woman and then attempt to corroborate any memories of abuse the woman recalls while under hypnosis.
C. advise the woman that hypnosis is not an appropriate procedure for retrieving memories because it is likely to produce false recollections of abuse.
D. refer the woman to another psychologist who uses hypnosis but is unfamiliar with the woman’s suspicions so that the procedure will be more objective.

A

The use of hypnosis for the purpose of recalling repressed memories is addressed in the document titled Final conclusions of the American Psychological Association Working Group on Investigation of Memories in Childhood Abuse [Psychology, Public Policy, and Law, 4(4), 933-940, 2000].

a. Incorrect See explanation for response c.
b. Incorrect See explanation for response c.
c. CORRECT This response is most consistent with the conclusions drawn in the above-named document. (It is also consistent with research showing that hypnosis tends to elicit false memories.)
d. Incorrect See explanation for response c.

The correct answer is: advise the woman that hypnosis is not an appropriate procedure for retrieving memories because it is likely to produce false recollections of abuse.

How well did you know this?
1
Not at all
2
3
4
5
Perfectly
13
Q

Which of the following programs best exemplifies a primary prevention program?
Select one:

A. use of personality tests as screening devices to identify high-risk individuals so those individuals can be provided with appropriate treatment
B. use of public education programs to inform the public about the negative effects of alcohol abuse
C. training of primary school teachers to recognize early signs of behavior disorders
D. use of educational programs to improve community attitudes toward former mental patients

A

There are three types of prevention programs: Primary prevention programs are designed to prevent a mental disorder from occurring; secondary prevention programs are designed to detect disorders early so that appropriate treatments can be applied; and tertiary prevention programs are aimed at preventing the recurrence or worsening of existing disorders.

a. Incorrect When a program involves identifying high-risk individuals so that those individuals can be provided with an intervention, it is classified as a secondary prevention program.
b. CORRECT An educational program designed to prevent alcohol abuse before it occurs is an example of a primary prevention program.
c. Incorrect Prevention programs that involve the early identification of disorders are classified as secondary prevention programs.
d. Incorrect A program designed to improve community attitudes toward former mental patients (presumably to ensure that the patients remain in the community) is an example of a tertiary prevention program.

The correct answer is: use of public education programs to inform the public about the negative effects of alcohol abuse

How well did you know this?
1
Not at all
2
3
4
5
Perfectly
14
Q

D. W. Sue (1978) describes “worldview” in terms of two independent dimensions – locus of control and locus of responsibility. From this perspective, an Anglo-American therapist with an internal locus of control and internal locus of responsibility is most likely to experience problems when working with an African American client who has an:
Select one:

A. external locus of control and internal locus of responsibility.
B. external locus of control and external locus of responsibility.
C. internal locus of control and external locus of responsibility.
D. internal locus of control and internal locus of responsibility.

A

Answer C is correct. Sue uses his model of worldview to describe issues and problems that Anglo-American therapists may face when working with clients from culturally diverse backgrounds. D. W. Sue and D. Sue discuss this issue in Counseling the culturally different, New York, John Wiley & Sons, 1999. They conclude that an IC-ER worldview is likely to “pose the most difficult problems for the IC-IR white therapist” (p. 180) because the client is likely to challenge the therapist’s authority, view the therapist as part of the “establishment that has oppressed minorities” (p. 181), be reluctant to self-disclose, and prefer adopting an active role in the therapy process.

The correct answer is: internal locus of control and external locus of responsibility.

How well did you know this?
1
Not at all
2
3
4
5
Perfectly
15
Q

Cross’s (1991, 2001) Black Racial Identity Development Model includes four stages, the last of which is:
Select one:

A. autonomy.
B. internalization.
C. incorporation.
D. integrative awareness.

A

The current version of Cross’s model includes the following four stages:

PRE-ENCOUNTER
ENCOUNTER
IMMERSION-EMMERSION INTERNALISATION

For additional information on this model, see the Clinical Psychology chapter of the written study materials.

a. Incorrect See explanation for response b.
b. CORRECT The final stage of Cross’s model - i.e., internalization - is characterized by the development of “inner security” as identity conflicts are resolved along with increasing biculturalism/multiculturalism.
c. Incorrect See explanation for response b.
d. Incorrect See explanation for response b.

The correct answer is: internalization.

How well did you know this?
1
Not at all
2
3
4
5
Perfectly
16
Q

A person-centered (Rogerian) therapist would most likely respond to a client’s transference by:
Select one:

A. interpreting it.
B. challenging it.
C. disregarding it.
D. reflecting it.

A

Transference is not expected to develop in person-centered therapy, at least not at the intense level that it does in psychodynamic therapy.

a. Incorrect See explanation for response c.
b. Incorrect See explanation for response c.
c. CORRECT Transference in person-centered therapy is essentially accepted and disregarded rather than interpreted as it is in many other forms of therapy. This is consistent with the here-and-now approach of person-centered therapy.
d. Incorrect See explanation for response c.

The correct answer is: disregarding it.

How well did you know this?
1
Not at all
2
3
4
5
Perfectly
17
Q

Under hypnosis, a person is LEAST likely to:
Select one:

A. experience alterations of memory, perception, and mood
B. believe just about anything, no matter how preposterous.
C. do things he/she is otherwise fundamentally opposed to doing.
D. stop feeling pain (even severe pain) to a greater degree than if he/she had been given a pain killer.

A

Hypnosis has been defined as a state of increased suggestibility brought about by certain procedures.

a. Incorrect Hypnosis may involve experiencing alterations of memory, perception, and mood in response to suggestions.
b. Incorrect Hypnotized people are typically willing to accept even absurd suggestions (e.g., the suggestion that there is a pink elephant in the room).
c. CORRECT Although hypnotized people are suggestible, they will not engage in behaviors that they would otherwise refuse to perform.
d. Incorrect This is true for at least some individuals. The research has found that hypnotism can be a very effective pain killer; sometimes even more effective than acupuncture, morphine, aspirin, and other pain relievers.

The correct answer is: do things he/she is otherwise fundamentally opposed to doing.

How well did you know this?
1
Not at all
2
3
4
5
Perfectly
18
Q

Which of the following minimizes deviation and thereby helps a family (or other system) maintain the status quo?
Select one:

A. negative feedback
B. positive feedback
C. joining
D. paradox

A

From a family systems perspective, feedback refers to the return or provision of information to the system about the consequences of an event or action.

a. CORRECT Negative feedback acts to maintain a system’s current level of functioning or homeostasis.
b. Incorrect Positive feedback produces a change in the system.
c. Incorrect Joining is a technique used by structural family therapists and involves blending with the family by, for example, adopting its communication and affective style.
d. Incorrect The question does not describe paradox.

The correct answer is: negative feedback

How well did you know this?
1
Not at all
2
3
4
5
Perfectly
19
Q

Cross’s (1991) Black Racial Identity Development Model predicts that an African American in the pre-encounter stage of identity development will:
Select one:

A. exhibit a great deal of confusion and turmoil related to race.
B. have adopted a mainstream (White) identity.
C. be hostile to members of all racial/ethnic minority groups.
D. exhibit “healthy cultural paranoia”.

A

Cross’s model divides the pre-encounter stage into two substages: assimilation and anti-Black.

a. See explanation for response b.
b. CORRECT Adoption of a mainstream identity is characteristic of the assimilation substage, which is the first substage in the pre-encounter stage.
c. Incorrect This is not predicted by Cross’s model.
d. Incorrect This is also not predicted by Cross’s model.

The correct answer is: have adopted a mainstream (White) identity.

How well did you know this?
1
Not at all
2
3
4
5
Perfectly
20
Q

Issues related to attachment, safety, and security would be of most interest to a practitioner of:
Select one:

A. object relations family therapy.
B. Milan systemic family therapy.
C. reality therapy.
D. Adlerian therapy.

A

Of the therapeutic approaches listed, only one prioritizes the importance of attachment, safety, and security.

a. CORRECT Early attachment and issues related to safety and security are key concerns for practitioners of object relations psychotherapy.
b. Incorrect See explanation above.
c. Incorrect See explanation above.
d. Incorrect See explanation above.

The correct answer is: object relations family therapy.

How well did you know this?
1
Not at all
2
3
4
5
Perfectly
21
Q

A client describes a recurring dream of his to his therapist in which the client’s parents are standing on top of a stairway and refuse to let him climb up to the bedroom. If the therapist is a practitioner of Gestalt Therapy, she will probably:
Select one:

A. analyze the content of the dream.
B. ask the client to discuss his relationship with his parents.
C. see the dream as indicative of the client’s current progress in therapy.
D. see the dream as representing parts of the client’s personality that need to be integrated.

A

The major goals of Gestalt Therapy are to help a client achieve self-responsibility and function as a systematic whole. Dream work is one of the many techniques Gestaltians use.

a. Incorrect See explanation for response d.
b. Incorrect See explanation for response d.
c. Incorrect See explanation for response d.
d. CORRECT A goal of Gestalt Therapy is to integrate the separate, fragmented parts of a client’s personality into a unified whole. In line with this goal, Gestaltians see elements of recurring dreams as representations of parts of the self that have not been fully accepted and integrated.

The correct answer is: see the dream as representing parts of the client’s personality that need to be integrated.

How well did you know this?
1
Not at all
2
3
4
5
Perfectly
22
Q

What are the three approaches to healthcare?
Select one:

A. The Group Model, Open Model, and Regency Model.
B. The Stanford Model, Dual Model, and House Model.
C. The Open Model, Bismarck Model, and Balanced Model.
D. The Private Model, Beveridge Model, and Bismarck Model.

A

The correct answer is D. There are three approaches to healthcare: the private model, the Beveridge model (public funds), and the Bismarck model (mix of public and private funds). Answers A, B and C are incorrect as they do not relate to healthcare approaches.

The correct answer is: The Private Model, Beveridge Model, and Bismarck Model.

How well did you know this?
1
Not at all
2
3
4
5
Perfectly
23
Q

Ridley (1984) addressed the distrust that often exists between African American clients and their therapists and proposed that the therapist’s ethnicity is the MOST important factor for:
Select one:

A. a confluent paranoiac.
B. a cultural paranoiac.
C. a nonparanoiac.
D. a dissonant paranoiac.

A

Ridley’s model distinguishes between:

nonparanoics
functional paranoics
healthy cultural paranoics and
confluent paranoics.

a. CORRECT A confluent paranoiac exhibits high cultural and functional paranoia, and his or her nondisclosure in therapy is due to a combination of pathology and the effects of racism. According to Ridley, these clients do better with a therapist of the same racial/ethnic group.
b. Incorrect A cultural paranoiac does not possess views as extreme as a confluent paranoiac and may benefit from a White or African American therapist.
c. Incorrect See explanation for response a.
d. Incorrect See explanation for response a.

The correct answer is: a confluent paranoiac.

How well did you know this?
1
Not at all
2
3
4
5
Perfectly
24
Q

For Minuchin, scapegoating and overprotection of a child by the child’s mother and father are both forms of:
Select one:

A. triangulation.
B. detouring.
C. a stable coalition.
D. marital skew.

A

Minuchin describes three types of “rigid triangles” that all involve boundary disturbances.

a. Incorrect Minuchin uses the term triangulation to refer to the situation in which the child’s loyalty to one parent means rejection of the other parent.
b. CORRECT As defined by Minuchin, detouring occurs when the tension between husband and wife is reduced through the attention they pay to the child. This attention can involve either blaming the child for the family’s problems (scapegoating) or overprotection of the child who is identified by them as ill or weak.
c. Incorrect A stable coalition occurs when the child and a parent consistently “gang up” against the other parent.
d. Incorrect Marital skew is a term used by Lidz to describe situations in which a dominant partner has serious pathology, while the other partner is dependent and provides support.

The correct answer is: detouring.

How well did you know this?
1
Not at all
2
3
4
5
Perfectly
25
Q

From the Freudian perspective, interpretation:
Select one:

A. is appropriate only during the final “working through” phase of therapy.
B. is most useful when it evokes material at the deepest level of the unconscious.
C. helps the client uncover and gain insight into unconscious material.
D. involves restating and clarifying the client’s statements in clearer terms.

A

Interpretation is a key process in Freudian psychoanalysis and is applied to free associations, dreams, resistances, etc.

a. Incorrect Interpretations are made throughout therapy.
b. Incorrect The opposite is true. Interpretation is most successful when it addresses unconscious material that is just below the surface.
c. CORRECT This is the most accurate description of interpretation.
d. Incorrect This is a better description of clarification than intepretation.

The correct answer is: helps the client uncover and gain insight into unconscious material.

How well did you know this?
1
Not at all
2
3
4
5
Perfectly
26
Q

Smith, Glass, and Miller (1980) used meta-analysis to combine the results of 475 psychotherapy outcome studies published between 1941 and 1976 and obtained a mean effect size of ___, which indicated that the average therapy client was “better off” than about 80% percent of individuals who needed therapy but did not receive it.
Select one:

A. 0.4
B. 0.55
C. 0.85
D. 0.98

A

For the licensing exam, you want to be familiar with the Smith, Glass, and Miller (1980) study as well as with the interpretation of an effect size, which indicates the difference between average individuals in the treatment and control groups on the outcome measure in terms of standard deviation units.

a. Incorrect See explanation for response c.
b. Incorrect See explanation for response c.
c. CORRECT If you didn’t remember the effect size reported by Smith et al., being familiar with the areas under the normal curve may have helped you identify the correct answer to this question: In a normal distribution, 84% of cases fall below the score that is one standard deviation above the mean. Therefore 80% of cases will fall below the score that is slightly below one standard deviation above the mean.
d. Incorrect See explanation for response c.

The correct answer is: 0.85

How well did you know this?
1
Not at all
2
3
4
5
Perfectly
27
Q

The National Institute of Mental Health Multimodal Treatment Study of ADHD (MTA) compared the effectiveness of four treatments - medication management alone, behavioral treatment alone, combined medication and behavioral treatment, and routine community care. The results of the initial study indicated that:
Select one:

A. behavioral treatment alone is as effective as medication alone.
B. medication alone or behavioral treatment alone is as effective as the combined treatment.
C. medication alone or the combined treatment is more effective than behavioral treatment alone or routine community care.
D. behavioral treatment alone or the combined treatment is more effective than medication alone or routine community care.

A

Overall, the initial MTA results indicated that the combination treatment or medication management alone were significantly more effective than intensive behavioral treatment alone or routine community care.

a. Incorrect See explanation for response c.
b. Incorrect See explanation for response c.
c. CORRECT Of the answers given, this is the best one. The initial results (which is what this question is asking about) found that medication management alone and the combined treatment had similar effects (and better effects than behavioral treatment alone or routine community care) for alleviating the core symptoms of ADHD. However, follow-up studies found that the superiority of medication alone and the combined treatment was true for short-term effects but not long-term effects. See, e.g., P. S. Jensen et al., Findings from the NIMH Multimodal Treatment Study of ADHD (MTA): Implications and applications for primary care providers, Journal of Developmental & Behavioral Pediatrics, 2001, 22(1), 60-73.

d. Incorrect See explanation for response c.
The correct answer is: medication alone or the combined treatment is more effective than behavioral treatment alone or routine community care.

How well did you know this?
1
Not at all
2
3
4
5
Perfectly
28
Q

Stead et al.’s (2012) review of the literature found that ________ is the most effective form of nicotine replacement therapy for treating cigarette smoking.
Select one:

A. nicotine gum
B. nicotine nasal spray
C. nicotine tablet/lozenge
D. nicotine patch

A

L. F. Stead et al. evaluated the effectiveness nicotine gum, the nicotine patch, nicotine tablets/lozenges, nicotine inhaler, and nicotine nasal spray and found that all forms of nicotine replacement therapy were more effective than a placebo for reducing cigarette smoking

a. Incorrect See explanation for answer b.
b. CORRECT These investigators found that, of the nicotine replacement therapies, nicotine nasal spray was most effective and nicotine gum was least effective (Nicotine replacement therapy for smoking cessation, Cochrane Database Systematic Reviews, CD000146, 2012).
c. Incorrect See explanation for answer b.
d. Incorrect See explanation for answer b.

The correct answer is: nicotine nasal spray

How well did you know this?
1
Not at all
2
3
4
5
Perfectly
29
Q

The parents of 4-year-old Walter report that, even as a baby, he didn’t like to be held, which they attributed to colic. They report that he still doesn’t like to be touched and that he doesn’t like to play with other children and “seems to be in his own world” much of the time. When not engaged in an activity (he only likes playing with cars and dominos), Walter rocks constantly and has a hard time sitting still, but he can focus for hours when he’s lining up his toy cars or dominos. Walter’s cognitive development and language acquisition have been normal, but he does seem to have some trouble reading social cues. Which DSM-5 diagnosis should be considered first for Walter?
Select one:

A. Autism Spectrum Disorder
B. Asperger’s Disorder
C. Stereotypic Movement Disorder
D. Pervasive Developmental Disorder NOS

A

Answer A is correct: Autism Spectrum Disorder involves persistent deficits in social communication and interaction and restricted, repetitive patterns of behavior, interests, and activities.

Answer B: Asperger’s Disorder is not a diagnosis in the DSM-5.

Answer C: Although Walter engages in stereotypic movements (rocking), he exhibits other symptoms that are consistent with Autism Spectrum Disorder (limited interests, difficulties with social reciprocity).

Answer D: Pervasive Developmental Disorder NOS is not a diagnosis in the DSM-5.

The correct answer is: Autism Spectrum Disorder

How well did you know this?
1
Not at all
2
3
4
5
Perfectly
30
Q

For people with Specific Phobia, blood-injection-injury type:
Select one:

A. relaxation techniques are the treatment-of-choice.
B. relaxation techniques have good short-term but poor long-term effects.
C. relaxation techniques are most effective when they include in vivo exposure to feared stimuli.
D. relaxation techniques are normally contraindicated.

A

Specific Phobia, blood-injection-injury type is associated with a unique physiological reaction to feared stimuli and, therefore, requires a treatment approach that differs from those used with other forms of the disorder.

a. Incorrect See explanation for response d.
b. Incorrect See explanation for response d.
c. Incorrect See explanation for response d.
d. CORRECT For people with the blood-injection-injury type, feared stimuli produce an initial increase in heart rate and blood pressure, which is immediately followed by a drop in both and fainting. In contrast, people with other types of Specific Phobia experience only an increase in heart rate and blood pressure. Because of the physiological response associated with the blood-injection-injury type, treatment involves tensing (rather than relaxing) muscles in the presence of feared stimuli.

The correct answer is: relaxation techniques are normally contraindicated.

How well did you know this?
1
Not at all
2
3
4
5
Perfectly
31
Q

Because research has found low to moderate inter-rater reliability for the diagnosis of __________, there is some controversy among experts regarding its validity.
Select one:

A. Specific Phobia
B. Generalized Anxiety Disorder
C. Panic Disorder
D. Obsessive Compulsive Disorder

A

Inter-rater reliability has been found to be a problem for a number of diagnoses, including some of the Anxiety Disorders.

a. Incorrect See explanation for response b.
b. CORRECT Reported inter-rater reliability coefficients for Generalized Anxiety Disorder (GAD) vary, but tend to be lower than those reported for other Anxiety Disorders. The primary reasons for the low inter-rater reliability for GAD are disagreements regarding the severity of symptoms (i.e., what constitutes “excessive anxiety and worry”) and whether it should be the primary or secondary diagnosis. See, e.g., L. A. Papp and M. S. Kleber, Phenomenology of Generalized Anxiety Disorder, in D. J. Stein and E. Hollander (Eds.), Textbook of anxiety disorders (pp. 109-118), Washington, D.C., American Psychiatric Press, 2002. Note that low inter-rater reliability has also been reported for PTSD.
c. Incorrect See explanation for response b.
d. Incorrect See explanation for response b.

The correct answer is: Generalized Anxiety Disorder

How well did you know this?
1
Not at all
2
3
4
5
Perfectly
32
Q

A client with Borderline Personality Disorder is most likely to respond to her therapist in which of the following ways?
Select one:

A. with hostility and suspiciousness about the therapist’s intentions
B. in an inappropriately sexually provocative manner
C. in a manner that fluctuates between argumentativeness/hostility and excessive admiration
D. in a consistently demanding manner and with hypersensitivity to criticism

A

A key distinguishing characteristic of the various Personality Disorders is the nature of the individual’s interpersonal relationships.

a. Incorrect This sounds more like Paranoid Personality Disorder.
b. Incorrect This is characteristic of Histrionic Personality Disorder.
c. CORRECT Instability in relationships is a primary feature of Borderline Personality Disorder. People with this disorder may be overly complimentary and flattering toward a person one day and extremely hostile and critical the next.
d. Incorrect This sounds more likely Narcissistic Personality Disorder.

The correct answer is: in a manner that fluctuates between argumentativeness/hostility and excessive admiration

How well did you know this?
1
Not at all
2
3
4
5
Perfectly
33
Q

When shown the word “hot” a patient says “cold” and when shown the word “rock” the patient says “stone.” These errors are most suggestive of:
Select one:

A. surface dyslexia.
B. deep dyslexia.
C. pure alexia.
D. literal alexia.

A

Alexia and dyslexia are terms used to describe reading disorders.

a. Incorrect Surface dyslexia is characterized by reading without comprehension due to an inability to read words that are irregularly spelled (e.g., light, sword).
b. CORRECT Deep dyslexia involves several reading errors including semantic paralexia, which is characterized by the substitution of words with similar meanings (e.g., cold for hot).
c. Incorrect Pure alexia is also known as “word blindness” and refers to an inability to read words, even when they have been written by the person him/herself.
d. Incorrect People with literal alexia can read whole words but not individual letters.

The correct answer is: deep dyslexia.

How well did you know this?
1
Not at all
2
3
4
5
Perfectly
34
Q

A psychologist develops a theory of alcohol abuse that describes it as the result of a combination of a genetic predisposition toward alcoholism and exposure to certain environmental factors. The psychologist’s theory is best described as an example of the ____________ model.
Select one:

A. psychogenic
B. sociogenic
C. diathesis-stress
D. SORC

A

The psychologist’s theory views alcohol abuse as the result of a combination of biological and environmental factors.

a. Incorrect As its name implies, the psychogenic model focuses on the psychological contributions to a disorder.
b. Incorrect The sociogenic model focuses on societal factors that contribute to a disorder, such as societal norms, stereotypes, and institutional biases.
c. CORRECT The diathesis-stress model reflects a biopsychosocial perspective and conceptualizes disorders as the result of a combination of biological, psychological, and environmental (social) factors. More specifically, it proposes that disorders are due to a combination of predispositional factors (the diathesis) and exposure to certain life stressors.
d. Incorrect The SORC model is used by cognitive-behavioral therapists as the framework for assessment. Use of this model involves considering the situation in which the problem occurs (S); the observations (thoughts, assumptions, and beliefs) made by the client in response to the situation (O); the client’s emotional, psychological, and behavioral responses to his/her observations (R); and the consequences of those responses (C).

The correct answer is: diathesis-stress

How well did you know this?
1
Not at all
2
3
4
5
Perfectly
35
Q

As defined in the DSM-5, a diagnosis of ADHD, predominantly inattentive presentation is assigned when a child exhibits:
Select one:

A. at least six symptoms of inattention and no symptoms of hyperactivity-impulsivity.
B. at least six symptoms of inattention and fewer than six symptoms of hyperactivity-impulsivity.
C. at least eight symptoms of inattention and fewer than four symptoms of hyperactivity-impulsivity.
D. at least eight symptoms of inattention and fewer than eight symptoms of hyperactivity-impulsivity.

A

Answer B is correct: The DSM-5 distinguishes between three subtypes of ADHD: predominantly inattentive presentation, predominantly hyperactive-impulsive presentation, and combined presentation. The number of symptoms listed in this answer (at least six symptoms of inattention and fewer than six symptoms of hyperactivity-impulsivity) is the requirement for the predominantly inattentive type. For the predominantly hyperactive-impulsive presentation, the requirement is at least six symptoms of hyperactivity-impulsivity and less than six symptoms of inattention; and for the combined presentation, the requirement is at least six symptoms of inattention and at least six symptoms of hyperactivity-impulsivity. (Note that, for individuals over age 17, the minimum number of symptoms for the diagnosis of each subtype is five rather than six.)

Answers A, C, and D: See explanation for answer B.
The correct answer is: at least six symptoms of inattention and fewer than six symptoms of hyperactivity-impulsivity.

How well did you know this?
1
Not at all
2
3
4
5
Perfectly
36
Q

Studies examining the risk factors for recidivism among juvenile offenders has identified which of the following as being the BEST predictor of re-offending?
Select one:

A. IQ
B. socioeconomic status
C. age at first offense
D. history of maltreatment/abuse

A

Risk and protective factors for recidivism among juvenile offenders was investigated by C. Schwalbe et al. [North Carolina Assessment of Risk (NCAR): Reliability and predictive validity with juvenile offenders, Journal of Offender Rehabilitation, 40(1/2), 1-22, 2004.]

a. Incorrect Schwalbe et al. report a correlation of .142 between IQ and re-offense.
b. Incorrect These investigators report a correlation of .065 for SES and re-offense.
c. CORRECT Schwalbe et al. found the strongest correlation (r = -.346) between age at first offense and reoffense (the younger the age the first offense, the higher the risk for recidivism).
d. Incorrect These authors report a correlation of .112 between history of maltreatment and re-offense.

The correct answer is: age at first offense

How well did you know this?
1
Not at all
2
3
4
5
Perfectly
37
Q

The DSM-5 requires that which of the following be ruled out as the cause of an individual’s symptoms before assigning a diagnosis of a sexual dysfunction?
Select one:

A. other mental disorders, physiological/medical factors, and partner factors
B. a nonsexual mental disorder, severe relationship distress, and the effects of a substance, medication, or other medical condition
C. a nonsexual mental disorder, the effects of a substance/medication, and a lack of information about sexuality
D. other mental disorder/condition, the effects of a substance/medication, and psychosocial factors

A

Answer B is correct: The following is included as a diagnostic criterion for a DSM-5 diagnosis of Erectile Disorder, Premature Ejaculation, and other sexual dysfunctions: “The sexual dysfunction is not better explained by a nonsexual mental disorder or as a consequence of severe relationship distress or other significant stressors and is not attributable to the effects of a substance/medication or another medical condition.”

Answers A, C, and D: See explanation above.

The correct answer is: a nonsexual mental disorder, severe relationship distress, and the effects of a substance, medication, or other medical condition

How well did you know this?
1
Not at all
2
3
4
5
Perfectly
38
Q

With regard to substance use, “tolerance” is characterized by:
Select one:

A. the need to increase the amount of the drug to achieve desired effects or the development of withdrawal symptoms when the amount of the drug is not increased.
B. the need to increase the amount of the drug to achieve desired effects accompanied by a persistent craving for the drug.
C. the need to increase the amount of the drug to achieve desired effects or a markedly diminished effect of the drug with continued use of the same amount.
D. the need to increase the amount of the drug to avoid withdrawal symptoms accompanied by a persistent craving for the drug.

A

Answer C is correct: Tolerance is one of the symptoms of Substance Use Disorder, and this answer accurately describes tolerance.

The correct answer is: the need to increase the amount of the drug to achieve desired effects or a markedly diminished effect of the drug with continued use of the same amount.

How well did you know this?
1
Not at all
2
3
4
5
Perfectly
39
Q

Following her divorce nine months ago, Melody, age 29, developed intermittent abdominal pain and frequent morning headaches. She’s concerned that these symptoms mean that she has cancer or other life threatening illness, and constant worry about her health has made it difficult for her to care for her children, spend time with her friends, and concentrate at work. Assuming that a physical exam has ruled out a physical cause for her symptoms, the most likely DSM-5 diagnosis for Melody is:
Select one:

A. Somatoform Disorder NOS
B. Conversion Disorder with mixed symptoms
C. Pain Disorder
D. Somatic Symptom Disorder with predominant pain

A

Answer D is correct: The most likely diagnosis for Melody is Somatic Symptom Disorder, which is characterized by the presence of one or more somatic symptoms accompanied by excessive thoughts, feelings, or behaviors related to the symptoms. Because her symptoms involve only pain, the specifier “with predominant pain” is appropriate.

Answer A: Somatoform Disorder NOS is not a DSM-5 diagnosis.

Answer B: A diagnosis of Conversion Disorder requires the presence of symptoms that involve a disturbance in voluntary motor or sensory functioning and suggest a serious neurological or other medical condition with evidence that the symptoms are incompatible with recognized neurological and medical conditions.

Answer C: Pain Disorder is not a DSM-5 diagnosis.

The correct answer is: Somatic Symptom Disorder with predominant pain

How well did you know this?
1
Not at all
2
3
4
5
Perfectly
40
Q

Research on the effectiveness of the SSRIs has found that:
Select one:

A. a higher dose of an SSRI is usually required when treating OCD than when treating depression.
B. a lower dose of an SSRI is usually required when treating OCD than when treating depression.
C. a moderate dose of an SSRI is usually required when treating OCD but the correct dose for depression depends on the severity of symptoms.
D. a moderate dose of an SSRI is usually required when treating depression but the correct dose for OCD depends on the severity of symptoms.

A

The SSRIs have been identified as effective treatments for several disorders including Obsessive Compulsive Disorder (OCD) and Major Depressive Disorder.

a. CORRECT The research on SSRI dosage is limited and not entirely consistent. However, based on their meta-analysis of the studies, H. Bloch and colleagues concluded that a higher dose is required when treating OCD than when treating depression [Meta-analysis of the dose response relationship of SSRI in obsessive-compulsive disorder, Molecular Psychiatry, 15(8), 850-855, 2010].
b. Incorrect See explanation above.
c. Incorrect See explanation above.
d. Incorrect See explanation above.

The correct answer is: a higher dose of an SSRI is usually required when treating OCD than when treating depression.

How well did you know this?
1
Not at all
2
3
4
5
Perfectly
41
Q

Which of the following is most true about Conduct Disorder?
Select one:

A. Individuals with childhood-onset type are more likely than those with adolescent-onset type to develop adult Antisocial Personality Disorder.
B. Individuals with adolescent-onset type are more likely than those with childhood-onset type to develop adult Antisocial Personality Disorder.
C. The majority of individuals with Conduct Disorder receive a diagnosis of Antisocial Personality Disorder in adulthood, regardless of age of onset.
D. Conduct Disorder is normally associated with Antisocial Personality Disorder only when there is a family history of antisocial behavior.

A

According to the DSM, the long-term outcomes for Conduct Disorder vary but, for the majority of individuals, it remits by adulthood.

a. CORRECT This is the best answer since there is evidence that an early onset of symptoms places the child at greatest risk for Antisocial Personality Disorder in adulthood.
b. Incorrect Adolescent onset is less associated with antisocial behavior in adulthood.
c. Incorrect See explanation above.
d. Incorrect See explanation above.

The correct answer is: Individuals with childhood-onset type are more likely than those with adolescent-onset type to develop adult Antisocial Personality Disorder.

How well did you know this?
1
Not at all
2
3
4
5
Perfectly
42
Q

Studies investigating the genetic contribution to major depression have found that:
Select one:

A. the risk for depression is significantly higher for biological offspring who have one parent with depression than for those who have two parents with the disorder.
B. the risk for depression is significantly higher for biological offspring who have two parents with depression than for those who have only one parent with the disorder.
C. the risk for depression is about the same for biological offspring who have either one parent or two parents with depression.
D. the risk for depression is significantly higher for biological offspring who have a mother with depression than for those who have a father with depression.

A

Although the research has confirmed a genetic contribution to depression, results related to the link between depression for parents and their biological offspring are contrary to what might be expected.

a. Incorrect See explanation for response c.
b. Incorrect See explanation for response c.
c. CORRECT The studies have confirmed that having a biological parent with depression increases an offspring’s risk for major depression as well as a number of other disorders. However, the risk for depression is similar regardless of whether the child has one parent or two parents with major depression. See, e.g., Lieb, R. et al., Parental major depression and the risk of depression and other mental disorders in offspring: A prospective-longitudinal community study, Archives of General Psychiatry, 59, 365-374, 2002.
d. Incorrect The research has generally found that, in terms of rates of major depression, maternal and paternal depression affects offspring similarly. In the Lieb et al. study, the rate for depression was somewhat higher for the biological children of fathers with depression but the rates for children of fathers or mothers with the disorder were not significantly different.

The correct answer is: the risk for depression is about the same for biological offspring who have either one parent or two parents with depression.

How well did you know this?
1
Not at all
2
3
4
5
Perfectly
43
Q

In Alzheimer’s disease, neuron loss occurs throughout the brain. However, the greatest loss is in which of the following areas?
Select one:

A. medial temporal lobe
B. anterior parietal lobe
C. medial occipital lobe
D. posterior frontal lobe

A

The distinctive characteristics of Alzheimer’s disease are neuron loss, neurofibrillary tangles (tangles of protein), and amyloid plaques (clumps of scar tissue).

a. CORRECT The medial temporal lobe includes the entorhinal cortex, amygdala, and hippocampus, which are involved in memory and are most affected by Alzheimer’s disease.
b. Incorrect It is the posterior (not anterior) parietal lobe that is affected by Alzheimer’s.
c. Incorrect The occipital lobe is not one of the brain areas substantially affected by this disorder.
d. Incorrect The prefrontal area of the frontal cortex is affected more than the posterior area.

The correct answer is: medial temporal lobe

How well did you know this?
1
Not at all
2
3
4
5
Perfectly
44
Q

As described in the DSM-5, level of severity of Alcohol Use Disorder and other Substance Use Disorders is based on:
Select one:

A. degree of functional impairment.
B. number of symptoms.
C. the presence of tolerance and/or withdrawal.
D. the degree of distress caused by use of the substance.

A

Answer B is correct: The DSM-5 distinguishes between three levels of severity of the Substance Use Disorders based on the number of symptoms: mild (2 or 3 symptoms); moderate (4 or 5 symptoms); severe (6 or more symptoms).

Answers A, C, and D: See explanation for answer B.

The correct answer is: number of symptoms.

How well did you know this?
1
Not at all
2
3
4
5
Perfectly
45
Q

Individuals who receive a diagnosis of which of the following Personality Disorders are most likely to no longer meet all of the diagnostic criteria for the disorder when they reach middle age?
Select one:

A. Schizotypal Personality Disorder
B. Antisocial Personality Disorder
C. Paranoid Personality Disorder
D. Avoidant Personality Disorder

A

Although Antisocial Personality Disorder (APD) is considered to be chronic, symptoms often decrease or remit with increasing age.

a. Incorrect See explanation for response b.
b. CORRECT The reduction in symptoms in APD by middle age most often involves a decrease in criminal behaviors while other characteristic symptoms - especially problems related to interpersonal relationships - persist to some degree. See, e.g., J. Paris, Personality disorders over time: Implications for therapy, American Journal of Psychotherapy, 58(4), 420-429, 2004.
c. Incorrect See explanation for response b.

d. Incorrect See explanation for response b.
The correct answer is: Antisocial Personality Disorder

How well did you know this?
1
Not at all
2
3
4
5
Perfectly
46
Q

Presence of which of the following symptoms would help confirm a DSM diagnosis of Inhalant Intoxication?
Select one:

A. increased appetite, dry mouth, and tachycardia
B. unsteady gait, slurred speech, and tremor
C. nausea, vomiting, and muscle aches
D. fatigue, vivid and unpleasant dreams, and increased appetite

A

Answer B is correct: Symptoms of Inhalant Intoxication include dizziness, nystagmus, incoordination, slurred speech, unsteady gait, lethargy, depressed reflexes, psychomotor retardation, tremor, generalized muscle weakness, blurred vision, stupor or coma, and euphoria.

Answer A: Increased appetite, dry mouth, and tachycardia are symptoms of Cannabis Intoxication.

Answer C: Nausea, vomiting, and muscle aches are symptoms of Opioid Withdrawal.

Answer D: Fatigue, vivid and unpleasant dreams, and increased appetite are symptoms of Stimulant Withdrawal.

The correct answer is: unsteady gait, slurred speech, and tremor

How well did you know this?
1
Not at all
2
3
4
5
Perfectly
47
Q

It is often difficult to distinguish between Major and Mild Neurocognitive Disorder, Delirium, and Major Depressive Disorder in older adults. However, the presence of which of the following suggests that Delirium is the appropriate DSM-5 diagnosis?
Select one:

A. deficits in memory
B. fatigue or loss of energy
C. impaired attention and awareness
D. flight of ideas

A

Answer C is correct: As described in the DSM-5, the essential feature of Delirium is a disturbance in attention and awareness plus an additional disturbance in cognition (e.g., memory deficit, disorientation, language impairment, deficit in visuospatial ability, or perceptual distortion).

Answer A: Memory impairment is characteristic of all three disorders and, therefore, is not useful for distinguishing between them.

Answer B: Fatigue or a loss of energy is characteristic of Major Depressive Disorder.

Answer D: Flight of ideas is characteristic of manic and hypomanic episodes.

The correct answer is: impaired attention and awareness

How well did you know this?
1
Not at all
2
3
4
5
Perfectly
48
Q

Annabelle, age 32, is brought to a clinician by her sister who says she thinks Annabelle is in a cult. She states that Annabelle used to be a very fun-loving, intellectually curious, and affectionate individual, but since she became affiliated with this group over two years ago, she “doesn’t seem to be the same person.” The clinician notices that Annabelle is very quiet, doesn’t make eye contact, and appears “zoned out.” When her sister reaches out to touch her, Annabelle flinches as if in pain. Annabelle states that she’s a “new person” now and can “see reality for what it really is.” Based on this information, the most likely DSM-5 diagnosis for Annabelle is:
Select one:

A. Other Specified Dissociative Disorder
B. Posttraumatic Stress Disorder
C. Schizoid Personality Disorder
D. Major Depressive Disorder

A

Answer A is correct: Of the diagnoses listed, Other Specified Dissociative Disorder is the best fit. This diagnosis is assigned when the individual has some dissociative symptoms (in this case, appearing zoned out and the radical change in her personality as reported by her sister) that do not meet the full criteria for one of the other dissociative disorders. Because Annabelle’s symptoms appear to be related to her association with a cult, the full diagnosis would be Other Specified Dissociative Disorder with identity disturbance due to prolonged and intense coercive persuasion.

Answer B: While PTSD should be considered as a possibility (the client may have been subjected to abuse in the group), no information is provided in the question suggesting that she has the classic symptoms of PTSD such as intrusive thoughts, avoidance, and alterations in arousal and reactivity.

Answer C: The diagnosis of a Personality Disorder requires a persistent pattern of maladaptive behavior with an onset in adolescence or early adulthood. From her sister’s description, Annabelle was previously fun-loving, intellectually curious, and affectionate, and the personality change occurred after she joined the cult.

Answer D: Major Depressive Disorder is a condition to consider in this case given the client’s “shut down” presentation. However, the information provided in the question is not sufficient for this to be the most likely diagnosis.

The correct answer is: Other Specified Dissociative Disorder

How well did you know this?
1
Not at all
2
3
4
5
Perfectly
49
Q

For a person with Social Anxiety Disorder, panic attacks are:
Select one:

A. the result of interoceptive conditioning.
B. cued by specific social situations.
C. infrequent and nearly always unexpected.
D. indicative of a co-diagnosis of Panic Disorder.

A

Social Anxiety Disorder is also known as Social Phobia and is characterized by anxiety associated with one or more specific social situations.

a. Incorrect Interoceptive conditioning refers to classical conditioning that involves internal cues and is not a distinguishing characteristic of the panic attacks associated with Social Anxiety Disorder.
b. CORRECT A person with Social Anxiety Disorder may experience panic attacks but these attacks are related to known feared situations and are expected rather than unexpected. When the individual experiences recurrent unexpected panic attacks, the appropriate diagnosis is likely to be Panic Disorder or co-morbid Panic Disorder and Social Anxiety Disorder.
c. Incorrect See explanation for response b.

d. Incorrect See explanation for response b.
The correct answer is: cued by specific social situations.

How well did you know this?
1
Not at all
2
3
4
5
Perfectly
50
Q

Eliana, age 32, tells you that she often has trouble concentrating on her work and falling asleep at night because she’s constantly worrying about having cancer. She states that her older sister had surgery for breast cancer last year and her mother’s sister and uncle both had lung cancer. Eliana says she gets a physical exam every six months and has been reassured by the doctor that she’s okay; but she knows that, for some people, there are no symptoms of cancer until it’s too late to do anything about it. Eliana also tells you that she keeps checking her breasts for lumps and gets scared whenever she coughs because she thinks it might be an early sign of lung cancer. Eliana’s symptoms are most suggestive of which of the following DSM-5 disorders?
Select one:

A. Body Dysmorphic Disorder
B. Somatic Symptom Disorder
C. Hypochondriasis
D. Illness Anxiety Disorder

A

Answer D is correct: Illness Anxiety Disorder is characterized by a preoccupation with having a serious illness, an absence of somatic symptoms or the presence of mild somatic symptoms, a high level of anxiety about one’s health, and performance of excessive health-related behaviors (care-seeking type) or maladaptive avoidance of doctors, hospitals, etc. (care-avoidant type).

Answer A: Body Dysmorphic Disorder is characterized by a preoccupation with a defect or flaw in appearance that appears minor or is unobservable to others.

Answer B: Somatic Symptom Disorder involves the presence of somatic symptoms, while Illness Anxiety Disorder is characterized by the absence of somatic symptoms or the presence of only minimal symptoms. Eliana is worried about her health but does not actually have physical symptoms.

Answer C: Hypochondriasis is not a DSM-5 diagnosis.

The correct answer is: Illness Anxiety Disorder

How well did you know this?
1
Not at all
2
3
4
5
Perfectly
51
Q

A person with Dissociative Amnesia is unable to recall anything that happened to her during the first few hours after she was physically assaulted. As defined in the DSM, this is referred to as:
Select one:

A. localized amnesia.
B. selective amnesia.
C. anterograde amnesia.
D. retrograde amnesia.

A

Dissociative Amnesia can take one of several form - e.g., localized, selective, and generalized.

a. CORRECT The loss of memory for personal information limited to a circumscribed period of time is referred to as localized amnesia.
b. Incorrect Selective amnesia refers to a loss of memory for some, but not all, events during a circumscribed period.
c. Incorrect Anterograde amnesia is a general term that refers to the inability to form new memories and does describe the woman’s loss of memory. However, because it does not refer specifically to memory loss associated with Dissociative Amnesia, it is not the best answer to this question.
d. Incorrect Retrograde amnesia refers to a loss of memory for information already stored in long-term memory.

The correct answer is: localized amnesia.

How well did you know this?
1
Not at all
2
3
4
5
Perfectly
52
Q

Which of the following is characterized by intense sexually arousing fantasies, sexual urges, or behaviors that involve touching or rubbing against a nonconsenting person.
Select one:

A. fetishism
B. frotteurism
C. exhibitionism
D. trichotillomania

A

Paraphilias (Paraphilic Disorders in DSM-5) involve intense sexually arousing fantasies, sexual urges, or behaviors related to unusual objects, activities, or situations.

a. Incorrect Fetishism is characterized by intense sexually arousing fantasies, sexual urges, or behaviors involving the use of nonliving objects (e.g., shoes, gloves).
b. CORRECT The information in the question accurately describes frotteurism.
c. Incorrect Exhibitionism involves intense sexual arousal from exposure of one’s genitals to an unexpecting person.
d. Incorrect Trichotillomania is an impulse control disorder that involves recurrent pulling of one’s own hair, causing noticeable hair loss.

The correct answer is: frotteurism

How well did you know this?
1
Not at all
2
3
4
5
Perfectly
53
Q

Project MATCH compared motivational enhancement therapy (MET), cognitive-behavioral coping skills therapy (CBT), and twelve-step facilitation (TSF) as treatments for alcoholism and found that:
Select one:

A. MET, CBT, and TSF had similar effects on drinking.
B. the effects of CBT on drinking were significantly greater than the effects of MET or TSF.
C. the effects of MET on drinking were significantly greater than the effects of CBT or TSF.
D. CBT and MET had significant effects on drinking only when combined with TSF.

A

Project MATCH was a five-year longitudinal study conducted by the National Institute on Alcohol Abuse and Alcoholism in the 1990s. It compared the effectiveness of MET, CBT, and TSF on two measures of alcohol consumption (percentage of days abstinent and average number of drinks per day) and tested the hypothesis that treatments are most effective when they are matched to certain patient characteristics.

a. CORRECT The study found that the three treatments had nearly identical positive effects on both outcome measures but that few patient-treatment matches resulted in an improvement in outcomes. Also, a subsequent analysis of the data by R. B. Cutler and D. A. Fishbain suggested that the apparent treatment effects were actually the result of selection effects - i.e., were due to the fact that individuals who were more likely to continue drinking dropped out of the study, while individuals who were more likely to stop drinking stayed in the study (Are alcoholism treatments effective? The Project MATCH data, BMC Public Health, 5, 75, 2005).
b. Incorrect See explanation for answer a.
c. Incorrect See explanation for answer a.
d. Incorrect See explanation for answer a.

The correct answer is: MET, CBT, and TSF had similar effects on drinking.

How well did you know this?
1
Not at all
2
3
4
5
Perfectly
54
Q

A physician is most likely to prescribe which of the following for a 36-year old man who has just received a diagnosis of Premature Ejaculation?
Select one:

A. a drug that stimulates dopamine receptors
B. a drug that inhibits GABA receptors
C. a drug that inhibits the reuptake of serotonin
D. a drug that stimulates the reuptake of endorphins

A

Premature ejaculation has been linked to low serotonin levels.

a. Incorrect Increased dopamine activity can cause or exacerbate premature ejaculation.
b. Incorrect GABA helps reduce anxiety. Therefore, it would likely be more effective to stimulate (rather than inhibit) GABA receptors.
c. CORRECT SSRIs (e.g., dapoxetine) are currently used to treat premature ejaculation.
d. Incorrect Endorphins have not been linked to premature ejaculation.

The correct answer is: a drug that inhibits the reuptake of serotonin

How well did you know this?
1
Not at all
2
3
4
5
Perfectly
55
Q

Research investigating the effectiveness of neurofeedback as a treatment for ADHD has generally found which of the following?
Select one:

A. It is not an effective approach for most individuals with this disorder.
B. It is an effective approach and exerts its strongest impact on symptoms of inattention and impulsivity.
C. It is an effective approach and exerts its strongest impact on symptoms of hyperactivity.
D. It is an effective approach only when used in conjunction with a central nervous system stimulant.

A

The results of recent research on the effects of neurofeedback (EEG biofeedback) as a nonpharmacological treatment for ADHD are summarized in a meta-analysis conducted by M. Arns, S. de Ridder, U. Strehl, M. Breteler, and A. Coenen [Efficacy of neurofeedback treatment in ADHD: The effects on inattention, impulsivity and hyperactivity: A meta-analysis, Clinical EEG and Neuroscience, 49(3), 180-189, 2009].

a. Incorrect See explanation for response b.
b. CORRECT Arns et al.’s meta-analysis produced a large effect size for inattention and impulsivity and a medium effect size for hyperactivity. Their results also indicated that the positive effects of neurofeedback remained stable or improved over a two-year period.
c. Incorrect See explanation for response b.

d. Incorrect The studies suggest that neurofeedback has benefits for about 70 to 80% of individuals with ADHD and that its effects are similar to those for stimulant medication. In other words, neurofeedback does not have to be combined with a stimulant drug but is effective when used alone.
The correct answer is: It is an effective approach and exerts its strongest impact on symptoms of inattention and impulsivity.

How well did you know this?
1
Not at all
2
3
4
5
Perfectly
56
Q

Functional brain imaging of a patient with mild-to-moderate Alzheimer’s disease is most likely to find which of the following?
Select one:

A. increased metabolism in the frontal lobes and basal ganglia
B. decreased metabolism in the prefrontal cortex and thalamus
C. reduced metabolism in the hippocampus and entorhinal cortex
D. increased metabolism in the hippocampus and amygdala

A

Knowing that atrophy in the hippocampus has been linked to the memory loss associated with Alzheimer’s disease would have helped you identify the correct answer to this question.

a. Incorrect Increased metabolism in the frontal lobes and basal ganglia has been linked to Obsessive-Compulsive Disorder.
b. Incorrect Decreased metabolism in the prefrontal cortex and thalamus has been linked to Schizophrenia.
c. CORRECT Neuronal degeneration in the medial temporal structures (entorhinal cortex, hippocampus, and amygdala) has been linked to Alzheimer’s disease. On a PET scan, this is manifested as reduced metabolism in these structures.
d. Incorrect Increased metabolism in the hippocampus and amygdala has not been linked to Alzheimer’s disease.

The correct answer is: reduced metabolism in the hippocampus and entorhinal cortex

How well did you know this?
1
Not at all
2
3
4
5
Perfectly
57
Q

A school psychologist is contacted by the father of one of the school’s students. He informs the psychologist that the family is moving to another state and requests that she provide him with a copy of his child’s records. The psychologist should:
Select one:

A. provide a copy of the records to the father as requested because he has a legal right to them.
B. provide a copy of the records to the father in a sealed envelope that is marked “confidential” and instruct him to give it to the school psychologist at the new school.
C. request the name and address of the new school and forward the records to the school psychologist once the family has moved.
D. refuse to release a copy of the records because they are the property of the school.

A

Although parents do have a legal right to examine school records, this does not mean they should necessarily be given a copy of them.

a. Incorrect See explanation for response c.
b. Incorrect See explanation for response c.
c. CORRECT The action described in this answer is most consistent with Paragraph 2.3.7 of the APA’s General Guidelines for Providers of Psychological Services, requirements related to confidentiality provided in the Ethics Code, and the provisions of the Family Education Rights and Privacy Act (FERPA).
d. Incorrect See explanation for response c.

The correct answer is: request the name and address of the new school and forward the records to the school psychologist once the family has moved.

How well did you know this?
1
Not at all
2
3
4
5
Perfectly
58
Q

When working in military settings, a clinical psychologist should keep in mind that:
Select one:

A. clients should be informed of potential limits to confidentiality at the outset of the delivery of services.
B. confidential client information should not be revealed to military personnel without a client’s consent except when the client is a danger to self or others.
C. client confidentiality is treated in the same way that it is treated in other settings only when the client is self-referred.
D. legal and ethical requirements related to confidentiality are suspended as required by military rules and policies.

A

Department of Defense rules and policies may require breaches of confidentiality that would, in other situations, be unethical and illegal.

a. CORRECT Normally, client confidentiality should not be breached even in military settings. However, under certain circumstances (defined by the Department of Defense), military personnel may have access to confidential client information. APA has determined that breaches of confidentiality are acceptable in this situation when two conditions have been met: (1) a previous process has already addressed any options that would uphold the psychologist’s ethical responsibilities (i.e., whenever the ethical issue in question has been addressed previously), and (2) the conditions of the Department of Defense’s regulations have clearly been met (in most circumstances, a breach is required only when there is a demonstrable “need to know”). For a brief discussion of this issue, see the American Psychologist, 49(7), p. 665, 1994.
b. Incorrect See explanation for response a.
c. Incorrect See explanation for response a.
d. Incorrect See explanation for response a.

The correct answer is: clients should be informed of potential limits to confidentiality at the outset of the delivery of services.

How well did you know this?
1
Not at all
2
3
4
5
Perfectly
59
Q

Following a review of the case, an HMO decides to deny further payment for sessions for a current client of yours. You believe that the client would benefit from additional therapy. You should:
Select one:

A. comply with the HMO’s request as long as the limits of treatment were discussed with the client at the onset of therapy.
B. continue to provide therapy to the client without compensation if necessary until other arrangements can be made.
C. consult with the client about her options in this situation.
D. file a complaint against the HMO.

A

An issue frequently addressed in the literature on managed care is the problem created when the managed care entity decides to discontinue coverage.

a. Incorrect It would be inappropriate both legally and ethically for a psychologist to cooperate immediately with an HMO’s decision if he/she believes that additional care is needed.
b. Incorrect This may not be necessary in this situation and may not even be required in other situations, especially when there are other options.
c. CORRECT Of the responses given, this is the best one. The psychologist should discuss the client’s options with him/her (one of which might be to file an appeal).
d. Incorrect If the psychologist believed that termination of services is contraindicated, he/she could file an appeal with the HMO (not a complaint).

The correct answer is: consult with the client about her options in this situation.

How well did you know this?
1
Not at all
2
3
4
5
Perfectly
60
Q

Dr. Dilemma is contacted by an insurance company, which wants information about a previous client of hers. The client’s fee was paid by the insurance company when he was seeing her, and the insurance company now wants information for part of its regular peer review process. Dr. Dilemma’s best course of action would be to:
Select one:

A. provide the insurance company with the information it requests.
B. provide the insurance company with the information it requests only after ensuring that the company will take steps to protect the client’s confidentiality.
C. provide the insurance company with the information it requests in a way that allows the client’s identity to be concealed.
D. refuse to release any information to the company unless she feels it is in the client’s best interests.

A

Problems related to confidentiality constituted one of the original objections to the peer review process. Insurance companies and other third-party payers have, however, been fairly responsible in finding ways to maintain the confidentiality of clients.

a. Incorrect See explanation for response b.
b. CORRECT Normal confidentiality requirements are relaxed in cases of peer review because of the necessity to provide review committees with information about clients. However, confidentiality is always an issue when releasing information about a client and, consequently, this would be the best course of action.
c. Incorrect Given the nature of the situation, this would not be possible. Even if it were, it would probably not be acceptable to the insurance company. In cases of peer review, the practice is for the insurance company to safeguard the client’s confidentiality (e.g., by removing the client’s name from the clinical information before passing it on to the committee).
d. Incorrect See explanation for response b. (In this situation, as in others involving the release of information, you would want to release only information you deem to be relevant.)

The correct answer is: provide the insurance company with the information it requests only after ensuring that the company will take steps to protect the client’s confidentiality.

How well did you know this?
1
Not at all
2
3
4
5
Perfectly
61
Q

When a research study requires participants to be deceived about its real purpose prior to their participation, they:
Select one:

A. do not have to be told the true purpose of the study as long as the deception was justified by the prospective value of the study.
B. do not have to be told the true purpose of the study unless not to do so would be likely to cause them psychological harm.
C. must be debriefed about the true purpose of the study immediately after their participation.
D. should be debriefed about the true purpose of the study as soon as possible after their participation.

A

This issue is addressed in Standard 8.07 of the APA’s Ethics Code and Principle III.26 of the Canadian Code of Ethics.

a. Incorrect See explanation for response d.
b. Incorrect See explanation for resposne d.
c. Incorrect See explanation for response d.
d. CORRECT This answer is most consistent with ethical requirements. For example, Standard 8.07(c) of the Ethics Code states that “psychologists explain any deception that is an integral feature of the design and conduct of an experiment to participants as early as is feasible, preferably at the conclusion of their participation but no later than at the conclusion of the data collection.”

The correct answer is: should be debriefed about the true purpose of the study as soon as possible after their participation.

How well did you know this?
1
Not at all
2
3
4
5
Perfectly
62
Q

During an initial session with a client, the client tells Dr. Paphian that he and his wife have not been getting along well lately, and he thinks they may need couples counseling. As the client describes his wife, Dr. Paphian realizes that he had a brief affair with her ten years ago (prior to her marriage). Dr. Paphian should:
Select one:

A. make an appointment with the wife and discuss the matter with her before agreeing to see the couple together in therapy.
B. see the husband in individual therapy only and refer them to another therapist for couples therapy.
C. see the couple in therapy but not bring up the issue of his previous relationship unless the wife does.
D. not see the husband or the couple in therapy but, instead, make a referral to another therapist.

A

This situation is covered by the Ethic Code Standard 10.07, which prohibits psychologists from providing therapy to former sexual partners.

a. Incorrect See explanation for response d.
b. Incorrect See explanation for response d.
c. Incorrect See explanation for response d.
d. CORRECT Seeing the couple in therapy would clearly violate the requirements of Ethical 10.07. Seeing the husband only is not as directly addressed by the Code, but certainly would represent a situation that might impair your objectivity or otherwise interfere with effectively performing your functions as a psychologist (Ethical Standard 3.05). It would also not be consistent with the requirement that psychologists avoid and minimize harm to clients (Ethical Standard 3.04).

The correct answer is: not see the husband or the couple in therapy but, instead, make a referral to another therapist.

How well did you know this?
1
Not at all
2
3
4
5
Perfectly
63
Q

When a psychologist’s work with clients will be supervised:
Select one:

A. the client must be told about this arrangement.
B. the client must be told about this arrangement and be given the supervisor’s name.
C. the client must be told about this arrangement only if his/her identity cannot be disguised in discussions with the supervisor.
D. the client must be told about this arrangement and, when the supervisor has legal responsibility for the case, must also be given the supervisor’s name.

A

This issue is addressed by Standard 10.01(c) of the Ethics Code.

a. Incorrect See explanation for response d.
b. Incorrect See explanation for response d.
c. Incorrect This sounds more like the requirement for consultation.
d. CORRECT The requirements for consultation and supervision are different and, even in a supervisory relationship, the information that must be given to the client varies, depending on the exact nature of the supervision. A client must always be told when a psychologist is being supervised, but need be told the supervisor’s name when the supervisor is legally responsible for the case.

The correct answer is: the client must be told about this arrangement and, when the supervisor has legal responsibility for the case, must also be given the supervisor’s name.

How well did you know this?
1
Not at all
2
3
4
5
Perfectly
64
Q

Dr. Acedia receives a phone call from a woman who he was sexually involved with for six months. The affair ended four years ago and Dr. Acedia has had little contact with the woman since that time. The woman says she is having trouble getting over the death of her younger sister and would like to begin counseling with Dr. Acedia. According to the Ethics Code:
Select one:

A. it is acceptable for Dr. Acedia to work with the woman in therapy since they are no longer involved in a relationship.
B. it is acceptable for Dr. Acedia to work with the woman in therapy since the relationship ended more than two years ago.
C. it is acceptable for Dr. Acedia to work with the woman in therapy only if he determines that to do so is not clinically contraindicated and will not be exploitative.
D. it is not acceptable for Dr. Acedia to work with the woman in therapy.

A

The issue of therapy with former sex partners is covered by Standard 10.07 of the APA’s Ethics Code.

a. Incorrect See explanation for response d.
b. Incorrect See explanation for response d.
c. Incorrect See explanation for response d.
d. CORRECT Standard 10.07 states that “Psychologists do not accept as therapy clients/patients persons with whom they have engaged in sexual intimacies.”

The correct answer is: it is not acceptable for Dr. Acedia to work with the woman in therapy.

How well did you know this?
1
Not at all
2
3
4
5
Perfectly
65
Q

You hear a rumor that one of your co-workers has a habit of sexually harassing the younger female employees. You start paying closer attention to his behavior and discover that the rumors are not unfounded. Your best course of action in this situation would be to:
Select one:

A. maintain a record of your observations and present them to the company’s Director.
B. approach the young women who have been harassed and suggest they file a complaint with the Ethics Committee.
C. discuss your observations and the rumors you have heard with the co-worker.
D. do nothing since your co-worker’s behaviors are basically harmless.

A

Standard 3.04 of the Ethics Code requires that psychologists attempt to minimize harm against individuals with whom psychologists work, and Standards 1.04 and 1.05 require psychologists to take action when a colleague is acting unethically.

a. Incorrect Maintaining a record of your observations is not necessarily something you would need to do. Also, making a report to the Director of the organization would, in most cases, not be the best initial course of action.
b. Incorrect This might be acceptable. A problem, though, is there is nothing in the question that says your co-worker is a psychologist or other professional. Be sure to read carefully!
c. CORRECT This is always a best first course of action if you feel comfortable in confronting the individual who has allegedly acted unethically.
d. Incorrect As a psychologist, you have a responsibility to take action when you see a colleague sexually harassing co-workers and others.

The correct answer is: discuss your observations and the rumors you have heard with the co-worker.

How well did you know this?
1
Not at all
2
3
4
5
Perfectly
66
Q

A psychologist wants to assess a client with a test that has not yet been validated for the client’s ethnic group. To be consistent with the provisions of the Ethics Code, the psychologist:
Select one:

A. should explain the limitations of the test results to the client.
B. should assess the client only if the test results are “imminently needed.”
C. should assess the client only if the assessment has been approved by the insurance company.
D. should not assess the client using the test under any circumstances.

A

Psychologists use assessment instruments whose validity and reliability have been established for use with members of the population tested. When such validity or reliability has not been established, psychologists describe the strengths and limitations of test results and interpretation [Ethical Standard 9.02(b)].

a. CORRECT This answer is most consistent with the requirements of the Ethics Code.
b. Incorrect See explanation above.
c. Incorrect See explanation above.
d. Incorrect See explanation above.

The correct answer is: should explain the limitations of the test results to the client.

How well did you know this?
1
Not at all
2
3
4
5
Perfectly
67
Q

You suspect a colleague of violating ethical guidelines. To be consistent with the provisions of the Ethics Code, you should:
Select one:

A. report the colleague to the local Ethics Committee immediately.
B. report the colleague to the local Ethics Committee only if the ethical violation is a serious one.
C. attempt to handle the situation informally by discussing the violation with the colleague and file a report only if he is uncooperative.
D. either handle the situation informally or file a report, depending on which course of action you believe is most appropriate.

A

The provisions of the Ethics Code allow psychologists to use their own discretion about the best course of action in cases where another psychologist is suspected of acting unethically.

a. Incorrect See the explanation for response d.
b. Incorrect See the explanation for response d.
c. Incorrect See the explanation for response d.
d. CORRECT Psychologists can either attempt to resolve an ethical infraction informally or file a complaint with the Ethics Committee, depending on which course of action they deem most appropriate. (Of course, it is always necessary to consider client confidentiality first before taking any action, but none of the responses given address this issue.)

The correct answer is: either handle the situation informally or file a report, depending on which course of action you believe is most appropriate.

How well did you know this?
1
Not at all
2
3
4
5
Perfectly
68
Q

An expert witness is recognized by the court to be qualified to offer opinions on a specific issue. To qualify as an expert witness, a psychologist must:
Select one:

A. have a valid professional license.
B. have a relevant doctoral degree from an accredited school.
C. have appropriate education, training, and experience.
D. be certified as a forensic psychologist by the ABPP.

A

An expert witness is ordinarily approved by the court through the process of voir dire during which the witness’s education, training, and experience are examined.

a. Incorrect See explanation for response c.
b. Incorrect See explanation for response c.
c. CORRECT Requirements for expert testimony are provided in state and federal law. For example, Federal Rule of Evidence 702 states: “A witness who is qualified as an expert by knowledge, skill, experience, training, or education may testify in the form of an opinion or otherwise if: (a) the expert’s scientific, technical, or other specialized knowledge will help the trier of fact to understand the evidence or to determine a fact in issue; (b) the testimony is based on sufficient facts or data; (c) the testimony is the product of reliable principles and methods; and (d) the expert has reliably applied the principles and methods to the facts of the case.”
d. Incorrect See explanation for response c.

The correct answer is: have appropriate education, training, and experience.

How well did you know this?
1
Not at all
2
3
4
5
Perfectly
69
Q

Dr. Baron’s husband owns an advertising agency that caters to small businesses. One of Dr. Baron’s clients has just started a new business and tells Dr. Baron that he’s in need of someone to handle his advertising. If Dr. Baron recommends her husband, this is:
Select one:

A. acceptable since her husband’s business is unrelated to her work as a therapist.
B. acceptable as long as Dr. Baron never discusses the client with her husband.
C. acceptable as long as Dr. Baron discusses the possible consequences of the situation with the client.
D. unacceptable because of Dr. Baron’s therapeutic relationship with the client.

A

This issue is not addressed directly by the Ethics Code. However, the general “spirit” of the Code is not to become involved with a therapy client in any way that is outside the scope of the therapeutic relationship.

a. Incorrect See explanation for response d.
b. Incorrect See explanation for response d.
c. Incorrect See explanation for response d.
d. CORRECT R. Woody addresses this issue in Legally safe mental health practice: Psycholegal questions and answers (Madison, CT, Psychosocial Press, 1997). He argues that any recommendations to a client should be limited to issues that are within the “perimeter of treatment” and that do not benefit the therapist beyond his/her customary fee.

The correct answer is: unacceptable because of Dr. Baron’s therapeutic relationship with the client.

How well did you know this?
1
Not at all
2
3
4
5
Perfectly
70
Q

During your first session with a client, she reveals that she’s been seeing another therapist for the past four months. The woman seems unsure about why she has come to see you. Your best course of action would be to:
Select one:

A. continue seeing the client and help her clarify her motives and goals for coming to see you.
B. explain to the client that it would be unethical to see her while she is receiving similar services from another therapist and discontinue seeing her.
C. contact the other therapist to see if your services are redundant and, if they are, tell the client you cannot continue seeing her while she is seeing the other therapist.
D. help the client clarify her reasons for seeing both you and the other therapist and then evaluate whether it is appropriate for her to continue seeing both of you.

A

This is a fairly easy question. You’d want to make sure that you’ve read all of the responses before answering, however, since response a is not entirely incorrect but response d is better.

a. Incorrect See explanation for response d.
b. Incorrect See explanation for response d.
c. Incorrect See explanation for response d.
d. CORRECT Although psychologists are not prohibited from seeing clients who are receiving services from another mental health professional, a psychologist would not want to duplicate the services of another professional for an extended period of time. From both a clinical and ethical perspective, the best course of action would be to help the woman clarify her goals and needs with regard to the services provided by you and the other therapist and then determine the best course of action.

The correct answer is: help the client clarify her reasons for seeing both you and the other therapist and then evaluate whether it is appropriate for her to continue seeing both of you.

How well did you know this?
1
Not at all
2
3
4
5
Perfectly
71
Q

The use of a collection agency to collect delinquent fees from a former client is:
Select one:

A. unethical and illegal.
B. unethical but legal.
C. ethical under certain circumstances.
D. always ethical.

A

A therapist, like anyone else, has a right to be paid for his/her services.

a. Incorrect See explanation for response c.
b. Incorrect See explanation for response c.
c. CORRECT The use of a collection agency is not unethical as long as certain precautions are taken: Ideally, the client should be told at the outset of therapy what the therapist’s policy is regarding the collection of delinquent fees. Even more important, the client should be notified of the intention to use a collection agency if delinquent fees are not paid within a specified period of time (See Standard 6.04(e) in the Ethics Code).
d. Incorrect See explanation for response c.

The correct answer is: ethical under certain circumstances.

How well did you know this?
1
Not at all
2
3
4
5
Perfectly
72
Q

You are hired by a university to teach a graduate seminar in the psychology department. In terms of the Ethics Code, you are explicitly required to do which of the following?
Select one:

A. ensure that the course content reflects the most recent research in the area
B. ensure that accurate information about the seminar is available to all interested parties
C. ensure that the seminar content is consistent with the interests and needs of the students
D. ensure that the grading and evaluation of students is fair

A

The issue addressed by this question is covered in Ethical Standard 7.02 of the Ethics Code.

a. Incorrect This is generally a good idea but it is not explicitly required by the Code. Also, there would be times when it would not apply - e.g., when the seminar is covering the history of clinical psychology.
b. CORRECT Of the responses given, only this one represents the explicit requirements of the Ethics Code.
c. Incorrect Although student interests and needs are a consideration when defining course content, they are certainly not the only consideration. Also, this is not explicitly required by the Code.
d. Incorrect Again, this is a good idea, but it is not specifically stated in the Ethics Code.

The correct answer is: ensure that accurate information about the seminar is available to all interested parties

How well did you know this?
1
Not at all
2
3
4
5
Perfectly
73
Q

With regard to sexual harassment, the APA’s Ethical Principles of Psychologists and Code of Conduct:
Select one:

A. prohibits denying employment, tenure, or promotion to an individual because he/she has filed a sexual harassment charge.
B. prohibits denying employment, tenure, or promotion to an individual because he/she has filed or is the subject of a sexual harassment charge.
C. prohibits denying employment, tenure, or promotion to an individual because he/she has filed or is the subject of a sexual harassment charge or has been found guilty of sexual harassment.
D. none of the above.

A

This issue is addressed in Standard 1.08 of the Ethics Code.

a. Incorrect See explanation for response b.
b. CORRECT This response best summarizes the provisions of Standard 1.08. While the Code prohibits denying a person employment, tenure, or promotion solely on the basis of having filed or been the subject of a sexual harassment charge, the Code also states that, “This does not preclude taking action based upon the outcome of such proceedings or considering other appropriate information.”
c. Incorrect See explanation for response b.
d. Incorrect See explanation for response b.

The correct answer is: prohibits denying employment, tenure, or promotion to an individual because he/she has filed or is the subject of a sexual harassment charge.

How well did you know this?
1
Not at all
2
3
4
5
Perfectly
74
Q

A therapist has been seeing a couple for several weeks. Although they seem reluctant to bring in their 13 year old daughter, the therapist finally convinces them that the problems they have been experiencing seem to be related to their daughter and that it would be good to include her in therapy sessions. It soon becomes clear that the girl has been sexually abused by her father. When the therapist confronts the man about this, he admits that he molested his daughter on two occasions but says that he has not done so in over a year. The girl confirms that this is true. The parents plead with the therapist not to make a report to the authorities and promise that they will continue in treatment to make sure that the father doesn’t sexually abuse the girl again. The therapist decides to comply with the wishes of the parents since the abuse occurred in the past and the family is apparently committed to obtaining psychological help. Which of the following statements best describes the ethical and legal issues in this case?
Select one:

A. The therapist is acting ethically and legally since he is clearly taking the welfare of his clients into consideration.
B. The therapist is acting ethically since the abuse has occurred in the past but may be acting illegally.
C. The therapist is acting unethically and illegally by not making a report to the appropriate authorities.
D. There is not enough information to make a judgment in this case.

A

In their discussion of a similar case, COPS (American Psychologist, July 1988) concluded that “in view of APA policy statements and guidelines and relevant state laws pertaining to the reporting of child abuse, the psychologist was obligated to report the abuse to the appropriate state social service authorities. Failing to do so violated both professional standards and legal obligations.”

a. Incorrect A psychologist is legally and ethically bound to report cases of child abuse.
b. Incorrect The fact that the abuse occurred in the past does not free a psychologist from the responsibility of making a report to the appropriate authorities.
c. CORRECT The psychologist’s failure to make a child abuse report in this situation violates ethical and legal requirements.
d. Incorrect There is sufficient information to conclude that a report should have been made.

The correct answer is: The therapist is acting unethically and illegally by not making a report to the appropriate authorities.

How well did you know this?
1
Not at all
2
3
4
5
Perfectly
75
Q

During the first therapy session with a new client, you realize that he is the father of a young woman you have been seeing in therapy for three months. The daughter is 22 years old and no longer lives with her parents. The most relevant ethical issue in this situation is:
Select one:

A. confidentiality.
B. multiple relationship.
C. nonmaleficience.
D. informed consent.

A

This is a straightforward question that may have seemed “too obvious.” When taking the exam, be sure not to sabotage your ability to choose the correct response to an easy item by assuming that it is a “trick question.”

a. Incorrect See explanation for response b.
b. CORRECT Multiple relationships occur when a psychologist is involved in more than one role with a person (and at least one role is a professional relationship) or has a professional relationship with one person and becomes involved in a relationship with another individual who is related to that person. In most situations, multiple relationships should be avoided.
c. Incorrect See explanation for response b.
d. Incorrect See explanation for response b.

The correct answer is: multiple relationship.

How well did you know this?
1
Not at all
2
3
4
5
Perfectly
76
Q

A psychologist is working in the employee assistance program (EAP) of a large corporation. She is approached by a supervisor who referred an employee to the EAP several weeks ago when the employee was having problems at home that began interfering with her job performance. The employer now wants to know if the employee has been receiving counseling from the psychologist and, if so, how the sessions are going. In fact, the employee has seen the psychologist four times and has another session scheduled for the next week. The psychologist should:
Select one:

A. inform the supervisor that she has seen the employee but provide no information about the employee’s treatment.
B. inform the supervisor that she has seen the employee and provide a summary of the employees progress with regard to the reason for the referral.
C. remind the supervisor of the need to maintain confidentiality and provide him with no information about the employee.
D. tell the supervisor that she will have to get permission from the employee before providing the supervisor with any information about the employees progress.

A

Overall, confidentiality in an EAP is dealt with in the same way it is handled elsewhere.

a. Incorrect See explanation for response c.
b. Incorrect See explanation for response c.
c. CORRECT The confidentiality of clients of EAPs may be breached only in legally defined situations - e.g., when the client signs a release or is believed to be a danger to him/herself or others. Therefore, the psychologist would not even want to say anything that indicates whether or not she is seeing the employee (answer d).
The literature has changed a bit regarding this issue. The Ethics and Professional Issues chapter provides this discussion about the issue:
Employee assistance programs (EAPs): Confidentiality may be breached in the context of an EAP in the same situations that it can be breached in other contexts - that is, when the client signs an authorization for release of confidential information and in situations when disclosing confidential information without an authorization is permitted or required by law (e.g., when the client is believed to be a danger to him/herself or others).
The correct answer to this question is most consistent with this excerpt.
d. Incorrect See explanation for response c.

The correct answer is: remind the supervisor of the need to maintain confidentiality and provide him with no information about the employee.

How well did you know this?
1
Not at all
2
3
4
5
Perfectly
77
Q

Which of the following best describes the requirements of the ethics codes of the American and Canadian Psychological Associations with regard to obtaining informed consent from research participants?
Select one:

A. An informed consent is always required unless the study will involve deception and the deception meets ethical guidelines.
B. An informed consent is not required as long as participants will be debriefed at the end of the study.
C. An informed consent may not be required when the study is unlikely to cause harm or is exempted by legal or institutional regulations.
D. An informed consent is always required.

A

Informed consents are addressed in Standards 8.02 and 8.05 of the APA’s Ethics Code and Principles I.19 and I.20 of the Canadian Code of Ethics.

a. Incorrect An informed consent cannot be obtained when a study involves deception, but there are other circumstances when an informed consent is not necessary, so this is not the best answer.
b. Incorrect Debriefing is not a substitute for obtaining an informed consent.
c. CORRECT This answer best describes ethical requirements regarding dispensing with informed consents in research.
d. Incorrect Informed consents are not always required.

The correct answer is: An informed consent may not be required when the study is unlikely to cause harm or is exempted by legal or institutional regulations.

How well did you know this?
1
Not at all
2
3
4
5
Perfectly
78
Q

Of the cases processed by the APA’s Ethics Committee as the result of revocation of a psychologist’s license by a state licensing board, the most common complaint processed by the committee is which of the following?
Select one:

A. loss of licensure in another jurisdiction
B. sexual misconduct
C. breach of confidentiality
D. issues related to competence

A

The correct answer is: B. A review of the reports of the APA’s Ethics Committee published in the American Psychologist from 2000 through 2008 reveals that sexual misconduct with an adult is the most frequent reason for the complaints processed by the Ethics Committee; followed by, in order, inappropriate practice involving child custody, nonsexual dual relationship, and inappropriate practice involving insurance or fees.

The correct answer is: sexual misconduct

How well did you know this?
1
Not at all
2
3
4
5
Perfectly
79
Q

A psychologist conducts a custody evaluation for a divorce proceeding. In terms of making a specific recommendation about custody, the psychologist should:
Select one:

A. avoid making a recommendation and, instead, leave the final decision to the court.
B. make a recommendation only if the assessment has involved procedures that have been validated for this purpose.
C. make a recommendation only if he/she was not retained by one of the parents but was retained by the court.
D. make a recommendation only if it is based on adequate psychological data and consideration of the best interests of the child.

A

There are no prohibitions against making recommendations in custody situations. However, a psychologist must always be very careful about recommendations that may have an important impact on the lives of others.

a. Incorrect Making specific recommendations is not always prohibited.
b. Incorrect While this is important, it is not the best answer of those given.
c. Incorrect This doesn’t really make any sense since, in many cases, the psychologist will have been hired by one of the parties rather than by the court.
d. CORRECT This is the closest to the provisions of APA’s Guidelines for Child Custody Evaluations in Family Law Proceedings.

The correct answer is: make a recommendation only if it is based on adequate psychological data and consideration of the best interests of the child.

How well did you know this?
1
Not at all
2
3
4
5
Perfectly
80
Q

You receive a subpoena duces tecum requiring you to testify about a current client at a trial and to bring specific documents from the client’s file. You contact the client who states that he does not want you to testify or release his records. You should:
Select one:

A. notify the court that you will not be appearing at the trial because the client has not given you permission to do so.
B. contact the attorney who issued the subpoena to request that you be released from it.
C. advise the attorney who issued the subpoena that you will not be appearing at the trial because the client has not given permission for you to release confidential information.
D. do nothing further until you receive an order from the court.

A

A subpoena legally requires a psychologist to appear at a designated time at a deposition or trial. When the client invokes the privilege, the subpoena is still valid and the psychologist must appear unless he/she has been released from it.

a. Incorrect See explanation for response b.
b. CORRECT This issue is addressed by S. H. Behnke and J. T. Hilliard (The essentials of Massachusetts mental health law, W. W. Norton, New York, 1998). They note that, when the client invokes the privilege, the psychologist should contact the client’s attorney or his/her own attorney to discuss the matter. In addition, the psychologist can contact the attorney who issued the subpoena to request to be released from it. Unless the psychologist receives a written release, he/she must appear as requested (but, of course, will not reveal confidential information without the client’s consent). Additional information about responding to a subpoena is provided in the Ethics and Professional Issues chapter of the written study materials.
c. Incorrect See explanation for response b.
d. Incorrect See explanation for response b.

The correct answer is: contact the attorney who issued the subpoena to request that you be released from it.

How well did you know this?
1
Not at all
2
3
4
5
Perfectly
81
Q

To be consistent with ethical requirements, when asked by a client’s insurance company to electronically transfer confidential information about the client to the company, you should:
Select one:

A. refuse to do so.
B. also send a written (non-electronic) copy of the information to the company.
C. make sure the records are clearly marked “confidential.”
D. make sure that personal identifiers are coded or removed.

A

Standard 6.02(b) of the APA’s Ethics Code states that, “If confidential information concerning recipients of psychological services is entered into databases or systems of records available to persons whose access has not been consented to by the recipient, psychologists use coding or other techniques to avoid the inclusion of personal identifiers.”

a. Incorrect See explanation for response d.
b. Incorrect See explanation for response d.
c. Incorrect See explanation for response d.
d. CORRECT Since you wouldn’t have control over who sees the records once they’ve been transmitted, this course of action would most likely ensure that client information is kept confidential. You would probably also want to mark the information as confidential but this would not be adequate.

The correct answer is: make sure that personal identifiers are coded or removed.

How well did you know this?
1
Not at all
2
3
4
5
Perfectly
82
Q

A clinical psychologist routinely administers personality tests to her clients to assist in diagnosis. She also uses the test data in her ongoing research project. This is:
Select one:

A. ethical as long as the identity of the clients is not made available to the research project.
B. ethical as long as the research is exempt from an informed consent.
C. unethical unless she gets each client’s consent to participate in the research study.
D. unethical since this represents a multiple relationship.

A

It would be necessary to obtain an informed consent from the clients before conducting the assessment, and the informed consent must include “an explanation of the nature and purpose of the assessment” [Standard 9.03(a)].

a. Incorrect This would not be sufficient.
b. Incorrect Psychological testing ordinarily does require a consent both in therapy and research settings, and, even when a formal consent is not required, appropriate information about the purpose of testing is mandated.
c. CORRECT The clients believe that they are being tested for diagnostic and treatment purposes. To use the test data for other reasons without their consent would be unethical.
d. Incorrect Dual (multiple) relationships are, of course, discouraged but are not entirely prohibited. There is no information provided in the question that suggests that this situation would constitute a potentially harmful multiple relationship, so this is not the best answer.

The correct answer is: unethical unless she gets each client’s consent to participate in the research study.

How well did you know this?
1
Not at all
2
3
4
5
Perfectly
83
Q

During a court-ordered evaluation of a defendant to determine his competence to stand trial, the defendant reveals information that confirms his guilt. The psychologist conducting the evaluation should:
Select one:

A. include this information in his evaluation report since the evaluation was court-ordered.
B. include the information in his evaluation report because privilege is waived whenever a defendant’s mental status is part of the defense.
C. include only information relevant to the defendant’s competence in the evaluation report.
D. not provide any information to the court without a waiver of confidentiality from the defendant or his attorney.

A

A competence to stand trial hearing is conducted to determine a defendant’s current mental status. Although defendants cannot ordinarily be forced to provide incriminating evidence against themselves (as provided by the Fifth Amendment), most jurisdictions do compel defendants to undergo a competence evaluation if the court determines it is appropriate.

a. Incorrect See explanation for response c.
b. Incorrect See explanation for response c.
c. CORRECT The defendant is protected in this situation by laws that prohibit the use of evaluation data to determine the defendant’s guilt without his/her consent. In discussing this issue, G. B. Melton et al. conclude that the therapist’s best course of action is not to include any possibly incriminating evidence in the competency report but, instead, to include only information relevant to determining the defendant’s competence to stand trial (Psychological evaluations for the court, New York, Guilford, 1987).
d. Incorrect See explanation for response c.

The correct answer is: include only information relevant to the defendant’s competence in the evaluation report.

How well did you know this?
1
Not at all
2
3
4
5
Perfectly
84
Q

Which of the following is a correct use of HIV- or AIDS-related language?
Select one:

A. AIDS virus
B. AIDS-infected person
C. HIV virus
D. person with HIV

A

Several organizations have published guidelines for referring to HIV and AIDS. The following information is from the Style Sheet for HIV published by the Joint United Nations Programme on HIV/AIDS.

a. Incorrect There is no “AIDS virus.” People with AIDS are infected with the human immunodeficiency virus (HIV).
b. Incorrect People are infected with HIV, not AIDS.
c. Incorrect HIV includes the word “virus,” so HIV virus is redundant.
d. CORRECT This phrase is consistent with the Joint United Nation’s Style Sheet as well as with the APA’s Publication Manual guidelines for the use of nonbiased language.

The correct answer is: person with HIV

How well did you know this?
1
Not at all
2
3
4
5
Perfectly
85
Q

The duty to warn/protect established by the Tarasoff decision applies to:
Select one:

A. clients who are a danger to themselves.
B. clients who have committed a past crime against a person or property.
C. clients who are a danger to an identifiable person or, in some jurisdictions, an identifiable class (group) of persons.
D. clients who may pose a danger to others because of past violent behavior or their current mental state.

A

The application of the Tarasoff decision remains controversial and continues to be modified by case law.

a. Incorrect See explanation for response c.
b. Incorrect See explanation for response c.
c. CORRECT This response best describes the current status of Tarasoff. Most jurisdictions that have adopted this ruling require psychologists to warn/protect an identifiable potential victim of a client; and, in some jurisdictions, this has been extended to an identifiable class of victims. Additional information about the Tarasoff decision is provided in the chapter on ethics and professional issues in the written study materials.
d. Incorrect See explanation for response c.

The correct answer is: clients who are a danger to an identifiable person or, in some jurisdictions, an identifiable class (group) of persons.

How well did you know this?
1
Not at all
2
3
4
5
Perfectly
86
Q
An organizational psychologist conducts a(n) \_\_\_\_\_\_\_\_\_ to determine the relative worth of a job compared to other jobs.
Select one:
A. utility analysis
B. needs assessment
C. performance appraisal
D. job evaluation
A

For the exam, you want to be familiar with several “vocabulary words” that describe important activities in organizational settings.

a. Incorrect Utility refers to the feasibility of a measure, often in terms of its costs.
b. Incorrect A needs assessment (analysis) is conducted to identify training needs.
c. Incorrect A performance appraisal is an assessment of an employee’s job performance.
d. CORRECT Job evaluations are conducted for the purpose of establishing the worth of a job so that an appropriate wage can be defined for that job.

The correct answer is: job evaluation

How well did you know this?
1
Not at all
2
3
4
5
Perfectly
87
Q

An assumption underlying the notion of “groupthink” is that:
Select one:

A. pressures toward uniformity limit task effectiveness.
B. increasing cohesiveness results in more extreme decisions.
C. excessive pressures toward conformity produce reactance.
D. lack of familiarity with task demands leads to maintenance of the status quo.

A

Groupthink is characterized by a decrease in willingness to consider divergent points of view, resulting in inappropriate decisions and actions (although not necessarily more extreme ones).

The correct answer is A.

According to Janis (1972, 1982), groupthink results when groups become excessively cohesive. For information on other conditions that contribute to groupthink, see the Industrial-Organizational chapter of the written study materials.

The correct answer is: pressures toward uniformity limit task effectiveness.

How well did you know this?
1
Not at all
2
3
4
5
Perfectly
88
Q

A company decides to change the way it pays its sales force by moving away from uncertain commissions to a more predictable salary. According to Maslow’s need hierarchy theory, this change will have the greatest impact on employees whose most prepotent need is which of the following?
Select one:

A. growth
B. esteem
C. safety
D. existence

A

Maslow distinguished between five basic needs: physiological, safety, social, esteem, and self-actualization.

a. Incorrect Growth is not one of Maslow’s needs.
b. Incorrect Esteem needs are satisfied by providing employees with opportunities that maximize self-respect and the approval and recognition of others.
c. CORRECT Safety needs are satisfied by providing employees with a secure environment. In the situation described in this question, the company is reducing uncertainty by providing a salary and, thereby, improving employees’ sense of security.
d. Incorrect Existence is not one of Maslow’s needs.

The correct answer is: safety

How well did you know this?
1
Not at all
2
3
4
5
Perfectly
89
Q

A needs assessment ordinarily consists of three components. These are:
Select one:

A. evaluative, formative, and summative analysis.
B. job, person, and organizational analysis.
C. unfreezing, changing, and refreezing.
D. knowledge, skills, and abilities.

A

A needs assessment is conducted in organizations to determine training needs.

a. Incorrect See explanation for response b.
b. CORRECT A comprehensive needs assessment includes an analysis of the job requirements, the people performing the job, and the goals of the organization. Additional information about needs assessment is provided in the Industrial-Organizational Psychology chapter of the written study materials.
c. Incorrect See explanation for response b.
d. Incorrect See explanation for response b.

The correct answer is: job, person, and organizational analysis.

How well did you know this?
1
Not at all
2
3
4
5
Perfectly
90
Q

The owner of a company has decided that, to determine how well employees are performing, employees will be watched while performing their jobs for a predetermined period of time. Based on the relevant research, you predict that the employees’ performance during the period of observation will:
Select one:

A. be inhibited if the task is easy.
B. be inhibited if the task is complex.
C. be enhanced whether the task is easy or complex.
D. be inhibited whether the task is easy or complex.

A

It has been found that, in certain circumstances, the presence of others inhibits task performance (social inhibition), while in other circumstances, the presence of others facilitates task performance (social facilitation).

a. Incorrect When the task is simple or well-learned, the presence of others tends to enhance task performance.
b. CORRECT The presence of others tends to inhibit performance when the task is complex or unfamiliar.
c. Incorrect See explanations for responses a and b.
d. Incorrect See explanations for responses a and b.

The correct answer is: be inhibited if the task is complex.

How well did you know this?
1
Not at all
2
3
4
5
Perfectly
91
Q

Research conducted by Mayo and his colleagues at the Hawthorne plant of the Western Electric Company in the mid-1920s led to identification of the “Hawthorne effect” which attributed observed increases in worker productivity to which of the following?
Select one:

A. changes in the physical characteristics of the work environment
B. a change in the reward structure that more directly linked performance to pay
C. the direct result of evaluation apprehension
D. the special attention the workers received as research participants

A

The original intent of the Hawthorne studies was to investigate the effects of altering the level of illumination and other work conditions on worker productivity. However, the results indicated that other factors - especially human motives, needs, and relationships - had a stronger impact on worker productivity and satisfaction.

a. Incorrect See explanation for response d.
b. Incorrect See explanation for response d.
c. Incorrect See explanation for response d.
d. CORRECT In one series of studies, May found that, regardless of the change in the level of illumination, worker productivity increased. Subsequent research suggested that the increase was not due to changes in illumination but to the special attention workers received as research participants and the novelty of participating in a research study.

The correct answer is: the special attention the workers received as research participants

92
Q

The major disadvantage of a behaviorally anchored rating scale (BARS) is that:
Select one:

A. it is highly susceptible to rater biases.
B. it requires considerable time and effort to develop.
C. it is lacking in content validity.
D. it doesn’t provide adequate information for employee feedback and counseling.

A

When using a behaviorally anchored rating scale, the rater rates an employee on a continuum that is anchored by critical incidents.

a. Incorrect Although a behaviorally anchored rating scale may not be as successful at reducing rater biases as its proponents claim, it isn’t “highly susceptible” to rater biases.
b. CORRECT This is the primary problem with this type of rating scale. The development of a behaviorally anchored rating scale is a team effort that involves both managers and subordinates.
c. Incorrect The method of developing this type of scale helps ensure that it has adequate content validity.
d. Incorrect An advantage of behavioral anchors (critical incidents) is that they provide an employee with specific information about his/her strengths and weaknesses.

The correct answer is: it requires considerable time and effort to develop.

93
Q

It is found that a selection test is a better predictor of job success for applicants from higher socioeconomic levels than for applicants from lower socioeconomic levels. In this situation, socioeconomic level is best described as a(n):
Select one:

A. suppressor variable.
B. dependent variable.
C. moderator variable.
D. organismic variable.

A

Any predictor that is more valid for one class or subset of examinees than it is for another is said to have differential validity.

a. Incorrect A suppressor variable is irrelevant to the criterion, but affects criterion scores and thus acts as a source of error and suppresses (reduces) the correlation between the predictor and the criterion.
b. Incorrect In a research study, the dependent variable is measured to see if it is affected by the independent variable. For example, in a study assessing the effects of marijuana on task performance, marijuana is the independent variable and task performance is the dependent variable.
c. CORRECT A moderator variable is a variable that “moderates” the validity of a test i.e., a variable that make a test a better (or worse) predictor for a particular class or subset of examinees. Socioeconomic status is, therefore, a moderator variable in this situation.
d. Incorrect An organismic variable (as opposed to a manipulated variable) is one that can be measured but cannot be manipulated or controlled by the researcher. For example, in a study assessing the relationship between gender and beer consumption, both variables are organismic since the researcher cannot control either the gender or the beer consumption of the subjects.

The correct answer is: moderator variable.

94
Q

Which of the following questions best addresses the issue of interactional justice in an organization?
Select one:

A. Did the employees consider the outcomes of an organizational decision reasonable and just?
B. Was the organizational procedure used to make the decision fair?
C. Did the employees feel they were treated fairly when informed about the decision?
D. Were the employer’s reasons for making the decision reasonable and valid?

A

Researchers interested in the impact of employee perceptions related to justice in the workplace distinguish between three types of justice - procedural, interactional, and distributive.

a. Incorrect The reasonableness or fairness of the outcomes of a decision contribute to perceptions of distributive justice.
b. Incorrect The process used to make a decision contributes to perceptions of procedural justice.
c. CORRECT The ways in which an employee is treated (e.g., how a supervisor treats a supervisee) contribute to his/her perceptions of interactional justice.
d. Incorrect The reasons that contributed to decision-making are not directly related to perceptions of distributive, procedural, or interactional justice.

The correct answer is: Did the employees feel they were treated fairly when informed about the decision?

95
Q

Research investigating the relationship between unemployment and mental health has generally found that:
Select one:

A. the rates of depression and other neurotic disorders are substantially higher among the unemployed than the employed.
B. the rates of depression and other neurotic disorders are about the same among the unemployed and the employed.
C. mental health problems are likely to occur only when symptoms were evident prior to unemployment.
D. paradoxically, about 20 to 25% of unemployed individuals report an improvement in mental health after job loss.

A

Most of the research has reported a relationship between job loss and mental and physical health problems.

a. CORRECT Several studies have found rates of depression and other neurotic disorders to be about twice as high among the unemployed.
b. Incorrect See explanation for response a.
c. Incorrect This is not always true and, therefore, not the best response.
d. Incorrect A small percentage of people report an improvement in mental health following job loss (less than 10%) and it is often only transitory.

The correct answer is: the rates of depression and other neurotic disorders are substantially higher among the unemployed than the employed.

96
Q

In evaluating the criterion-related validity of a selection test, you discover that race/ethnicity is a moderator variable. Consequently, you will be interested in determining if the test has:
Select one:

A. adequate internal validity.
B. adequate reliability.
C. differential validity.
D. convergent validity.

A

In this situation, race/ethnicity is affecting (“moderating”) the relationship between a predictor (the selection test) and the criterion.

a. Incorrect See explanation for response c.
b. Incorrect See explanation for response c.
c. CORRECT For the exam, you want to have the term “moderator variable” linked with the term “differential validity.” Whenever a characteristic is acting as a moderator variable, this means that the validity coefficients for the predictor and criterion are different for groups that differ in terms of that variable.
d. Incorrect See explanation for response c.

The correct answer is: differential validity.

97
Q

To identify “critical incidents,” you would most likely:
Select one:

A. review job descriptions.
B. ask employees what they typically do on-the-job.
C. ask supervisors to identify employee behaviors that are associated with outstanding and inferior performance.
D. ask supervisors to identify specific supervisor behaviors that are associated with outstanding employee performance.

A

The critical incident technique was developed by Flanagan in the 1950s as a performance appraisal technique.

a. Incorrect See explanation for response c.
b. Incorrect See explanation for response c.
c. CORRECT As described by Flanagan, the critical incident technique begins by asking supervisors to observe employees and identify specific behaviors associated with particularly effective and ineffective job performance.
d. Incorrect See explanation for response c.

The correct answer is: ask supervisors to identify employee behaviors that are associated with outstanding and inferior performance.

98
Q

In organizations, frame-of-reference training is used to:
Select one:

A. facilitate a job analysis.
B. improve team productivity.
C. identify appropriate criterion measures.
D. increase rater accuracy.

A

Training of raters has been found to be the most effective way of reducing rater biases.

a. Incorrect See explanation for response d.
b. Incorrect See explanation for response d.
c. Incorrect See explanation for response d.
d. CORRECT Certain types of training seem to be more effective than others. Frame-of-reference training is particularly useful. It involves familiarizing raters with the multidimensional nature of job performance and with the actual content of each performance dimension.

The correct answer is: increase rater accuracy.

99
Q

A task-oriented leader is most effective when the work situation is characterized either by good leader-member relations, a structured task, and strong leader power or by moderately poor leader-member relations, an unstructured task, and weak leader power. This is predicted by which of the following?
Select one:

A. Fiedler’s contingency theory
B. Hersey and Blanchard’s situational leadership model
C. House’s path-goal theory
D. Vroom, Yetton, and Jago’s normative model

A

Of the theories listed, only one predicts that a task-oriented leader is most effective in extreme (highly favorable and highly unfavorable) conditions.

a. CORRECT Fielder proposed that low-LPC (task-oriented) leaders are most effective when the situation is either very favorable or very unfavorable in terms of the leader’s ability to lead.
b. Incorrect See explanation for response a.
c. Incorrect See explanation for response a.
d. Incorrect See explanation for response a.

The correct answer is: Fiedler’s contingency theory

100
Q

According to ______________, a person’s career concept can be described as linear, expert, spiral, or transitory.
Select one:

A. Super
B. Holland
C. Brousseau and Driver
D. Tiedeman and O’Hara

A

For the exam, you want to be familiar with the theories of all of the individuals listed in the answers to this question. These are described in the Industrial-Organizational Psychology chapter of the written study materials.

a. Incorrect See explanation for response c.
b. Incorrect See explanation for response c.
c. CORRECT Brousseau and Driver distinguish between the four career concepts listed in the question and describes them in terms of three dimensions: frequency of job change, direction of change, and type of change in job content.
d. Incorrect See explanation for response c.

The correct answer is: Brousseau and Driver

101
Q

From the perspective of equity theory, a worker’s motivation is related to his/her perceptions regarding fairness of:
Select one:

A. outcomes.
B. input/outcome ratios.
C. effort/performance ratios.
D. negative and positive outcomes.

A

Equity theory links motivation to social comparisons.

a. Incorrect See explanation for response b.
b. CORRECT According to equity theory, employees compare their own input/outcome ratios to the ratios of others doing similar work. Additional information about equity theory is provided in the Industrial-Organizational Psychology chapter of the written study materials.
c. Incorrect See explanation for response b.
d. Incorrect See explanation for response b.

The correct answer is: input/outcome ratios.

102
Q

Which of the following theories would predict that job enrichment would have a detrimental effect on job performance?
Select one:

A. two-factor
B. activation
C. equity
D. scientific management

A

Job enrichment involves altering the nature of a job in order to improve worker motivation and satisfaction.

a. Incorrect Job enrichment was originally developed from two-factor theory, which proposes that job motivation and satisfaction are related to the availability of motivator factors (e.g., responsibility, recognition, independence).
b. Incorrect Activation theory proposes that energy expenditure and performance are positively related to stimulus variability. Enriched jobs provide stimulus variability and therefore should increase activation (motivation) level and performance.
c. Incorrect Equity theory would also view job enrichment as positive since it would increase the outcome portion of the input-outcome ratio and thereby make inequities less likely.
d. CORRECT According to scientific management theory, workers are motivated only by external rewards (especially pay), not the intrinsic factors that are provided by job enrichment.

The correct answer is: scientific management

103
Q

The theory of social facilitation predicts that:
Select one:

A. the mere presence of others increases the likelihood that the dominant response will be activated.
B. when members of opposing groups are of equal status, increased contact between groups reduces intergroup hostility.
C. people will work less hard when in a group than when alone.
D. people believe that others think and act more like them than they actually do.

A

The theory of social facilitation predicts that, under certain circumstances, the mere presence of others facilitates (enhances) performance.

a. CORRECT If the dominant response is the right one, then the presence of others will enhance performance. (When the dominant response is incorrect, the presence of others results in social inhibition.)
b. Incorrect This describes the equal status contact hypothesis.
c. Incorrect This describes social loafing.
d. Incorrect This response refers to the false consensus bias.

The correct answer is: the mere presence of others increases the likelihood that the dominant response will be activated.

104
Q

Due to recent changes in technology, a company vice president realizes that many employees will need to be retrained. The first step in identifying training needs will most likely be:
Select one:

A. performing a job evaluation.
B. conducting a needs assessment.
C. identifying training objectives.
D. performing a personnel audit.

A

This question is pretty straightforward. It simply requires you to be familiar with the terms used in organizational psychology to describe the procedures involved in developing training programs.

a. Incorrect Job evaluations are used to set wages and salaries.
b. CORRECT Needs analysis (assessment) is done to determine training needs. It encompasses several other analyses such as task (job) analysis and person analysis.
c. Incorrect This is an outcome of a needs analysis.
d. Incorrect A personnel audit is part of an organizational assessment which, in turn, is part of a needs assessment.

The correct answer is: conducting a needs assessment.

105
Q

“Criterion contamination” is most likely to be a problem when the measure of job performance:
Select one:

a. is subjectively scored.
b. is objectively scored.
c. has limited “floor” or “ceiling.”
d. is susceptible to “practice effects.”

A

Criterion contamination occurs when a rater’s knowledge of a ratee’s performance on the predictor biases how the rater scores the ratee on the criterion.

a. CORRECT - A subjectively scored measure of job performance (criterion measure) is susceptible to a variety of rater biases, including the bias that produces criterion contamination.
b. Incorrect - See explanation above.
c. Incorrect - See explanation above.
d. Incorrect - See explanation above.

The correct answer is: is subjectively scored.

106
Q

The most important contributor to a predictor’s incremental validity is its criterion-related validity coefficient: The higher the coefficient, the greater the validity. However, incremental validity is also affected by other factors. Specifically, a predictor will have the greatest incremental validity when:
Select one:

A. the selection ratio is moderate and the base rate is high.
B. the selection ratio is low and the base rate is moderate.
C. the selection ratio is high and the base rate is moderate.
D. the selection ratio is moderate and the base rate is low.

A

The incremental validity of a predictor is determined by several factors including the validity coefficient, the selection ratio, and the base rate.

a. Incorrect See explanation for response b.
b. CORRECT A predictor will have the greatest incremental validity when the selection ratio is low (there are lots of applicants for the job) and when the base rate is moderate (the current selection technique is moderately accurate).
c. Incorrect See explanation for response b.
d. Incorrect See explanation for response b.

The correct answer is: the selection ratio is low and the base rate is moderate.

107
Q

Research investigating the effects of job satisfaction has generally found that:
Select one:

A. it is correlated with physical and psychological health and longevity.
B. it is correlated with physical health and longevity but not with mental health.
C. it is correlated with psychological health but not with physical health or longevity.
D. it is not correlated with physical or mental health or longevity.

A

Studies assessing the consequences of job satisfaction have found that it not only affects certain on-the-job factors but also factors outside the job environment.

a. CORRECT Job satisfaction has been found to correlate with longevity, while dissatisfaction has been associated with both psychological and physical disorders (e.g., ulcers, high blood pressure, alcoholism).
b. Incorrect Job satisfaction has been found to be related to all three factors.
c. Incorrect Job satisfaction has been found to be related to all three factors.
d. Incorrect Job satisfaction has been found to be related to all three factors.

The correct answer is: it is correlated with physical and psychological health and longevity.

108
Q

A dance teacher notices that one of her students performs better when his classmates are present than when he practices alone. This is an example of:
Select one:

A. participant modeling.
B. social facilitation.
C. deindividuation.
D. social comparison.

A

This question suggests that the student does better in the presence of other students.

a. Incorrect The question does not say that the student is actually learning by observing the behavior of other students.
b. CORRECT Social facilitation occurs when the presence of others enhances performance.
c. Incorrect Deindividuation refers to increased willingness to engage in antisocial behaviors when one’s anonymity is maintained.
d. Incorrect Social comparison theory predicts that we judge our behavior by comparing it to that of comparable others.

The correct answer is: social facilitation.

109
Q

Feelings of emotional exhaustion, depersonalization, and low personal accomplishment are symptomatic of:
Select one:

A. chronic boredom.
B. low morale.
C. repeated role conflicts.
D. job burnout.

A

The three symptoms listed have been identified as the core features of job burnout.

a. Incorrect Chronic boredom (“underload”) might induce symptoms similar to these, but this triad of symptoms has been specifically linked to burnout.
b. Incorrect The research has not linked these symptoms to low morale, which may or may not be associated with burnout.
c. Incorrect Role conflicts are a source of stress and are likely to produce unpleasant symptoms but these three have not been linked to role conflicts.
d. CORRECT These are the three components of job burnout identified by S. E. Jackson, R. L. Schwab, and R. S. Schuler (Toward an understanding of the burnout phenomenon, Journal of Applied Psychology, 71, 630-540, 1986).

The correct answer is: job burnout.

110
Q

When aversive counterconditioning is being used to eliminate a fetish, the fetish object is the:
Select one:

A. unconditioned stimulus.
B. unconditioned response.
C. conditioned stimulus.
D. conditioned response.

A

Aversive conditioning is a form of counterconditioning which involves pairing a stimulus associated with the target behavior with a stimulus that naturally evokes an unpleasant response in order to eliminate the target behavior. A fetish is a nonsexual object (e.g., a shoe or glove) that evokes a sexual response.

a. Incorrect In the terms of classical conditioning, the unconditioned stimulus is a stimulus that elicits the desired response before conditioning, or training, occurs. In aversive conditioning, the stimulus that naturally evokes the aversive response serves as the unconditioned stimulus. In the aversive conditioning of a fetish, the unconditioned stimulus is often electric shock.
b. Incorrect The unconditioned response is the response that is naturally elicited by the unconditioned stimulus.
c. CORRECT The conditioned stimulus is one which is paired with the unconditioned stimulus until the former elicits the same response as the latter. In the aversive conditioning of a fetish, the fetish is paired with an unconditioned stimulus until the fetish elicits the same response (e.g., aversion) as the unconditioned stimulus.
d. Incorrect The response that occurs as a result of conditioning is the conditioned response. The newly-acquired aversion to the fetish would represent the conditioned response.

The correct answer is: conditioned stimulus.

111
Q

As defined by Aaron Beck, __________ involves drawing a specific conclusion about an experience in the absence of supporting evidence for that conclusion.
Select one:

A. selective abstraction
B. emotional reasoning
C. personalization
D. arbitrary inference

A

For the exam, you want to be familiar with the cognitive distortions listed in the answers so that you can identify the correct response to questions like this one. Information on these distortions is provided in the Learning Theory chapter of the written study materials.

a. Incorrect See explanation for response d.
b. Incorrect See explanation for response d.
c. Incorrect See explanation for response d.
d. CORRECT Arbitrary inference occurs when a person draws a conclusion about an event or experience without there being corroborating evidence and/or when there is contradictory evidence.

The correct answer is: arbitrary inference

112
Q

Physical guidance is sometimes necessary when using _________ but must be used with caution because it can have negative consequences (e.g., can elicit aggressive or avoidance behaviors).
Select one:

A. aversive counterconditioning
B. negative practice
C. response cost
D. overcorrection

A

Physical guidance involves manually guiding the individual through the desired movements.

a. Incorrect See explanation for response d.
b. Incorrect See explanation for response d.
c. Incorrect See explanation for response d.
d. CORRECT Physical guidance may be a necessary component of overcorrection in order to ensure that the individual engages in the positive practice that is part of this intervention.

The correct answer is: overcorrection

113
Q

The ability to carry on a conversation with a friend at a party where there is loud music and a great deal of background noise is attributable to:
Select one:

A. sustained attention.
B. divided attention.
C. sequential attention.
D. selective attention.

A

You could probably figure this one out even if you’re not familiar with the research since the terms given in the answers “sound like” what they are.

a. Incorrect Sustained attention refers to the ability to maintain attention to a particular stimulus over an extended period.
b. Incorrect Divided attention refers to the ability to focus attention on more than one event simultaneously.
c. Incorrect Sequential attention is a “made up” term.
d. CORRECT Selective (focused) attention enables a person to focus on one event while filtering out or ignoring irrelevant events.

The correct answer is: selective attention.

114
Q

To establish a behavior that an individual does not naturally emit, you would use which of the following?
Select one:

A. intermittent reinforcement
B. shaping
C. response generalization
D. priming

A

One of the difficulties with operant conditioning is that it is necessary to wait until the organism emits a response so that the consequence of the response (reinforcement or punishment) can be manipulated. To overcome this problem, the desired behavior can be “shaped”.

a. Incorrect See explanation for response b.
b. CORRECT Shaping involves reinforcing successive approximations to the desired behavior. When using shaping, it isn’t necessary to wait for the behavior to be emitted. Instead, the behavior is gradually developed or formed by reinforcing responses that come closer and closer to it.
c. Incorrect See explanation for response b.
d. Incorrect See explanation for response b.

The correct answer is: shaping

115
Q

A parent decides to use time-out to reduce her son’s misbehavior by having him sit in the corner for ten minutes each time he misbehaves. The boy quickly learns, however, that, if he whines while in the corner, his mother shortens the length of the time-out period. In this situation, the mother’s willingness to shorten the time-out period is being controlled by:
Select one:

A. escape conditioning.
B. avoidance conditioning.
C. positive reinforcement.
D. positive punishment.

A

In this situation the mother’s behavior is being increased or maintained by removal of the boy’s whining.

a. CORRECT The mother’s behavior is being maintained by negative reinforcement, which is also known as escape conditioning. For additional information on escape conditioning, see the Learning Theory chapter of the written study materials.
b. Incorrect See explanation above.
c. Incorrect See explanation above.
d. Incorrect See explanation above.

The correct answer is: escape conditioning.

116
Q

A depressed client says she feels worthless and unlikable and every time she tries to do something to make things better, she is faced with barriers put up by her parents, husband, and boss, who don’t seem to recognize her efforts. In terms of the third component of Beck’s cognitive triad, you would expect the client to also say:
Select one:

A. other people are “out to get her.”
B. she had a “lousy childhood.”
C. she feels that things will never change.
D. nothing seems to make her feel good.

A

This person’s statements reflect a negative view of the SELF and the WORLD, which are two of the three elements of Beck’s cognitive triad.

a. Incorrect See explanation for response c.
b. Incorrect See explanation for response c.
c. CORRECT The third element is a negative view of the FUTURE. According to Beck, depressed people feel that failures, disappointments, etc. will never stop or change.
d. Incorrect See explanation for response c.

The correct answer is: she feels that things will never change.

117
Q

As described by Beck, automatic thoughts involve:
Select one:

A. interpreting experiences in patterned or automatic ways.
B. enduring schemas that have been repeatedly reinforced.
C. shoulds, “musts,” and “oughts.”
D. episodic memories.

A

Automatic thoughts are a primary target in Beck’s cognitive therapy.

a. CORRECT As their name implies, automatic thoughts are automatic or reflexive. They also usually have an interpretive component (e.g., “this is awful”).
b. Incorrect Although automatic thoughts may be the result of cognitive schemas, Beck does not associate them with repeated reinforcement.
c. Incorrect Shoulds, musts, and oughts are of interest to practitioners of REBT.
d. Incorrect See explanation above.

The correct answer is: interpreting experiences in patterned or automatic ways.

118
Q

The belief that distorted schemas develop early in life and leave the individual susceptible to depression or other disorders when faced with stress is most consistent with the views of:
Select one:

A. Lewinsohn.
B. Beck.
C. Seligman.
D. Rehm.

A

The term “schema” should have been the clue to the correct response to this question.

a. Incorrect Lewinsohn proposed that depression is related to a lack of contingent reinforcement.
b. CORRECT This is a good summary of Beck’s view.
c. Incorrect Seligman is associated with the notion of learned helplessness.
d. Incorrect Rehm is associated with the self-control model of depression, which focuses on the processes of self-monitoring, self-evaluation, and self-reinforcement.

The correct answer is: Beck.

119
Q

The underlying premise of Albert Ellis’ Rational Emotive Behavior Therapy (REBT) is that dysfunctional behaviors are:
Select one:

A. the result of irrational thoughts and beliefs.
B. the result of incongruence between self and experience.
C. the result of “automatic thoughts.”
D. the result of a lack of “awareness.”

A

Ellis’ REBT assumes that irrational ways of thinking underlie dysfunctional behaviors.

a. CORRECT This is the underlying premise of REBT.
b. Incorrect This is the underlying premise of Rogers’ client-centered therapy.
c. Incorrect Although REBT views irrational thoughts as determiners of dysfunctional behaviors, the notion of “automatic thoughts” is more characteristic of Beck’s theory of depression and other forms of maladaptive behavior.
d. Incorrect This is the underlying premise of Perls’ Gestalt therapy.

The correct answer is: the result of irrational thoughts and beliefs.

120
Q

Which of the following is an example of elaborative rehearsal?
Select one:

A. ensuring that the same cues are present when information is encoded and retrieved
B. gradually removing prompts and reinforcement during the learning process
C. practicing a new skill past the point of mastery
D. relating new information to previously acquired information

A

The rehearsal of new information helps ensure that it is transferred from short- to long-term memory. Two types of rehearsal are distinguished - maintenance and elaborative.

a. Incorrect This describes “encoding specificity.”
b. Incorrect This is not a description of elaborative rehearsal.
c. Incorrect This describes “overlearning.”
d. CORRECT The term elaborative rehearsal is used to describe the process of making new information meaningful in order to enhance its retention and retrieval. Relating new information to previously acquired information is one type of elaborative rehearsal.

The correct answer is: relating new information to previously acquired information

121
Q

A pigeon is placed in a cage that has two levers. Lever #1 delivers reinforcement on a VI-30-second schedule, while Lever #2 delivers reinforcement on a VI-60-second schedule. What proportion of the pigeon’s pecks will be on Lever #1?
Select one:

A. one-third
B. two-thirds
C. one-half
D. three-fourths

A

When using a concurrent VI schedule, each lever or key delivers reinforcement on a different variable interval schedule. In this situation, the animal matches the relative frequency of its pecks on each key to the relative frequency of reinforcement obtained with that key.

a. Incorrect See explanation for response b.
b. CORRECT In the situation described in the question, the pigeon will peck the VI-30 key twice as often as the VI-60 key. Put another way, 2/3 of the pigeon’s pecks will be on the VI-30 key and the remaining 1/3 will be on the VI-60 key.
c. Incorrect See explanation for response b.
d. Incorrect See explanation for response b.

The correct answer is: two-thirds

122
Q

Subjects in a research study are asked to read the following list of words: pin, sewing, thread, sharp, haystack, injection, point, pain. They are then asked to recall as many words from the list as possible, and many subjects recall the word “needle” even though it is not one of the words in the list. This illustrates which of the following?
Select one:

A. false memory induction
B. imagination inflation
C. Deese-Roediger-McDermott paradigm
D. Loftus misinformation effect

A

The four phenomena listed in the answers to this question have been used to study or explain false memories.

a. Incorrect The false memory induction procedure creates false memories by repeatedly asking subjects about events they never experienced.
b. Incorrect Imagination inflation involves creating false memories of an event by asking subjects to imagine the event before asking them to recall if it happened to them.
c. CORRECT The Deese-Roediger-McDermott (DRM) effect refers to the tendency to erroneously recall a word from a word list that contains semantically related words. It has been used to study false memory and identify people who are prone to creating false memories.
d. Incorrect The Loftus misinformation effect occurs when the original memory of an event is altered by subsequent exposure to misleading information about that event.

The correct answer is: Deese-Roediger-McDermott paradigm

123
Q

A friend asks you how you liked the concert you went to last Saturday night. As you try to recall the concert, you realize that your memory is being affected by other concerts you have attended in the past. In other words, your memory of last Saturday’s concert is being affected by:
Select one:

A. your implicit memory of concerts.
B. a lack of encoding specificity.
C. positive memory transfer.
D. your schema for concerts.

A

In this situation, your memory of a particular concert is affected by your previous experiences with concerts.

a. Incorrect See explanation for response d.
b. Incorrect See explanation for response d.
c. Incorrect See explanation for response d.
d. CORRECT A schema is a group or cluster of knowledge about an object or event. Your recollection of last Saturday’s concert is being affected by your “cluster of knowledge” regarding concerts.

The correct answer is: your schema for concerts.

124
Q

A client says, “I don’t know where they come from, but these thoughts about swearing and cursing just keep coming back. I just can’t seem to get them out of my mind. All I can think about is cursing and swearing.” Which of the following techniques would be most useful for alleviating this client’s problem?
Select one:

A. thought stopping
B. biofeedback
C. behavioral rehearsal
D. overcorrection

A

This client is exhibiting obsessions.

a. CORRECT Although thought stopping is often used to treat obsessions and compulsions, there is some evidence that it is most effective for obsessions, while flooding and other exposure techniques are better for eliminating compulsions. However, of the responses given, this is the best answer.
b. Incorrect Biofeedback is ordinarily used to control involuntary bodily functions and activities such as blood pressure, muscle tension, and body temperature.
c. Incorrect The technique of behavioral rehearsal is not associated with Obsessive-Compulsive Disorder.
d. Incorrect Overcorrection is used to reduce undesirable overt behaviors.

The correct answer is: thought stopping

125
Q

Which of the following provides reinforcement on a variable ratio schedule of reinforcement?
Select one:

A. surprise quizzes
B. piecework
C. bi-weekly paychecks
D. slot machines

A

When a variable ratio reinforcement schedule is being used, reinforcement is delivered after a variable number of responses.

a. Incorrect “Surprise” quizzes provide opportunities for reinforcement on a variable interval schedule.
b. Incorrect Piecework represents a fixed ratio schedule of reinforcement since workers are paid after completing a specified number of “pieces.”
c. Incorrect A bi-weekly pay schedule represents a fixed interval schedule of reinforcement.
d. CORRECT Slot machines reinforce players after a variable number of responses.

The correct answer is: slot machines

126
Q

A treatment for depression that is based on Rehm’s self-control theory is most likely to include:
Select one:

A. having the client keep a record of automatic thoughts.
B. having the client keep a record of positive experiences.
C. helping the client replace irresponsible behaviors with responsible ones.
D. using functional behavioral analysis to help the client identify the antecedents and consequences associated with maladaptive behaviors.

A

Rehm’s self-control theory is based on the assumption that depression is related to six deficits in self-control behavior: selective monitoring of negative events; selective monitoring of immediate (vs. delayed) consequences of behavior; stringent self-evaluative criteria; inaccurate attributions of responsibility; insufficient self-reward; and excessive self-punishment.

a. Incorrect This sounds more like Beck’s version of cognitive therapy.
b. CORRECT According to Rehm, depressed people focus too much on negative events. Treatment attempts to alter this by encouraging clients to recognize the positive events that occur.
c. Incorrect Although Rehm believes that depressed people make inaccurate attributions about responsibility, he doesn’t talk about “irresponsible behaviors.”
d. Incorrect Rehm’s approach does not focus on identifying the antecedents and consequences of maladaptive behaviors.

The correct answer is: having the client keep a record of positive experiences.

127
Q

A behavioral therapist is using positive reinforcement to increase a desired behavior. After the behavior is well-established, the psychologist switches from a continuous schedule of reinforcement to an intermittent one. This technique is referred to as:
Select one:

A. thinning.
B. fading.
C. shaping.
D. inoculation.

A

In this situation, the therapist is reducing (“thinning”) the amount of reinforcement.

a. CORRECT Thinning involves reducing reinforcement; for example, from a continuous to an intermittent schedule.
b. Incorrect Fading refers to the gradual removal of prompts, not reinforcements.
c. Incorrect Shaping refers to the reinforcement of successive approximations to the desired behavior.
d. Incorrect Inoculation is not a behavioral term and is a distractor.

The correct answer is: thinning.

128
Q

Use of which of the following provided Broadbent (1958) with support for his filter theory of attention?
Select one:

A. dismantling strategy
B. Stroop test
C. dichotic listening task
D. speeded-target monitoring task

A

Broadbent’s filter theory was the first of the “bottleneck” theories of attention.

a. Incorrect See explanation for response c.
b. Incorrect See explanation for response c.
c. CORRECT As described in the Learning Theory chapter of the written study materials, support for Broadbent’s theory was provided by research using the dichotic listening task in which participants listened to speech sounds presented to each ear simultaneously.
d. Incorrect See explanation for response c.

The correct answer is: dichotic listening task

129
Q

Which of the following is true about sensory memory?
Select one:

A. Information is retained for about 1/2 to 2 seconds.
B. Without rehearsal, information is retained for only 30 to 60 seconds.
C. With attention, information is retained for up to three minutes.
D. With rehearsal, information may be retained indefinitely.

A

Sensory memory holds information from all of the senses for a very brief period of time.

a. CORRECT Estimates of the duration of sensory memory vary somewhat from author to author, but it is generally described as being less than two seconds.
b. Incorrect Information in sensory memory cannot be deliberately retained through attention, rehearsal, or other techniques.
c. Incorrect See explanation above. d. Incorrect See explanation above.

The correct answer is: Information is retained for about 1/2 to 2 seconds.

130
Q

Lewinsohn’s (1974) behavioral model attributes depression to:
Select one:

A. a low rate of response-contingent reinforcement.
B. a lack of appropriate stimulus discrimination.
C. self-indoctrination.
D. negative self-attributions.

A

Lewinsohn’s (1974) behavioral model is based on operant conditioning and focuses on the consequence of behavior. Additional information about this model is provided in the Learning Theory chapter of the written study materials.

a. CORRECT Lewinsohn described depression as being primarily the result of a low rate of response-contingent reinforcement. According to his model, when a person’s behaviors - for instance, attempts to interact with family members and co-workers - are not reinforced those behaviors are extinguished. Lewinsohn also proposed that, in addition to eliminating or reducing certain behaviors, a low rate of response-contingent reinforcement elicits pessimism, low self-esteem, and other symptoms that are associated with depression.
b. Incorrect Lewinsohn focused on the lack of reinforcement rather than stimulus discrimination.
c. Incorrect See explanation for response a.
d. Incorrect See explanation for response a.

The correct answer is: a low rate of response-contingent reinforcement.

131
Q

For a therapist relying on Beck’s cognitive approach to treat a client suffering from an Anxiety Disorder, the initial cognitive restructuring will most likely focus on:
Select one:

A. reattribution.
B. cognitive rehearsal.
C. thought stopping.
D. decatastrophizing.

A

Cognitive therapists use a variety of cognitive strategies, and many of the strategies are the same regardless of the client’s disorder. However, some techniques are better suited to certain disorders, especially as initial interventions.

a. Incorrect Reattribution involves attributing responsibility to appropriate parties and is most useful when the client is depressed or guilt-ridden.
b. Incorrect Cognitive rehearsal requires the client to imagine each step involved in an activity or task.
c. Incorrect Thought stopping may be useful but it is usually classified as a behavioral (not cognitive restructuring) technique and would probably not be an initial intervention in treating a client with an Anxiety Disorder.
d. CORRECT Decatastrophizing is particularly useful for people with an Anxiety Disorder since they tend to overestimate the risk and consequences of perceived danger.

The correct answer is: decatastrophizing.

132
Q

Erik Erikson coined the term:
Select one:

A. adolescent storm and stress.
B. adolescent identity crisis.
C. midlife crisis.
D. sandwich generation.

A

Erik Erikson is probably best known for his psychosocial theory of development. One of the stages of this model is “identity versus identity confusion,” which is characteristic of adolescents.

a. Incorrect Stanley Hall discussed the storm and stress of adolescence.
b. CORRECT Erikson was the first major theorist to elucidate the notion of an adolescent identity crisis and is credited with coining the term.
c. Incorrect “Midlife crisis” was coined by the psychoanalyst Elliott Jacques in 1967.
d. Incorrect This term was not coined by Erikson.

The correct answer is: adolescent identity crisis.

133
Q

Which of the following is LEAST characteristic of infants who were exposed to cocaine in utero?
Select one:

A. They are abnormally insensitive to tactile, visual, and auditory stimulation.
B. They are excessively irritable and unresponsive to attempts to comfort.
C. They often have increased motor tone.
D. They often have a low birthweight and small head circumference.

A

Prenatal cocaine exposure is associated with both physical and behavioral problems in childhood.

a. CORRECT This is the opposite of what is true. Early exposure to cocaine is associated with oversensitivity to environmental stimuli, even stimuli of low intensity.
b. Incorrect This is one of the reasons why these children often exhibit disturbances in attachment.
c. Incorrect Cocaine-exposed children often have an unusual pattern of motor development as the result of excessive muscle tone.
d. Incorrect This is true about these children.

The correct answer is: They are abnormally insensitive to tactile, visual, and auditory stimulation.

134
Q

Piaget attributed the animistic thinking characteristic of children in the preoperational stage to:
Select one:

A. decentration.
B. lack of object permanence.
C. relativistic thinking.
D. egocentrism.

A

Familiarity with the characteristics of Piaget’s preoperational stage would have helped you choose the correct answer to this question.

a. Incorrect Decentration is an accomplishment of the concrete operational stage and contributes to the ability to conserve.
b. Incorrect Object permanence is an accomplishment of the sensorimotor stage, which precedes the preoperational stage.
c. Incorrect Relativistic thinking is not addressed in Piaget’s theory.
d. CORRECT According to Piaget, animism and magical thinking are attributable to precausal reasoning that, in turn, is due to egocentrism which underlies many of the limitations of the preoperational stage.

The correct answer is: egocentrism.

135
Q

Sexual activity among older adults is most related to:
Select one:

A. sexual activity earlier in life.
B. attitudes toward sex and sexuality.
C. interest in sex.
D. overall life satisfaction.

A

The research has found that several factors are determinants of sexual activity in old age. These factors include physical health, the availability of a partner, and past sexual activity.

a. CORRECT Once again, past behavior is a good predictor of future behavior: The research indicates that sexual activity in mid-life and earlier is a good predictor of activity in old age, especially for males.
b. Incorrect This is not as good a predictor as past activity, especially for males.
c. Incorrect Interest often exceeds activity.
d. Incorrect This is not as good a predictor as past activity.

The correct answer is: sexual activity earlier in life.

136
Q

At ______ months of age, most children string two words together to make a sentence - for example, “Daddy go,” “more juice,” and “eat cookie.”
Select one:

A. 12 to 16
B. 18 to 24
C. 26 to 32
D. 34 to 40

A

Stringing two words together to make a sentence is referred to as telegraphic speech.

a. Incorrect Between 12 and 16 months of age, infants babble and then say their first words.
b. CORRECT From about 18 to 24 months of age, infants combine two words together to form a sentence. This telegraphic speech usually contains only nouns, verbs, and adjectives.
c. Incorrect Between 26 and 32 months, most children are using three or more words to form a sentence.
d. Incorrect After 30 to 32 months of age, children use increasingly complex sentences.

The correct answer is: 18 to 24

137
Q

Studies investigating the “empty nest syndrome” have found that:
Select one:

A. marital satisfaction tends to decrease for middle-aged adults when all of their children leave home.
B. marital satisfaction tends to increase for middle-aged adults when all of their children leave home.
C. marital satisfaction tends to decrease for middle-aged men but increase for middle-aged women when all of their children leave home.
D. marital satisfaction tends to decrease for middle-aged women but increase for middle-aged men when all of their children leave home.

A

For many couples, middle age is a time of change that includes launching children into adulthood.

a. Incorrect See explanation for answer b.
b. CORRECT The studies have generally found that, contrary to what is commonly believed, men and women are most likely to experience an increase in marital satisfaction after all of their children leave home. See. e.g., V. R. Waldron and D. L. Kelley, Marriage at midlife: Counseling strategies and analytical tools, New York, Springer, 2009.
c. Incorrect See explanation for answer b.
d. Incorrect See explanation for answer b.

The correct answer is: marital satisfaction tends to increase for middle-aged adults when all of their children leave home.

138
Q

To manifest all of the following traits except _________, an offspring must inherit a homozygous pair of genes from his or her parents.
Select one:

A. blond hair
B. nearsightedness
C. Rh-negative blood
D. brown eyes

A

Some inherited traits are manifested only when an offspring inherits the same recessive gene from each parent - i.e., when the offspring inherits a homozygous pair of genes.

a. Incorrect See explanation for response d.
b. Incorrect See explanation for response d.
c. Incorrect See explanation for response d.
d. CORRECT The traits listed in answers a, b, and c (blond hair, nearsightedness, and Rh-negative blood) are all due to a pair of recessive genes, while brown eyes are transmitted by a single dominant gene. Consequently, an offspring must inherit only a single dominant gene from one parent to have brown eyes.

The correct answer is: brown eyes

139
Q

Reducing risk impact, reducing negative chain reactions, promoting self-esteem and self-efficacy, and providing opportunities are four techniques identified by __________ for promoting resiliency in high-risk children.
Select one:

A. Harlow
B. Rutter
C. Patterson
D. Bronfenbrenner

A

Knowing which of the individuals listed in the responses is most associated with research on risk and resiliency in childhood would have helped you identify the correct answer to this question.

a. Incorrect See explanation for response b.
b. CORRECT Of the individuals listed, only Rutter is associated with work on risk and resiliency. See the Lifespan Development chapter for additional information on Rutter’s research.
c. Incorrect See explanation for response b.
d. Incorrect See explanation for response b.

The correct answer is: Rutter

140
Q

In adolescence, sibling relationships usually become:
Select one:

A. less emotionally intense and more distant.
B. less egalitarian.
C. more competitive and conflictual.
D. more intimate and affectionate.

A

Although sibling relationships during childhood, adolescence, and adulthood are fairly stable, there are some predictable changes.

a. CORRECT Although there are individual differences in the nature of sibling relationships throughout the lifespan, the research has generally found that, as they develop more relationships with peers, adolescent siblings spend less time together and are less emotionally intimate with one another.
b. Incorrect This is the opposite of what is true. Sibling relationships in adolescence become less asymmetric and more egalitarian.
c. Incorrect Sibling relationships usually become less competitive and conflictual in adolescence than they were in prior years.
d. Incorrect The research has generally found that sibling relationships in adolescence are characterized by fewer interactions, less companionship, and less intimacy and affection.

The correct answer is: less emotionally intense and more distant.

141
Q

In terms of parenting styles, adolescent antisocial behavior is most associated with which of the following?
Select one:

A. authoritarian
B. rejecting-neglecting
C. permissive
D. instrumental

A

Antisocial behavior in adolescents has been linked to weak parental supervision, a lack of rules, and lax or inconsistent discipline.

a. Incorrect See explanation for response b.
b. CORRECT Of the parenting styles identified by Baumrind and others, the rejecting-neglecting style is most consistent with the parenting characteristics that have been found to predict delinquency and antisocial behavior during adolescence. See the Lifespan Development chapter for additional informtion on parenting styles.
c. Incorrect See explanation for response b.
d. Incorrect See explanation for response b. (Also note that “instrumental” is not one of Baumrind’s parenting styles.)

The correct answer is: rejecting-neglecting

142
Q

A school psychologist is trying to determine why a fourth grade student with an above-average IQ is doing poorly in school. He talks to the student’s parents and teacher separately and then meets with them jointly to discuss similarities and differences in the boy’s behaviors at home and school. From the perspective of Bronfenbrenner’s bioecological model, the psychologist is investigating which of the following?
Select one:

A. microsystem and mesosystem
B. microsystem and exosystem
C. macrosystem and mesosystem
D. macrosystem and exosystem

A

Bronfenbrenner’s model distinguishes between five interlocking contextual systems that impact a child’s development.

a. CORRECT The microsystem includes aspects of the child’s environment that affect him/her directly (e.g., school and home). The mesosystem refers to the interactions between elements of the microsystem (e.g., the interaction between home and school).
b. Incorrect The exosystem includes factors that indirectly affect the child (e.g., the parents’ workplaces and social networks).
c. Incorrect The macrosystem includes cultural values and customs, the economic system, etc.
d. Incorrect See explanations above.

The correct answer is: microsystem and mesosystem

143
Q

Which of the following aspects of perception develops FIRST during infancy?
Select one:

A. 20/20 vision
B. sensitivity to pictorial depth cues
C. preference for faces over other visual patterns
D. discrimination between happy and angry faces and sounds

A

The introduction of new methods for assessing vision and audition in infants in the last few decades has led to increased and more accurate information about perceptual development.

a. Incorrect Most experts state that children achieve 20/20 vision between 6 and 12 months of age, but some say it develops even later.
b. Incorrect Sensitivity to pictorial depth cues develops at about 6 months of age.
c. CORRECT Most studies show that preference for facial patterns over other visual patterns is apparent by two to three months of age, although there is some evidence that it may occur as early as 9 minutes of age. See, e.g., M. H. Johnson et al., Newborns’ preferential tracking of face-like stimuli and its subsequent decline, Cognition, 40, 1-19, 1991.
d. Incorrect This ability appears at about 7 months of age.

The correct answer is: preference for faces over other visual patterns

144
Q

According to Piaget, which of the following is a necessary prerequisite for the development of reversibility?
Select one:

A. self-awareness
B. meta-cognition
C. symbolism
D. hypothetico-deductive reasoning

A

Piaget argued that symbolism - and in particular language - is critical for reversibility and conservation.

a. Incorrect See explanation for response c.
b. Incorrect See explanation for response c.
c. CORRECT In Logical Operations and Social Life, Piaget states “complete reversibility presupposes symbolism” [cited in L. Smith (ed.), Sociological studies, London, Routledge, 1995].
d. Incorrect See explanation for response c.

The correct answer is: symbolism

145
Q

Offspring of parents who are both heterozygous for the phenylketonuria (PKU) gene have a ___% of having this disorder.
Select one:

A. 10
B. 25
C. 50
D. 75

A

PKU is an autosomal recessive disorder, which means that a person with the disorder has inherited one recessive allele (p) from each parent. A person is homozygous with regard to a PKU when he/she has two recessive alleles for that condition (pp), while a person is heterozygous with regard to PKU when he/she has two different alleles (Pp). (“P” refers to a normal allele and “p” refers to the recessive allele for PKU.)

a. Incorrect See explanation for answer b.
b. CORRECT When both parents are heterozygous for PKU (Pp), 25% of their offspring will not have the disorder and will not be carriers of the disorder (PP); 50% will be carriers of the disorder (Pp); and 25% will have the disorder (pp).
c. Incorrect When one parent is heterozygous for PKU (Pp) and the other is homozygous (pp), 50% of their offspring will be carriers of the disorder (Pp) and 50% will have the disorder (pp).
d. Incorrect See explanation for answer b.

The correct answer is: 25

146
Q

Research on the “synchrony effect” has established a relationship between circadian arousal, cognitive functioning, and age. These studies have found, for example, that the performance of older adults on cognitive tasks that require the ability to focus on goal-relevant information only and to inhibit prepotent responses is best in the:
Select one:

A. morning.
B. early afternoon.
C. early evening.
D. late evening.

A

Studies have found that the optimal time for the successful completion of certain types of cognitive tasks is related to circadian arousal, which varies with age: Older adults show peak arousal and task performance levels in the morning, while young adults show higher levels of both in the evening.

a. CORRECT Older adults do better on certain cognitive tasks (especially those that depend on inhibitory attentional processes) in the morning than during any other time of the day.
b. Incorrect See explanation above.
c. Incorrect See explanation above.
d. Incorrect See explanation above.

The correct answer is: morning.

147
Q
A four-year old who hates strings beans gets upset when his mother cuts them into small pieces because he thinks that he now has to eat more beans. A Piagetian would describe this as an example of which of the following?
Select one:
A. object concept
B. transduction
C. irreversibility
D. decentration
A

For the licensing exam, you want to be familiar with all of the terms listed in the answers to this question. These are described in the Lifespan Development chapter of the written study materials.

a. Incorrect As described by Piaget, the object concept (also known as object permanence) develops during the sensorimotor stage and allows a child to recognize that objects and people continue to exist even when he/she cannot perceive them.
b. Incorrect Transduction (also known as transductive and precausal reasoning) refers to the tendency of preoperational children to mentally link certain experiences, whether or not they actually have a causal relationship.
c. CORRECT Irreversibility is the inability to imagine reversing a physical operation and is a factor that contributes to a preoperational child’s inability to conserve.
d. Incorrect Decentration is the ability to focus on more than one aspect of an object or situation at a time. It is the opposite of centration, which is one of the factors that contributes to a preoperational child’s inability to conserve.

The correct answer is: irreversibility

148
Q

According to Carol Gilligan, adolescent girls need experiences that help them:
Select one:

A. plan for their future in the adult world.
B. separate successfully from their families.
C. stay connected to themselves and others.
D. explore alternative identities.

A

Gilligan considers adolescence to be a critical period for females.

a. Incorrect See explanation for response c.
b. Incorrect See explanation for response c.
c. CORRECT A number of experts have noted that girls are especially vulnerable during adolescence. Gilligan argues that this is because they are experiencing a “relational crisis” that involves psychological separation from themselves, others, and the world as a result of external pressures to fit cultural stereotypes. Consequently, during adolescence, girls need to be provided with experiences that help them “resist disconnection.” See C. Gilligan, Women’s psychological development: Implications for psychotherapy, in C. Gilligan et al. (Eds.), Women, Girls, and psychotherapy: Reframing resistance, New York, Haworth Press, 1991.
d. Incorrect See explanation for response c.

The correct answer is: stay connected to themselves and others.

149
Q

Research on Kohlberg’s theory of moral development has found that very few people ever reach Stage 6. However, some do engage in Stage 5 reasoning and base their moral judgments on:
Select one:

A. social contracts.
B. self-chosen universally-applicable standards.
C. rules and laws.
D. the motivation underlying an act.

A

Kohlberg’s theory distinguishes between three levels of moral development (preconventional, conventional, and postconventional) that each consist of two stages. See the Lifespan Development chapter for additional information about Kohberg’s theory.

a. CORRECT Stage 5 is the first stage in the postconventional level. People in this stage consider social contracts and democratically-chosen laws when making moral judgments.
b. Incorrect Internalized universal moral principles underlie the judgments made by people in Stage 6.
c. Incorrect People in Stage 4 base their moral judgments on existing rules and laws..
d. Incorrect An actor’s intentions are of concern to people in Piaget’s autonomous stage of moral development.

The correct answer is: social contracts.

150
Q

Antisocial behavior and later delinquency have been associated with:
Select one:

A. an authoritarian parenting style and consistent physical discipline.
B. an authoritarian parenting style and inconsistent, excessive discipline.
C. a laissez-faire parenting style and inconsistent warmth and concern.
D. a laissez-faire parenting style and harsh, inconsistent discipline.

A

A permissive, uninvolved parenting style has been associated with greater aggressiveness in children.

a. Incorrect See explanation for response d.
b. Incorrect See explanation for response d.
c. Incorrect See explanation for response d.
d. CORRECT Harsh inconsistent discipline, lack of positive parenting, and poor supervision have been linked with aggression and other antisocial behaviors.

The correct answer is: a laissez-faire parenting style and harsh, inconsistent discipline.

151
Q

One factor that limits the cognitive abilities of preoperational children is the inability to simultaneously consider several aspects of a situation at once. This inability is referred to as:
Select one:

A. irreversibility.
B. centration.
C. transduction.
D. decentration.

A

According to Piaget, the preoperational child is unable to conserve due to limitations in cognitive ability.

a. Incorrect Irreversibility is one of the cognitive limitations of the preoperational stage but refers to the inability to understand that an operation can be reversed.
b. CORRECT Centration refers to focusing on only one aspect of a situation while neglecting others. It is believed to be one of the primary reasons why preoperational children cannot conserve.
c. Incorrect Transductive reasoning is also characteristic of the preoperational stage. However, it refers to the child’s tendency to see a causal relationship between events that occur in sequence when none exists.
d. Incorrect Decentration is one of the abilities that emerge in the concrete operational stage and allow the child to conserve.

The correct answer is: centration.

152
Q

According to Carstensen’s (1999) socioemotional selectivity theory, differences in the preferences of adults for emotionally close versus novel social partners is related to:
Select one:

A. gender differences.
B. age differences.
C. differences in time perspective.
D. differences in basic personality traits.

A

Socioemotional selectivity theory predicts that people’s awareness of how much time they have left in life impacts their motivation and preferences, including preferences for emotionally close versus novel social partners.

a. Incorrect See explanation for response c.
b. Incorrect See explanation for response c.
c. CORRECT Research on socioemotional selectivity theory often compares the motivation and preferences of adults in different age groups. While this research generally finds that older adults have different preferences and sources of motivation than do younger adults, the differences are not due solely to age. Instead, they are often the result of awareness of time left to live as being either unlimited or limited. People who perceive time as unlimited tend to prefer novel social partners, while those who perceive time as limited prefer emotionally close social partners (and, as a result, are likely to have fewer social partners). Also, while older adults are more likely to perceive time as limited, some younger adults (e.g., those with a life-threatening illness) also have a limited time perspective and, like older adults, prefer emotionally close social partners. See, e.g., L. Carstensen, D. M. Isaacowitz, & S. T. Charles, Taking time seriously: A theory of socioemotional selectivity, American Psychologist, 54, 165-181, 1999.
d. Incorrect See explanation for response c.

The correct answer is: differences in time perspective.

153
Q

In the case of Brown v. Board of Education of Topeka (1954), research by Kenneth and Mamie Clark was used to support the argument that:
Select one:

A. school segregation contributes to a negative self-image among African American children.
B. school segregation contributes to low school achievement among African American children.
C. academic underachievement of African American children is related to parental and cultural expectations.
D. academic underachievement of African American children is related to low teacher expectations.

A

Kenneth and Mamie Clark are best known for their work on ethnic/racial identity in young children.

a. CORRECT These investigators found that young African-American children usually exhibited a preference for a White (vs. African-American) doll, and concluded that this was indicative of a poor self-image. Their findings were used by the plaintiffs in the Brown case as evidence of the negative effects of segregation on the self-image of African-American children.
b. Incorrect See explanation above.
c. Incorrect See explanation above.
d. Incorrect See explanation above.

The correct answer is: school segregation contributes to a negative self-image among African American children.

154
Q

Research investigating the effects of living with a stepparent on a child’s adjustment suggests that _________ age at the time of the remarriage and ______ gender are associated with fewer problems adjusting to a stepparent.
Select one:

A. younger; female
B. younger; male
C. older; female
D. older; male

A

Although the research on the effects of stepparents on children’s adjustment has not produced entirely consistent results, some generalizations can be made.

a. Incorrect See explanation for response b.
b. CORRECT The studies have generally shown that younger children have less trouble adjusting to a stepparent than do older school-age children and adolescents. In addition, while research on the impact of the child’s gender on adjustment to a stepparent is not entirely consistent, most studies have found that girls have more adjustment problems than boys do or, alternatively, that gender does not have a substantial impact on adjustment.
c. Incorrect See explanation for response b.
d. Incorrect See explanation for response b.

The correct answer is: younger; male

155
Q

Which of the following lists the correct order of the stages of grief as described by Elizabeth Kubler-Ross (1969)?
Select one:

A. anger, depression, denial, bargaining, acceptance
B. denial, anger, depression, bargaining, acceptance
C. bargaining, denial, depression, anger, acceptance
D. denial, anger, bargaining, depression, acceptance

A

Based on her research with terminally ill patients, Kubler-Ross (1969) identified five stages in coming to terms with one’s own death.

a. Incorrect See explanation for response d.
b. Incorrect See explanation for response d.
c. Incorrect See explanation for response d.
d. CORRECT This answer accurately lists the order of Kubler-Ross’s stages of grief. Note, however, that subsequent research has found that the stages do not necessarily occur in the order identified by Kubler-Ross and that stages may be repeated.

The correct answer is: denial, anger, bargaining, depression, acceptance

156
Q

The onset of stranger anxiety varies somewhat from child to child but, for most children, begins when the child is between ___________ months of age.
Select one:

A. 2 to 4
B. 5 to 7
C. 8 to 10
D. 11 to 13

A

Stranger anxiety is a normal developmental phenomenon and refers to the anxiety reaction exhibited by young children in the presence of an unfamiliar person.

a. Incorrect See explanation for response c.
b. Incorrect See explanation for response c.
c. CORRECT Although some children experience stranger anxiety when they are as young as six or seven months of age, most children do not do so until eight to ten months of age. Therefore, of the age ranges listed, this is the best one.
d. Incorrect See explanation for response c.

The correct answer is: 8 to 10

157
Q

Research on behavioral inhibition by Kagan (1998) and others suggests that this characteristic can be exacerbated by:
Select one:

A. parental overprotectiveness.
B. parental permissiveness.
C. a lack of interactional synchrony.
D. a chaotic home environment.

A

Based on the results of his research, J. Kagan concluded that behavioral inhibition is a temperamental style that has a biological etiology and is relatively stable over the lifespan but can be modified by environmental factors [Biology and the child, in N. Eisenberg (Ed.), Handbook of childhood psychology, vol. 3: Social, emotional, and personality development (pp. 177-235), New York, Wiley, 1998].

a. CORRECT Kagan found that children who exhibited a high degree of behavioral inhibition as infants tended to be shy and fearful as toddlers and cautious and introverted when they started school. He also found that an overprotective parenting style led to increased levels of behavioral inhibition in these children, apparently because it prevented them from developing appropriate coping strategies.
b. Incorrect Parental permissiveness has not been identified as a contributor to behavioral inhibition.
c. Incorrect Interactional synchrony occurs when a caregiver coordinates his or her behavior to the behaviors of an infant and has been identified as a factor that affects the development of attachment between a caregiver and infant.
d. Incorrect A chaotic home environment has not been identified as a factor that increases a child’s level of behavioral inhibition.

The correct answer is: parental overprotectiveness.

158
Q

Vygotsky’s approach to cognitive development has had the greatest influence on which of the following?
Select one:

A. the reciprocal teaching method
B. the teaching for understanding method
C. the use of computer-adaptive instruction for children with learning disabilities
D. genetic explanations for cognitive development

A

Vygotsky’s theory of cognitive development has had the greatest impact on educational theories and strategies.

a. CORRECT Brown and Palinscar’s (1989) reciprocal teaching method emphasizes the child’s ability to learn from others and was strongly influenced by Vygotsky’s approach to cognitive development (A. L. Brown and A. S. Palinscar, Guided, cooperative learning and individual knowledge acquisition, Lawrence Erlbaum Associates, Hillsdale, NJ, 1989).
b. Incorrect See explanation for response a.
c. Incorrect See explanation for response a.
d. Incorrect See explanation for response a.

The correct answer is: the reciprocal teaching method

159
Q

Piaget’s concrete operational stage of development is characteristic of children ages:
Select one:

A. 2 to 7 years.
B. 5 to 8 years.
C. 7 to 11 years.
D. 10 to 15 years.

A

Piaget distinguishes between four stages of cognitive development–sensorimotor, preoperational, concrete operational, and formal operations.Additional information about these stages is provided in the Lifespan Development chapter of the written study materials.

a. Incorrect Children ages 2 to 7 are in the preoperational stage.
b. Incorrect This is not the age range for the concrete operational stage.
c. CORRECT This is the age range for the concrete operational stage.
d. Incorrect The formal operational stage usually begins in adolescence.

The correct answer is: 7 to 11 years.

160
Q

The positive symptoms of __________ include hand tremor that has been described as “pill-rolling” since it resembles the motion of rolling a pill between the thumb and forefinger.
Select one:

A. Parkinson’s disease
B. Huntington’s chorea
C. multiple sclerosis
D. myasthenia gravis

A

Each of the disorders listed involves abnormalities in motor functioning.

a. CORRECT Parkinson’s disease includes both positive and negative symptoms, and the symptom described in this question is one of the former. Other positive symptoms are involuntary movements and muscular rigidity; negative symptoms include postural fixation, disturbances in equilibrium and locomotion, akinesia, and speech disturbances.
b. Incorrect Huntington’s chorea involves frequent involuntary irregular movements that affect the whole limbs or parts of limbs.
c. Incorrect Multiple sclerosis is a progressive disease that is due to a loss of myelin and that begins with paresthesias, muscle weakness, and visual disturbances.
d. Incorrect Myasthenia gravis means “severe muscle weakness” and is characterized by severe muscle fatigue after muscles have been exercised a few times.

The correct answer is: Parkinson’s disease

161
Q

The physiological phenomenon referred to as long-term potentiation takes place in the:
Select one:

A. spinal cord.
B. peripheral nervous system.
C. reticular activating system.
D. hippocampus.

A

Long-term potentiation (LTP) has been suggested as one of the physiological bases of learning and memory. It involves greater responsiveness of a post-synaptic neuron to low-intensity stimulation for an extended period of time.

a. Incorrect See explanation for response d.
b. Incorrect See explanation for response d.
c. Incorrect See explanation for response d.
d. CORRECT Long-term potentiation was first observed in the hippocampus, which is known to play a key role in learning and memory, especially in the consolidation of short-term memory into long-term memory.

The correct answer is: hippocampus.

162
Q

Several subcortical structures are involved in the mediation of emotion. Studies of decorticate animals, for instance, suggest that the __________ is responsible for a violent rage response to even trivial stimuli.
Select one:

A. hypothalamus
B. medulla
C. hippocampus
D. RAS

A

The limbic system (especially the amygdala) is most associated with emotion, but other structures of the brain also play a role.

a. CORRECT The hypothalamus is always a good guess since it’s involved in so many different functions. The studies have shown that decorticate animals tend to respond to even minor stimuli with extreme rage as long as the hypothalamus is left intact.
b. Incorrect The medulla is involved in many of the body’s vital functions including breathing and heart rate.
c. Incorrect The hippocampus is part of the limbic system, but it’s involved more with memory than emotion.
d. Incorrect The reticular activating system mediates attention, arousal, and the sleep-wake cycle.

The correct answer is: hypothalamus

163
Q

Classical conditioning involves the formation of an association between a conditioned stimulus and an unconditioned stimulus. The ________ is the area of the brain that is known to be involved in forming this type of implicit memory.
Select one:

A. hippocampus
B. cerebellum
C. pons
D. medial temporal lobes

A

Implicit memory refers to memories that do not require conscious recall and involve “knowing how” - e.g., how to ride a bicycle and how to play a familiar game. Most procedural (non-declarative) memories are implicit memories.

a. Incorrect The hippocampus plays an important role in explicit memory which involves conscious recollection of information and previous experiences - i.e., in “knowing that.” Most declarative memories are explicit memories. Other areas involved in explicit memory include the temporal lobes and prefrontal cortex.
b. CORRECT Implicit memory is mediated by several structures including the cerebellum, amygdala, and basal ganglia. The cerebellum is involved in classical conditioning, the cerebellum and basal ganglia mediate motor memories, and the amygdala plays a role in implicit emotional memories.
c. Incorrect The pons is located in the brainstem. It transmits information between the spinal cord and the brain and plays a role in sleep and arousal and facial expressions.
d. Incorrect The medial temporal lobes include areas of the temporal lobe, the hippocampus, and several other structures and are involved in explicit memory.

The correct answer is: cerebellum

164
Q

When asked to name as many different animals as possible, a patient with traumatic brain injury says, “Oh, you’ve got your big dogs and your little dogs, and then there’s black dogs, and white dogs, and brown dogs, and I had a black and white dog once and he was really nice.” The patient’s response represents perseveration, which is associated with damage to which of the following?
Select one:

A. occipital lobe
B. parietal lobe
C. temporal lobe
D. frontal lobe

A

The category fluency test involves asking a patient to name as many items as possible from a particular category (e.g., animals, furniture) and is one of the tests used to evaluate frontal lobe dysfunction.

a. Incorrect See explanation for response d.
b. Incorrect See explanation for response d.
c. Incorrect See explanation for response d.
d. CORRECT This patient is exhibiting perseveration - i.e., he is repeating “dog” rather than naming different animals. Perseveration on the category fluency test is indicative of damage in the dorsolateral area of the frontal lobe. See, e.g., J. A. Bourgeois et al., Casebook of psychosomatic medicine, Arlington, VA, American Psychiatric Publishing, 2009.
The correct answer is: frontal lobe

165
Q

As a result of brain injury, Thom Tenacity often has difficulty stopping a behavior once he starts it. For example, when Thom goes to the drawer to get a fork before dinner, he often ends up taking out all of the silverware; and when he intends to take only one jacket out of his closet, he ends up removing all of his jackets. Most likely, Thom’s problem is due to damage to which of the following areas of the brain?
Select one:

A. frontal lobe
B. parietal lobe
C. temporal lobe
D. occipital lobe

A

Thom’s behavior is referred to as perseveration.

a. CORRECT Perseveration (the inability to stop a behavior once it begins) has been linked to lesions in the dorsolateral area of the frontal lobes.
b. Incorrect See explanation for response a.
c. Incorrect See explanation for response a.
d. Incorrect See explanation for response a.

The correct answer is: frontal lobe

166
Q

Fluoxetine and other SSRIs act by:
Select one:

A. increasing the release of serotonin at synapses.
B. increasing the manufacture of serotonin by nerve cells.
C. increasing the sensitivity of nerve cells to serotonin.
D. increasing the availability of serotonin at synapses.

A

As long as you know what “SSRI” stands for, you should have been able to identify the right answer to this question.

a. Incorrect See explanation for response d.
b. Incorrect See explanation for response d.
c. Incorrect See explanation for response d.
d. CORRECT SSRI stands for selective serotonin re-uptake inhibitor, which means that the SSRIs exert their effects by reducing the uptake of serotonin at nerve synapses, thereby increasing their availability.

The correct answer is: increasing the availability of serotonin at synapses.

167
Q

The prodromal symptoms of a tyramine-induced hypertensive crisis include:
Select one:

A. headache, stiff neck, nausea, vomiting, and sweating.
B. orthostatic hypotension, insomnia, and edema.
C. paresthesias, myoclonus, and muscle pain.
D. irritability, confusion, dizziness, and cardiac arrhythmia.

A

A tyramine-induced hypertensive crisis may result when a person taking an MAOI consumes food containing tyramine.

a. CORRECT These are the prodromal symptoms of a tyramine-induced hypertensive crisis. A person experiencing these symptoms should seek immediate medical attention because this condition can be life-threatening.
b. Incorrect These are common side effects of the MAOIs.
c. Incorrect These are rare side effects of the MAOIs.
d. Incorrect These are symptoms of serotonin syndrome, which may result when an MAOI is taken in conjunction with an SSRI.

The correct answer is: headache, stiff neck, nausea, vomiting, and sweating.

168
Q

Wernicke’s encephalopathy is due to:
Select one:

A. a thiamine deficiency.
B. a deficiency of Vitamin C.
C. acute head trauma.
D. anoxia.

A

The primary symptoms of Wernicke’s encephalopathy are acute mental confusion, ataxia, and opthalmoplegia (abnormal eye movements).

a. CORRECT Wernicke’s encephalopathy is caused by a deficiency of thiamine, often as the result of chronic alcoholism. Additional information about this disorder is provided in the Physiological Psychology and Psychopharmacology chapter of the written study materials.
b. Incorrect See explanation above.
c. Incorrect See explanation above.
d. Incorrect See explanation above.

The correct answer is: a thiamine deficiency.

169
Q

The effectiveness of clomipramine in alleviating symptoms of Obsessive-Compulsive Disorder and trichotillomania suggests that these disorders are due to:
Select one:

A. oversensitivity to GABA.
B. an imbalance between acetylcholine and dopamine.
C. oversensitivity to serotonin or dopamine.
D. a lower-than-normal level of serotonin.

A

Clomipramine is a tricyclic antidepressant that has been found to be an effective treatment for OCD.

a. Incorrect See explanation for response d.
b. Incorrect See explanation for response d.
c. Incorrect See explanation for response d.
d. CORRECT Clomipramine is believed to exert its effects by blocking the reuptake of serotonin from synaptic clefts.

The correct answer is: a lower-than-normal level of serotonin.

170
Q

An advantage of structural magnetic resonance imaging (MRI) over computed tomography (CT) is that the former:
Select one:

A. provides clearer images.
B. is less likely to require sedation of the patient.
C. provides information on functional brain activity.
D. can detect electrical abnormalities.

A

Neuroimaging techniques are divided into two types - structural and functional. CT and MRI are both structural techniques.

a. CORRECT MRI uses magnetic fields to produce detailed cross-sectional images of the brain. An important advantage of MRI over a CT scan is that MRI has better resolution.
b. Incorrect Because MRI requires the patient to be completely still for a prolonged period of time, the patient may need to be sedated. A CT scan requires less time and, therefore, is less likely to require sedation.
c. Incorrect Functional magnetic resonance imaging (fMRI) provides information on the functional activities of the brain, but MRI is a structural technique that provides information on the structure of the brain only.
d. Incorrect This is an advantage of EEG (electroencephalography), which is why it is used in the assessment of epilepsy.

The correct answer is: provides clearer images.

171
Q

The extrapyramidal motor symptoms associated with Parkinson’s disease are caused primarily by an imbalance between which of the following neurotransmitters?
Select one:

A. dopamine and acetylcholine
B. epinephrine and norepinephrine
C. dopamine and serotonin
D. norepinephrine and acetylcholine

A

Knowing that Parkinson’s disease has been linked to a deficiency of dopamine and that acetylcholine is involved in the control of voluntary movements would have helped you identify the correct answer to this question.

a. CORRECT Dopamine is an inhibitory neurotransmitter in the extrapyramidal motor system, while acetylcholine is an excitatory neurotransmitter. Parkinson’s disease is caused by degeneration of dopaminergic neurons in the substantia nigra (which is part of the extrapyramidal motor system) which disrupts the balance between dopamine and acetylcholine and produces the tremors, rigidity, bradykinesia, and postural changes associated with this disorder.
b. Incorrect See explanation for answer a.
c. Incorrect See explanation for answer a.
d. Incorrect See explanation for answer a.

The correct answer is: dopamine and acetylcholine

172
Q

Recent research using brain imaging techniques, has found that significant atrophy in the __________ distinguishes people with Alzheimer’s dementia from healthy peers.
Select one:

A. vestibular nucleus
B. hypothalamus
C. entorhinal cortex
D. somatosensory cortex

A

Although pathology in several areas of the brain has been linked to Alzheimer’s Dementia, recent research has found that one particular area most clearly distinguishes between individuals with and without the disorder.

a. Incorrect See explanation for response c.
b. Incorrect See explanation for response c.
c. CORRECT MRIs have revealed that individuals in the early stages of Alzheimer’s Dementia have a reduction in the volume of the entorhinal cortex of up to 30% when compared to healthy peers and that those in the later stages show a reduction of up to 45%. See, e.g., M. Bobinski et al., MRI of entorhinal cortex in mild Alzheimer’s disease, Lancet, 353, 38-40, 1999.
d. Incorrect See explanation for response c.

The correct answer is: entorhinal cortex

173
Q

The frontal lobes mediate executive functions as well as complex emotions and behaviors. A person with damage to which area of the frontal lobes is most likely to engage in disinhibited, impulsive behaviors and exhibit emotional lability, distractibility, and poor judgment and insight?
Select one:

A. medial frontal area
B. anterior cingulate area
C. dorsolateral prefrontal area
D. orbitofrontal area

A

Research using neuroimaging techniques has pinpointed the location of specific emotional and behavior changes caused by frontal lobe damage.

a. Incorrect Damage to the medial frontal area produces akinesia, mutism, and weakness and loss of sensation in the lower extremities.
b. Incorrect Damage to the anterior cingulate area is associated with apathy and paucity of speech and movement.
c. Incorrect Damage to the dorsolateral prefrontal area produces impaired executive functioning, slowed information processing, and mood and personality changes.
d. CORRECT Damage to the orbitofrontal areas produces pseudopsychopathy (disinhibition and impulsive behavior), euphoria, lack of judgment and social tact, and distractibility.

The correct answer is: orbitofrontal area

174
Q

Four major dopamine pathways in the brain have been identified. Of these, excessive dopamine in the __________ pathway has been linked to the positive symptoms of Schizophrenia.
Select one:

A. mesolimbic
B. mesocortical
C. nigrostriatal
D. tuberoinfundibular

A

The four dopamine pathways listed in the answers to this question have been linked to different responses.

a. CORRECT The mesolimbic pathway plays a role in motivation and emotion, and excessive mesolimbic dopamine has been linked to the positive symptoms of Schizophrenia (hallucinations and delusions). It is believed that the typical antipsychotics exert their therapeutic effects by blocking dopamine receptors in the mesolimbic pathway. See, e.g., J. E. Kelsey, D. J. Newport, and C. B. Nemeroff, Principles of pharmacology for mental health professionals, Hoboken, NJ, John Wiley & Sons, 2006.
b. Incorrect The mesocortical pathway plays a role in emotion and cognition, and there is evidence that a low level of mesocortical dopamine contributes to some negative and cognitive symptoms of Schizophrenia.
c. Incorrect The nigrostriatal pathway is involved in motor control, and abnormalities in this pathway have been linked to Parkinson’s disease.
d. Incorrect The tuberoinfundibular pathway connects the hypothalamus to the pituitary gland and is involved in the secretion of prolactin and other hormones.
The correct answer is: mesolimbic

175
Q

Recent research indicates that people with Parkinson’s disease have lost at least 80% of __________-producing cells in the substantia nigra.
Select one:

A. norepinephrine
B. dopamine
C. serotonin
D. substance P

A

Knowing that dopamine is involved in the control of voluntary motor movements would have helped you select the correct response to this question.

a. Incorrect See explanation for response b.
b. CORRECT Degeneration of neurons that secrete dopamine contributes to movement disorders such as Parkinson’s disease and Huntington’s chorea.
c. Incorrect See explanation for response b.
d. Incorrect See explanation for response b.

The correct answer is: dopamine

176
Q

Sexual dimorphism refers to:
Select one:

A. the relative status of people based on their gender.
B. differences in structure between males and females of the same species.
C. having both male and female sexual characteristics.
D. being sexually attracted to both males and females.

A

The term sexual dimorphism is used to describe differences in structure in males and females.

a. Incorrect See explanation for response b.
b. CORRECT Gender differences in height, weight, and muscle strength are examples of sexual dimorphism. Additional information on sexual dimorphism is provided in the Physiological Psychology and Psychopharmacology chapter of the written study materials.
c. Incorrect See explanation for response b.
d. Incorrect See explanation for response b.

The correct answer is: differences in structure between males and females of the same species.

177
Q

Damage to which of the following areas of the brain is most likely to cause deficits in prospective memory?
Select one:

A. prefrontal region
B. occipito-temporal region
C. corpus callosum
D. parietal association cortex

A

Prospective memory refers to the ability to remember to perform an intended action at a particular point of time in the future - e.g., to give your colleague a telephone message the next time you see her.

a. CORRECT Prospective memory has been linked to the prefrontal lobes, and it has been hypothesized that executive cognitive functions (which have also been linked to this area of the brain) are involved in this aspect of memory. See, e.g., M. McDaniel et al., Prospective memory: A neuropsychological study, Neuropsychology, 13(1), 103-110, 1999.
b. Incorrect Although there is some evidence that the medial temporal region may be involved in prospective memory, the occipito-temporal area has not been implicated.
c. Incorrect The corpus callosum has not been linked to prospective memory.
d. Incorrect The parietal association cortex has not been linked to prospective memory.

The correct answer is: prefrontal region

178
Q

The assumption that changes in regional cerebral blood flow correspond to changes in neural activity underlies the use of all of the following brain imaging techniques except:
Select one:

A. PET.
B. fMRI.
C. CT.
D. SPECT.

A

Regional cerebral blood flow is associated with the brain activity (functioning), and all but one of the brain imaging techniques listed in the responses are functional techniques.

a. Incorrect See explanation for response c.
b. Incorrect See explanation for response c.
c. CORRECT CT (computerized tomography) is a structural brain imaging technique. It provides information on brain structure (e.g., tumors, lesions), not function.
d. Incorrect See explanation for response c.

The correct answer is: CT.

179
Q

Neurons in the ____________ respond to the taste, smell, and sight of food.
Select one:

A. hippocampus
B. reticular activating system
C. suprachiasmatic nucleus
D. orbitofrontal cortex

A

This is a difficult question, but knowing that the orbitofrontal cortex is involved in the conscious perception of odors (as noted in the Physiological Psychology chapter of the study materials) would have helped you identify the correct answer.

a. Incorrect The hippocampus is involved in the consolidation of long-term declarative memories.
b. Incorrect The reticular activating system is a network of nerve fibers that runs from the brain stem to the thalamus and is involved in wakefulness, arousal, and consciousness.
c. Incorrect The suprachiasmatic nucleus controls the body’s circadian rhythms.
d. CORRECT The orbitofrontal cortex is involved in several functions including evaluating the pleasurableness (reward value) of food in terms of its sensory features - i.e., its taste, smell, and sight.

The correct answer is: orbitofrontal cortex

180
Q

There is now evidence that nicotine enhances alertness and memory by mimicking __________ at nicotinic receptor sites.
Select one:

A. endorphins
B. GABA
C. serotonin
D. acetylcholine

A

Knowing that nicotinic receptors are a type of acetylcholine receptor would have helped you choose the correct response to this question.

a. Incorrect See explanation for response d.
b. Incorrect See explanation for response d.
c. Incorrect See explanation for response d.
d. CORRECT There is evidence that the nicotine in tobacco exerts its effects on physiology and behavior by binding to nicotinic receptors in the brain.

The correct answer is: acetylcholine

181
Q

Haloperidol and other conventional neuroleptic drugs are:
Select one:

A. most effective for alleviating the positive symptoms of Schizophrenia.
B. most effective for alleviating the negative symptoms of Schizophrenia.
C. equally effective for alleviating the positive and negative symptoms of Schizophrenia.
D. more effective for alleviating the ancillary symptoms of Schizophrenia than either the positive or negative symptoms.

A

The conventional antipsychotics have been found to be most effective for the positive symptoms of Schizophrenia.

a. CORRECT Positive symptoms include delusions, hallucinations, disorganized speech, and grossly disorganized or catatonic behavior.
b. Incorrect See explanation above.
c. Incorrect See explanation above.
d. Incorrect See explanation above.

The correct answer is: most effective for alleviating the positive symptoms of Schizophrenia.

182
Q

Tom J. was involved in a car accident in which he received extensive head trauma. Since the accident, Tom’s movements are jerky and uncoordinated, and he is unable to walk across the room unaided. Tom most likely suffered damage to the:
Select one:

A. medulla.
B. cerebellum.
C. cingulate cortex.
D. somatosensory cortex.

A

To answer this question, you need to know which of the brain areas listed is involved in the control of motor movement and balance. Information about these areas is provided in the Physiological Psychology and Psychopharmacology chapter of the written study materials.

a. Incorrect See explanation for response b.
b. CORRECT The cerebellum is located at the rear of the brain stem and is involved in the coordination of voluntary movements of the skeletal muscles and the regulation of balance.
c. Incorrect See explanation for response b.
d. Incorrect See explanation for response b.

The correct answer is: cerebellum.

183
Q
Tacrine hydrocholoride (Cognex), donepezil hydrochloride (Aricept), and rivastigmine (Exelon) slow down memory loss associated with Alzheimer's disease by:
Select one:

A. inhibiting the breakdown of serotonin.
B. increasing the sensitivity of dopamine receptors.
C. preventing the breakdown of acetylcholine.
D. blocking glutamate receptors.

A

The drugs listed in this question are cholinesterase inhibitors that are used to slow down memory loss in individuals with mild to moderate Alzheimer’s disease.

a. Incorrect See explanation for response c.
b. Incorrect See explanation for response c.
c. CORRECT Knowing that acetylcholine plays an important role in memory would have helped you identify this as the correct answer to this question.
d. Incorrect Memantine (Namenda) is also used to slow down memory loss in patients with Alzheimer’s disease but, unlike the three drugs listed in this question, exerts its effects by blocking glutamate receptors.
The correct answer is: preventing the breakdown of acetylcholine.

184
Q

Approximately _____ percent of left-handed people are left-hemisphere dominant for language.
Select one:

A. 5 to 10
B. 20 to 30
C. 50 to 60
D. 90 to 95

A

The majority of people are left-hemisphere dominant for language, while the rest are right-hemisphere dominant or have mixed dominance.

a. Incorrect See explanation for response c.
b. Incorrect See explanation for response c.
c. CORRECT The studies have not produced entirely consistent results but, overall, indicate that about 95 to 99% of right-handed people and 50 to 60% of left-handed people are left-hemisphere dominant for language. See, e.g., H. S. Kirshner, Handbook of neurological speech and language disorders, New York, Informa Healthcare, 1995.
d. Incorrect See explanation for response c.

The correct answer is: 50 to 60

185
Q

The correlation for IQ scores for biological siblings reared together is:
Select one:

A. .85.
B. .65.
C. .45.
D. .25.

A

Correlations between the IQ scores of individuals with varying levels of genetic similarity have confirmed a genetic contribution to IQ (e.g., Bouchard & McGue, 1981).

a. Incorrect The correlation for identical twins reared together is .85.
b. Incorrect The correlation for identical twins reared apart is .65.
c. CORRECT The correlation for biological siblings reared together is .45.
d. Incorrect A correlation of .25 is slightly higher than the correlation for biological siblings reared apart.

The correct answer is: .45.

186
Q
The MMSE (Mini Mental State Exam) is most often administered to:
Select one:

A. children as a measure of intelligence.
B. adolescents and adults as a measure of brain dysfunction.
C. older adults as a measure of cognitive functioning.
D. older adults as a measure of quality of life.

A

The MMSE was originally developed by Folstein in 1975 as a “practical method for grading the cognitive state.”

a. Incorrect See explanation for response c.
b. Incorrect See explanation for response c.
c. CORRECT The MMSE measures several aspects of cognitive functioning. The score is derived by considering both age and level of education, with a score of 24 or less (out of 30) suggesting cognitive impairment.
d. Incorrect See explanation for response c.

The correct answer is: older adults as a measure of cognitive functioning.

187
Q

On the MMPI-2, an attempt to “fake bad” is suggested when an examinee obtains:
Select one:

A. a very high F or K scale score.
B. a very high F scale score or a very low K scale score.
C. a very low F scale score or a very high K scale score.
D. a very low F or K scale score.

A

Faking bad refers to an attempt to exaggerate one’s symptoms and problems.

a. Incorrect See explanation for response b.
b. CORRECT The F Scale measures the extent to which an examinee answers in an atypical or deviant manner, and a very high score suggests random responding, significant pathology, or faking bad. The K Scale measures the degree to which an examinee describes him/herself in overly positive terms, and a low score may indicate lack of insight, disorientation, or faking bad.
c. Incorrect See explanation for response b.
d. Incorrect See explanation for response b.

The correct answer is: a very high F scale score or a very low K scale score.

188
Q

An examinee’s raw score on the MMPI-2’s F-Scale is 18 points higher than his raw score on the K-Scale. This suggests that the examinee:
Select one:

A. answered items randomly.
B. answered half the items true and half the items false.
C. has attempted to fake bad.
D. has attempted to fake good.

A

The MMPI-2 includes several validity scales and indices that are used to determine the validity of an examinee’s scores.

a. Incorrect See explanation for response c.
b. Incorrect See explanation for response c.
c. CORRECT The F-K Index is used to determine if an examinee has attempted to “fake bad” (exaggerate problems) or to “fake good” (deny problems). It is calculated by subtracting the examinee’s raw K-Scale score from his/her raw F-Scale score. When the result is positive and greater than 12, the examinee may have attempted to “fake bad.” When the result is negative, this suggests that the examinee has attempted to “fake good.” See, e.g., R. P. Granacher, Traumatic brain injury: Methods for clinical and neuropsychiatric assessment, Boca Raton, FL, CRC Press, 2008.
d. Incorrect See explanation for response c.

The correct answer is: has attempted to fake bad.

189
Q

Children with Attention-Deficit/Hyperactivity Disorder would be expected to obtain the lowest score on which of the following WISC-V indexes?
Select one:

A. Verbal Comprehension
B. Processing Speed
C. Working Memory
D. Visual Spatial

A

This is a difficult question, but knowing that individuals with ADHD tend to obtain low scores on tests involving processing speed would have helped you identify the correct answer to this question.

a. Incorrect See explanation for response b.

b. CORRECT Children with ADHD in the standardization sample for the WISC-V obtained the highest mean score on the Verbal Comprehension Index followed by, in order, the 
Visual Spatial
Fluid Reasoning
Working Memory 
Processing Speed Indexes 

(D. Wechsler, WISC-V: Technical and interpretive manual supplement, Bloomington, MN, PsychCorp, 2014).

c. Incorrect See explanation for response b.
d. Incorrect See explanation for response b.

The correct answer is: Processing Speed

190
Q

The Woodcock-Johnson III Tests of Cognitive Abilities WJ III COG) is based on:
Select one:

A. Spearman’s notion of general intelligence.
B. Luria’s distinction between simultaneous and sequential processing.
C. the Cattell-Horn-Carroll theory of cognitive abilities.
D. Sternberg’s model of “successful intelligence.”

A

The Woodcock-Johnson III consists of two co-normed batteries – the WJ III Tests of Cognitive Abilities and the WJ III Tests of Achievement.

a. Incorrect See explanation for response c.
b. Incorrect See explanation for response c.
c. CORRECT Both batteries are based on the Cattell-Horn-Carroll (CHC) theory of cognitive abilities. See the Psychological Assessment chapter for additional information on this theory.
d. Incorrect See explanation for response c.

The correct answer is: the Cattell-Horn-Carroll theory of cognitive abilities.

191
Q

Worse than chance responding on a forced-choice recognition memory test is used to support a diagnosis of:
Select one:

A. Conversion Disorder.
B. Malingering.
C. Dementia due to Head Trauma.
D. Dyslexia (Reading Disorder).

A

Forced-choice recognition tests include the Portland Digit Recognition Test, Word Memory Test, and the Hisock and Hisock procedure.

a. Incorrect See explanation for response b.
b. CORRECT The assumption underlying the use of this method to detect Malingering is that if an examinee responds at a level worse than chance, this suggests that he/she has deliberately chosen incorrect answers.
c. Incorrect See explanation for response b.
d. Incorrect See explanation for response b.

The correct answer is: Malingering.

192
Q

Overall, the best conclusion that can be drawn about clinical and actuarial predictions is that:
Select one:

A. clinical predictions are about equally as accurate as actuarial predictions.
B. clinical predictions are usually more accurate than actuarial predictions.
C. clinical predictions are as accurate as actuarial predictions only when the clinician has adequate experience and training.
D. actuarial predictions are often more accurate than clinical predictions.

A

Clinical predictions are based on the judgment of the decision-maker, while actuarial predictions rely on statistical techniques (e.g., a multiple regression equation).

a. Incorrect See explanation for response d.
b. Incorrect There is evidence that level of experience or training does not substantially affect the accuracy of clinical predictions.
c. Incorrect See explanation for response d.
d. CORRECT Most of the research has found that actuarial predictions are at least as good as – and often better than – clinical predictions and that this is true even when both types of predictions are based on identical data.

The correct answer is: actuarial predictions are often more accurate than clinical predictions.

193
Q

Most interpretations of performance on the Bender-Gestalt are directed toward:
Select one:

A. screening for brain damage.
B. assessing personality.
C. evaluating executive functioning.
D. assigning a psychiatric diagnosis.

A

The Bender Visual-Motor Gestalt Test is a brief assessment of visual-motor integration that may also provide interpretive information about an individual’s development and neuropsychological functioning.

a. CORRECT The Bender-Gestalt is considered to be a useful screener for neuropsychological impairment. Additional information about the Bender-Gestalt is provided in the Psychological Assessment chapter of the written study materials.
b. Incorrect See explanation above.
c. Incorrect See explanation above.
d. Incorrect See explanation above.

The correct answer is: screening for brain damage.

194
Q

Research has confirmed that, when making attributions about our own behavior, we ordinarily attribute behaviors that have positive outcomes to dispositional factors but behaviors that have negative outcomes to situational factors. This tendency is referred to as the:
Select one:

A. self-verification effect.
B. self-serving bias.
C. confirmation bias.
D. self-fulfilling prophecy.

A

For the exam, you want to be familiar with all of the phenomena listed in the answers to this question. Additional information on them is provided in the Social Psychology chapter of the written study materials.

a. Incorrect According to self-verification theory, people are motivated to maintain a stable self-concept and do so by seeking information that confirms the self-concept, even when that information is negative.
b. CORRECT As its name implies, the self-serving bias is the tendency to take responsibility for our actions when the actions have positive outcomes (i.e., to make dispositional attributions) but to blame external events for our actions when they have negative outcomes (i.e., to make situational attributions).
c. Incorrect The confirmation bias is the tendency to pay attention only to information that confirms our current beliefs.
d. Incorrect The self-fulfilling prophecy effect occurs when expectations about our own behavior or the behavior of others increases the likelihood that the behavior will actually occur.

The correct answer is: self-serving bias.

195
Q

Research by the French social psychologist Serge Moscovici (1985) suggests that, if you represent the minority position on an issue and want to convince other group members that your position is best, the best strategy is to:
Select one:

A. ingratiate yourself to the most influential member of the group.
B. let the majority know that you will support their views on other key issues.
C. not waiver from your position.
D. show willingness to compromise.

A

S. Moscovici (1985) proposed that, to be influential, a minority must become visible to other group members and create tension or conflict that causes members of the majority to reconsider their position.

a. Incorrect See explanation for response c.
b. Incorrect See explanation for response c.
c. CORRECT Moscovici’s research found that a minority that sticks to his/her position is most successful in swaying the opinion of the majority.
d. Incorrect See explanation for response c.

The correct answer is: not waiver from your position.

196
Q

The tendency to seek or pay attention only to information that confirms our beliefs is referred to as:
Select one:

A. functional fixedness.
B. belief perseverance.
C. the self-serving bias.
D. the confirmation bias.

A

Research has identified a number of biases that impact our judgments and decisions.

a. Incorrect Functional fixedness refers to the tendency to think of objects only in terms of their usual functions.
b. Incorrect Belief perseverance is the tendency to continue adhering to a belief even when the belief has been discredited.
c. Incorrect The self-serving bias is the tendency to make attributions about oneself that maintain one’s self-esteem - e.g., to attribute positive outcomes to dispositional factors but negative outcomes to situational factors.
d. CORRECT The confirmation bias occurs when we commit ourselves to one explanation or hypothesis about a phenomenon without adequately considering or testing other possibilities.

The correct answer is: the confirmation bias.

197
Q

Elaine considers her friend, Tom, to be quiet and reserved. She says that when she is alone with Tom or when they are in a small group, Tom listens more than he talks and is never one to “make a scene.” One day, however, Elaine observes Tom in a large crowd that is waiting for the arrival of a “special celebrity guest” at the opening of a new shopping mall. The guest is nearly 30 minutes late, and the crowd is beginning to show signs of impatience. Elaine is surprised when she sees Tom initiate loud “booing” and other derogatory comments while waving his clenched fist in the air in anger. Which of the following best explains Tom’s unusual behavior?
Select one:

A. deindividuation
B. social facilitation
C. catharsis
D. paradoxical intention

A

A number of studies have shown that anonymity increases the likelihood that an individual will engage in antisocial or other uncharacteristic behaviors.

a. CORRECT The increased tendency to act in uncharacteristic ways when anonymity is likely is referred to as “deindividuation” (Zimbardo, 1970). Deindividuation is believed to be the result of a decreased sense of responsibility, reduced self-consciousness, a lowered fear of evaluation, and/or a loss of other inhibitory mechanisms.
b. Incorrect Social facilitation refers to the tendency toward improved task performance in the presence of others.
c. Incorrect Catharsis is the therapeutic release of tension, anxiety, etc.
d. Incorrect Paradoxical intention is a behavioral technique and is not related to this situation.

The correct answer is: deindividuation

198
Q

Laboratory and dormitory studies have consistently shown that males and females react differently to crowding. Specifically, laboratory studies suggest that crowding is more detrimental for males, and dormitory studies indicate that:
Select one:

A. this gender difference is even more pronounced in real-life settings.
B. males utilize less effective coping strategies than females in crowded situations.
C. females utilize less effective coping strategies than males in crowded situations.
D. the gender difference occurs but is less pronounced in real-life settings.

A

Inconsistencies in the research on crowding seem to be due, in part, to the type of research (e.g., laboratory versus dormitory).

a. Incorrect See explanation for response c.
b. Incorrect See explanation for response c.
c. CORRECT For example, J. R. Aiello et al. (Field experiment research on human crowding, Paper presented at the annual meeting of the Eastern Psychological Association, New York, 1975) report that women in their study exhibited more signs of psychological distress in crowded dormitory rooms than men did, apparently because women rely on less effective coping mechanisms - i.e., men tend to leave while women increase their levels of interaction. (In laboratory studies, leaving the crowded situation was not an option.)
d. Incorrect See explanation for response c.

The correct answer is: females utilize less effective coping strategies than males in crowded situations.

199
Q

A depressive attributional style is characterized by attributions that emphasize:
Select one:

A. external, stable, and global factors.
B. external, unstable, and specific factors.
C. internal, stable, and global factors.
D. internal, unstable, and specific factors.

A

Attribution theorists distinguish between three dimensions: internal vs. external; global vs. specific; and stable vs. unstable.

a. Incorrect See explanation for response c.
b. Incorrect See explanation for response c.
c. CORRECT From the perspective of attribution theory, attributional style is a personality trait that mediates between negative events and depression. Depression results when negative events are attributed to internal, stable, and global factors.
d. Incorrect See explanation for response c.

The correct answer is: internal, stable, and global factors.

200
Q

According to Ajzen’s (1991) theory of planned behavior, a person’s ____________ is directly influenced by three factors - attitude toward the behavior, subjective norms, and perceived behavioral control.
Select one:

A. behavioral consistency
B. behavioral intention
C. locus of control
D. self-efficacy

A

I. Ajzen’s (1991) theory of planned behavior provides a model for understanding the relationship between attitudes and behavior. It predicts that attitudes alone do not determine behavior but, instead, interact with subjective norms and perceived behavioral control.

a. Incorrect See explanation for response b.
b. CORRECT Ajzen’s theory of planned behavior is an extension of the earlier theory of reasoned action. It describes the three factors listed in this question as determinants of a person’s intention to perform a behavior which, in turn, determines the person’s actual behavior. The stronger the behavioral intention, the more likely the person will perform the behavior.
c. Incorrect See explanation for response b.
d. Incorrect See explanation for response b.

The correct answer is: behavioral intention

201
Q

According to the Elaboration Likelihood model:
Select one:

A. persuasion via the peripheral route is more likely when the listener is uninvolved with the message and the communicator is appealing.
B. persuasion via the peripheral route is more likely when multiple sources present the persuasive message.
C. a minority group has the greatest persuasive influence when members have high status, are close to the majority in space and time, and are large in number.
D. a minority group has the greatest persuasive influence when it is consistent, somewhat rigid, but fair.

A

The Elaboration Likelihood Model proposes two routes of communication: a central route and a peripheral route.

a. CORRECT When a listener is uninvolved with the message or is distracted, the communicator is appealing, and/or the message appeals to fear, the listener is more susceptible to persuasion via the peripheral route.
b. Incorrect With multiple sources, the central route is the more likely channel of communication. (The central route is governed by the consistency and quality of the persuasive argument.)
c. Incorrect This is correct but doesn’t apply to the Elaboration Likelihood Model.
d. Incorrect This is also true but doesn’t have anything to do with the Elaboration Likelihood Model.

The correct answer is: persuasion via the peripheral route is more likely when the listener is uninvolved with the message and the communicator is appealing.

202
Q

Studies investigating the impact of sexually explicit movies have found all of the following to be true except:
Select one:

A. men who view X-rated films tend to rate their own sexual partner as more attractive.
B. men who view X-rated films that depict a man sexually overpowering a woman report being a little more likely to rape if they were assured they would get away with it.
C. after viewing X-rated films, men tend to recommend more lenient prison sentences for a convicted rapist.
D. after viewing X-rated films, men are more likely to view a woman’s friendliness as sexual in nature.

A

Research has shown that exposure to sexually explicit material - especially material that depicts violence against or domination over women - can have a negative impact on males.

a. CORRECT This is the opposite of what has been found - i.e., viewers tend to rate their partners as less attractive. See R. J. Harris, The impact of sexually explicit media, in J. Brant and D. Zillman (Eds.), Media effects: Advances in theory and research, Hillsdale, NJ, Erlbaum, 1994.
b. Incorrect This has been found to be true by the research.
c. Incorrect This is also reported in the research.
d. Incorrect Studies have found this to be one of the consequences of viewing sexually-explicit material.

The correct answer is: men who view X-rated films tend to rate their own sexual partner as more attractive.

203
Q

After being rejected by the college she most wanted to attend, a young woman decides that she didn’t really want to go to that college after all. This girl’s reaction is predicted by which of the following:
Select one:

A. equity theory.
B. fundamental attribution bias.
C. cognitive dissonance theory.
D. gain/loss theory.

A

In this situation, the young woman has changed her mind about the college in response to being rejected.

a. Incorrect Equity theory proposes that we compare our input/outcome ratio to that of others and the results of this comparison determine our level of motivation.
b. Incorrect The fundamental attribution bias addresses the tendency to attribute an actor’s behavior to dispositional factors.
c. CORRECT Cognitive dissonance predicts that, in this situation, the woman will change her attitude about the school in order to relieve the tension she feels as a result of not being accepted by the school.
d. Incorrect Gain/loss theory predicts that we tend to be more attracted to people when their response to us is first negative, then positive, than when it is consistently positive.

The correct answer is: cognitive dissonance theory.

204
Q

LISREL is used to analyze:
Select one:

A. unidirectional and bidirectional causal relationships between measured and latent variables.
B. unidirectional causal relationships between observed variables only.
C. orthogonal and oblique relationships between two or more variables or sets of variables.
D. orthogonal relationships between two or more variables or sets of variables only.

A

LISREL, or linear structural relations analysis, is a complex multivariate technique used to test the veracity of hypotheses about the causal relationships among a set of variables that are believed to measure or reflect latent traits.

a. CORRECT The LISREL model assumes (1) that there are both unidirectional and bidirectional relationships between latent traits and (2) that measured variables reflect both latent traits and the effects of measurement error.
b. Incorrect This better describes path analysis, which is another type of structural analysis that is used to test causal hypotheses. Additional information about LISREL and path analysis is provided in the Statistics and Research Design chapter of the written study materials.
c. Incorrect See explanation for response a.
d. Incorrect See explanation for response a.

The correct answer is: unidirectional and bidirectional causal relationships between measured and latent variables.

205
Q

To determine the degree of association between two variables that are reported in terms of ranks, you would use which of the following correlation coefficients?
Select one:

A. contingency
B. biserial
C. phi
D. Spearman

A

For the exam, you want to be familiar with the correlation coefficients listed in the answers to this question.

a. Incorrect The contingency coefficient is used when both variables are measured on a nominal scale.
b. Incorrect The biserial coefficient is the appropriate correlation coefficient when one variable is continuous and the other is an artificial dichotomy.
c. Incorrect The phi coefficient is used when both variables are true dichotomies.
d. CORRECT The Spearman rank-order correlation coefficient is also known as Spearman rho and is used when both variables are ranks.

The correct answer is: Spearman

206
Q

A researcher would use stepwise multiple regression when she wants to:
Select one:

A. identify the fewest number of predictors needed to account for criterion variability.
B. identify the fewest number of criteria needed to accurately predict performance.
C. statistically remove the effects of one or more moderator variables.
D. identify the optimal number of distinct criterion groups.

A

Stepwise multiple regression is a type of multiple regression.

a. CORRECT The results of a stepwise multiple regression analysis indicate the fewest number of predictors needed to obtain maximally accurate predictions. It involves either adding or subtracting one predictor at a time and calculating the multiple correlation coefficient to determine the effects of having more or less predictors.
b. Incorrect See explanation for response a.
c. Incorrect See explanation for response a.
d. Incorrect See explanation for response a.

The correct answer is: identify the fewest number of predictors needed to account for criterion variability.

207
Q

Which of the following measures of “effect size” (the magnitude of a treatment effect) indicates the amount of variability in an outcome measure accounted for by the effects of the treatment?
Select one:

A. eta squared
B. Cohen’s d
C. alpha
D. F-ratio

A

Even if you are unfamiliar with the various methods used to measure effect size, you may have been able to identify the correct answer to this question as long as you know that “amount of variability accounted for” is assessed by a squared correlation coefficient.

a. CORRECT Eta squared is the square of the correlation coefficient (i.e., the correlation between the treatment and the outcome) and is used as an index of effect size.
b. Incorrect Cohen’s d is also used as an index of effect size, but it is a measure of the mean difference between two groups; not a measure of “amount of variability accounted for.” Additional information about eta squared and Cohen’s d is provided in the Statistics and Research Design chapter of the written study materials.
c. Incorrect Alpha is the level of significance set by a researcher prior to analyzing his/her data.
d. Incorrect The F-ratio is the statistic calculated when using the analysis of variance.

The correct answer is: eta squared

208
Q

During the course of data analysis, a researcher more often double-checks results that seem to conflict with her hypothesis than results that confirm it. This is an example of:
Select one:

A. the experimenter expectancy effect.
B. demand characteristics.
C. the Pygmalion effect.
D. the correspondence bias.

A

In this situation, the experimenter’s behavior might have a biasing effect on the study’s results in the direction of her research hypothesis.

a. CORRECT Experimenter expectancy (bias) occurs when the experimenter’s behavior biases the research results in some (usually unconscious) way so that the results are consistent with the research hypothesis.
b. Incorrect Demand characteristics are cues in the research situation that communicate to subjects what behaviors are expected of them. Experimenter expectancies can act as a source of demand characteristics (although that wouldn’t be the case in this situation).
c. Incorrect The Pygmalion effect (aka the self-fulfilling or Rosenthal effect) occurs when a person’s expectations about another individual actually produce subtle changes in the individual’s behavior so that the behavior conforms to the person’s expectations.
d. Incorrect Correspondence bias is another name for the fundamental attribution bias, which is the tendency for observers to attribute another person’s behavior to dispositional (rather than situational) factors.

The correct answer is: the experimenter expectancy effect.

209
Q

If scores on two tests are correlated, multiplying each score in one set of test scores by five will ___________ the magnitude of the Pearson r.
Select one:

A. increase
B. decrease
C. have no effect on
D. increase or decrease

A

The Pearson r is measure of the degree of association between variables or, put another way, of the extent to which a score on one variable can accurately predict a score on another variable.

a. Incorrect See explanation for response c.
b. Incorrect See explanation for response c.
c. CORRECT This response makes intuitive sense. For instance, if SAT scores are highly predictive of GRE scores, then multiplying all SAT scores (or all GRE scores or all SAT and GRE scores) by a constant will not change this relationship. You still will be able to use SAT scores to predict GRE scores with the same degree of accuracy.
d. Incorrect See explanation for response c.

The correct answer is: have no effect on

210
Q

When evaluating a treatment for children with serious behavioral disorders, to overcome the ethical problems associated with the use of a wait-list control group, the best research design would be which of the following?
Select one:

A. single-subject
B. cross-sectional
C. placebo control
D. clinical trials

A

The ethical problem referred to in this question is the withholding of treatment to control group subjects for the duration of the research study.

a. CORRECT In single-subject designs, subjects act as their own controls. An ideal single-subject design in this situation would be the AB or the multiple-baseline design. These designs do not require removal of a treatment during the course of the study.
b. Incorrect Cross-sectional designs are used to assess the effects of an independent variable across different age groups.
c. Incorrect A placebo control group requires that some subjects receive no treatment (the placebo) during the course of the study and would not overcome the ethical problem associated with a wait-list control group.
d. Incorrect “Clinical trials” refers to research conducted in clinical settings and may involve a wait-list or placebo control group.

The correct answer is: single-subject

211
Q

A psychologist believes that the benefits of a cognitive-behavioral intervention for eating disorders is due to the intervention’s impact on the individual’s self-efficacy beliefs. In other words, the psychologist believes that, in this situation, self-efficacy beliefs are a:
Select one:

A. moderator variable.
B. mediator variable.
C. confounding variable.
D. suppressor variable.

A

Answer B is correct: The psychologist believes that self-efficacy beliefs mediate (are responsible for) the impact of the intervention on eating behaviors. A mediator variable accounts for the relationship between an IV and a DV. In other words, “A” (the IV) causes “B” (the mediator), which then causes “C” (the DV).

Answer A: A moderator variable is a variable that affects the strength of the relationship between independent variable (IV) and dependent variable (DV) but does NOT explain how the IV affects the DV. For example, if an intervention has beneficial effects for males but not for females, gender is a moderator variable.

Answer C: A confounding variable is a variable that makes it appear that there is a strong relationship between variables when there is actually a weak or no relationship. For example, if a researcher wants to evaluate the effects of lighting on reaction time and she does so by having the same subjects work in a well-lit room in the morning and a dimly lit room in the afternoon, fatigue may be a confounding variable. In this situation, any observed increase in reaction time in the dimly lit room might be due to increased fatigue that occurred as the result of the passage of time rather than to the change in lighting.

Answer D: A suppressor variable reduces or conceals the relationship between two variables. Consequently, statistically removing the effects of the suppressor variable increases the correlation between the two variables.

The correct answer is: mediator variable.

212
Q

Ex post facto research is distinguished from true experimental research by:
Select one:

A. an inability to randomly select participants from the population.
B. an inability to manipulate the independent variable(s).
C. the observation of participants in their natural environment.
D. the study of participants under conditions that resemble (but are not identical to) the target conditions.

A

Answer B is correct: Ex post facto is “after the fact” research. A study is considered to be ex post facto when the IV cannot be manipulated because it has already been applied. Note that some experts categorize ex post facto research as “quasi-experimental,” while others place it in a separate category (e.g., descriptive research).

The correct answer is: an inability to manipulate the independent variable(s).

213
Q

In a research study investigating the possible genetic transmission of ADHD, the “proband” is:
Select one:

A. the individual who has received a diagnosis of ADHD.
B. a biological parent of the individual who has received a diagnosis of ADHD.
C. a non-affected biological sibling of the individual who has received a diagnosis of ADHD.
D. a member of the control group who has been matched to the individual who has received a diagnosis of ADHD.

A

The proband in research study is also known as the index case.

a. CORRECT The affected individual who is of immediate concern or who was initially identified because of his/her disorder is referred to as the proband.
b. Incorrect See explanation for response a.
c. Incorrect See explanation for response a.
d. Incorrect See explanation for response a.

The correct answer is: the individual who has received a diagnosis of ADHD.

214
Q

When conducting a research study to test the effects of two weight reduction programs on weight loss, you decide to match participants in terms of age, weight, and socioeconomic status before you assign them to groups. By doing this, you are:
Select one:

A. trying to control systematic variance.
B. trying to increase experimental variance.
C. trying to decrease random error.
D. introducing a systematic bias.

A

By matching participants on extraneous variables, you are controlling their effects on the dependent variable by ensuring that the extraneous variables are equally represented in all groups.

a. CORRECT Matching is a method for controlling the effects of variables that are not of interest to the researcher but that are believed to have some systematic effect on the dependent variable. By matching participants on extraneous variables and then randomly assigning participants in matched pairs (or triads, etc.), a researcher can ensure that the groups are equal in terms of the extraneous variables.
b. Incorrect Experimental variance is increased by ensuring that groups are as different as possible in terms of the independent variable.
c. Incorrect Random error is controlled by ensuring that participants in different groups are not exposed to different experimental conditions.
d. Incorrect Matching controls systematic bias; it does not increase it.

The correct answer is: trying to control systematic variance.

215
Q

Use of the technique known as meta-analysis entails calculating an “effect size,” which can be best defined as:
Select one:

A. a statistically significant difference between experimental and control groups.
B. a standardized mean difference between experimental and control groups.
C. a zero-order correlation.
D. a partial regression coefficient.

A

The term meta-analysis is used to describe a quantitative integration of research studies involving the calculation and aggregation of “effect size” estimates.

a. Incorrect See explanation for response b.
b. CORRECT There are several ways to calculate an effect size, but a commonly used method involves subtracting the control group mean from the experimental group mean and dividing the result by either the control group standard deviation or the pooled within-group standard deviation. The result is a standard score that indicates the difference between the experimental and control groups in terms of standard deviation units.
c. Incorrect See explanation for response b.
d. Incorrect See explanation for response b.

The correct answer is: a standardized mean difference between experimental and control groups.

216
Q

When a predictor’s reliability coefficient is .75, its criterion-related validity coefficient can be:
Select one:

A.
no less than .75.

B.
no greater than .75.

C.
no less than the square root of .75.

D.
no greater than the square root of .75.

A

Knowing the formula for the relationship between reliability and validity would have allowed you to identify the correct answer to this question.

a. Incorrect See explanation for response d.
b. Incorrect See explanation for response d.
c. Incorrect See explanation for response d.
d. CORRECT As indicated by the formula in the Test Construction chapter of the written study materials, a test’s criterion-related validity coefficient cannot exceed the square root of its reliability coefficient.

The correct answer is: no greater than the square root of .75.

217
Q

To hire only the most qualified job applicants, a personnel director raises the cutoff score on the selection test. Which of the following accurately describes a consequence of changing the cutoff score in this way?
Select one:

A.
The number of true positives is reduced.

B.
The number of false negatives is reduced.

C.
The number of true negatives is reduced.

D.
The number of false positives is increased.

A

Drawing a scatterplot like the one in the Test Construction chapter of the written study materials would have helped you identify the correct answer to this question.

a. CORRECT Moving the predictor cutoff score to the right in the scatterplot (i.e., raising the predictor cutoff) will decrease the number of true and false positives and increase the number of true and false negatives.
b. Incorrect See explanation above.
c. Incorrect See explanation above.
d. Incorrect See explanation above.

The correct answer is: The number of true positives is reduced.

218
Q

Cross-validation is associated with which of the following phenomena?
Select one:

A.
utility analysis

B.
relevance

C.
criterion contamination

D.
shrinkage

A

Cross-validation refers to re-assessing a test’s criterion-related validity with a new sample to determine the generalizability of the original validity coefficient.

a. Incorrect In the context of employee selection, utility analysis is conducted to evaluate the costs and benefits of a selection procedure.
b. Incorrect In test construction, relevance refers to the extent to which test items contribute to achieving the stated goals of testing.
c. Incorrect Criterion contamination refers to the bias introduced into a person’s criterion score as a result of the scorer’s knowledge of the person’s performance on the predictor. Criterion contamination tends to artificially inflate the size of the criterion-related validity coefficient.
d. CORRECT The validity coefficient ordinarily “shrinks” (becomes smaller) on cross-validation because the chance factors operating in the original sample are not present in the cross-validation sample.

The correct answer is: shrinkage

219
Q

On the MMPI-2, a T-score of 70 means that an examinee’s score is at the:
Select one:

A.
68th percentile.

B.
70th percentile.

C.
84th percentile.

D.
97th percentile.

A

The T-distribution has a mean of 50 and a standard deviation of 10, which means that a T-score of 70 is two standard deviations above the mean.

a. Incorrect See explanation for response d.
b. Incorrect See explanation for response d.
c. Incorrect See explanation for response d.
d. CORRECT In a normal distribution (which is assumed for MMPI scores), about 97% of scores fall below the score that is two standard deviations above the mean.

The correct answer is: 97th percentile.

220
Q

Score transformations that alter the rank order and relative size of the distance between scores are referred to as _________ transformations.
Select one:

A.
linear

B.
nonlinear

C.
leptokurtic

D.
platykurtic

A

Raw scores are often transformed in order to simplify their interpretation, and these transformations can be either linear or nonlinear.

a. Incorrect Linear transformations preserve the rank order and relative size of the distance between scores.
b. CORRECT Nonlinear transformations alter the rank order and relative size of the distance between scores. Percentile ranks (which always have a flat distribution regardless of the distribution of the raw scores) are an example of a nonlinear transformation. Logarithms and square roots are other types of nonlinear transformations.
c. Incorrect A leptokurtic distribution is more “peaked” than a normal distribution.
d. Incorrect A platykurtic distribution is flatter than a normal distribution. While the distribution of some transformed scores (e.g., percentile ranks) are flatter than the distribution of raw scores, not all nonlinear transformations produce a platykurtic distribution.

The correct answer is: nonlinear

221
Q

A Wechsler IQ score is a(n):
Select one:

A.
percentile rank.

B.
standard score.

C.
ipsative score.

D.
stanine score.

A

Wechsler Full-Scale IQ scores and Index scores have a mean of 100 and standard deviation of 15, while the subtests have a mean of 10 and standard deviation of 3.
a. Incorrect See explanation for response b.

b. CORRECT An examinee’s score on the Wechsler IQ test indicates his/her performance in terms of the performance of examinees in the standardization sample–and, more specifically, in terms of standard deviation units. For example, a Full Scale IQ score of 115 is one standard deviation above the mean.
c. Incorrect See explanation for response b.
d. Incorrect See explanation for response b.

The correct answer is: standard score.

222
Q

To evaluate the concurrent validity of a new selection test for salespeople, you would:
Select one:

A.
conduct a factor analysis to confirm that the test measures the attributes it was designed to measure.

B.
have sales managers (“subject matter experts”) rate test items for relevance to success as a salesperson.

C.
administer the test to a sample of current salespeople and correlate their scores on the test with their recently assigned performance ratings.

D.
administer the test to salespeople before and after they participate a sales training program and correlate the two sets of scores.

A

Concurrent and predictive validity are types of criterion-related validity.

a. Incorrect This technique would be used to evaluate the test’s construct validity.
b. Incorrect This would help establish the test’s content validity.
c. CORRECT To evaluate a test’s criterion-related validity, scores on the predictor (in this case, the selection test) are correlated with scores on a criterion (measure of job performance). When scores on both measures are obtained at about the same time, they provide information on the test’s concurrent validity.
d. Incorrect This procedure would not provide information on the test’s concurrent validity.

The correct answer is: administer the test to a sample of current salespeople and correlate their scores on the test with their recently assigned performance ratings.

223
Q

In factor analysis, the _______ is the proportion of a test’s total variance that is accounted for by the common (identified) factors.
Select one:

A.
communality

B.
factor loading

C.
principal component

D.
eigenvalue

A

For the exam, you’ll want to be familiar with the definition of all of the terms listed in the answers to this question. Additional information about these terms is provided in the Test Construction chapter of the written study materials.

a. CORRECT Each test included in a factor analysis has a communality, which indicates the total amount of variability in test scores that has been explained by the factor analysis - i.e., by all of the identified factors.
b. Incorrect The factor loading is the correlation between a single test and an identified factor.
c. Incorrect In principal components analysis (which is similar to factor analysis), the principal component is equivalent to a factor.
d. Incorrect In principal components analysis, the eigenvalue indicates the total amount of variability accounted for by each component (factor).

The correct answer is: communality

224
Q

In the context of incremental validity, the positive hit rate is calculated by:
Select one:

A.
dividing the true positives by the false positives.

B.
dividing the true positives by the total positives.

C.
dividing the total positives by the true positives.

D.
dividing the total positives by the false positives.

A

The positive hit rate is the proportion of people who would have been selected on the basis of their predictor scores and were also successful on the criterion.

a. Incorrect See explanation for response b.
b. CORRECT The positive hit rate is calculated by dividing the number of true positives by the total number of positives.
c. Incorrect See explanation for response b.
d. Incorrect See explanation for response b.

The correct answer is: dividing the true positives by the total positives.

225
Q

A “communality” is best described as:
Select one:

A.
the proportion of variance accounted for by a single factor in multiple variables.

B.
the proportion of variance accounted for by multiple factors in a single variable.

C.
the proportion of variance accounted for by multiple factors in multiple variables.

D.
the total proportion of variance in a set of variables that is attributable to error.

A

A communality is a measure of “common variance.”

a. Incorrect See explanation for response b.
b. CORRECT When conducting a factor analysis, a separate communality is calculated for each variable (test). The communality represents the total amount of variability accounted for in a test by the identified factors and is a reflection of the amount of variance that the test has in common with the other measures included in the factor analysis.
c. Incorrect See explanation for response b.
d. Incorrect See explanation for response b.

The correct answer is: the proportion of variance accounted for by multiple factors in a single variable.